You are on page 1of 239

What are the 5 main platelet abnormalities?

1) ITP: peripheral platelet


destruction, antiplatelet antiboides, increased megakayrocytes\n2) TTP:
increased platelet aggregation leads to thrombosis and schistocytes,
increased LDH (There are neurologic & renal symptoms, & fever)\n3) DIC:
schistocytes & increased fibrin split products\n4) Aplastic anemia\n5)
Drugs (e.g. immunosuppressive agents)

A schistocyte or schizocyte

(from Greek schistos for "divided" or schistein for "to split", and kytos for
"hollow" or "cell") is a fragmented part of a red blood cell. \n irregularly
shaped, jagged and asymmetrical. \n a true schistocyte does not have
central pallor.
What are the signs of platelet abnormalities? 1) MICROhemorrhage:
mucous membrane bleeding, epistaxis, petechiae, purpura\n2) increased
bleeding time

Epistaxis = nosebleed\n Petechiae = a small (1

2mm) red or purple spot on the body\n Purpura small (0.31 cm) red or
purple discolorations on the skin that do not blanch on applying pressure.
Caused by bleeding underneath the skin.
What are the 3 main coagulation factor defects?

1) Hemophilia A

(factor VIII deficiency)\n2) Hemophilia B (factor IX deficiency)\n3) vW


disease: mild, mostcommon (deficiency of vonWillebrand factor leads to a
defect in platelet adhesion and a decrease in factor VIII survival)
What are the signs of a coagulation factor defect?

1) MACROhemorrhage:

hemarthroses (bleed into joints), easy bruising\n2) Increased PT &/or PTT


time
What is the initial response to highaltitude induced hypoxia?

1)

HemoglobinO2 curve shifts right as a result of increased 2,3DPG) in


order to release more O2 to tissue\n2) The P50 (partial pressure of O2 @
50% saturation of hemoglobin) will increase
What is one chronic response to highaltitude induced hypoxia? Increased
hemocrit due to increased erythropoietin
What is the action of 2,3DPG? 1) It binds to the beta chains of
deoxyhemoclogin and DECREASES the affinity of hemoglobin for O2\n2)
This rightshifts the HemoglobinO2 curve\n3) Is a response to high
altitude hypoxia

What is the pulmonary vascular response to increased altitude? B/C


barometric pressure decreases, there is a decrease in alveolar partial O2
pressure. \n This causes hypoxic vasoconstriction in the pulmonary
vessels\n This increases pulmonary arterial pressure (i.e. pulmonary
vascular resistance)\n If prolonged = Right heart hypertrophy
What causes acromegaly?

Unsuppressed secretion of growth

hormone (GH), usually due to a pituitary adenoma


What causes the visual disturbances often seen with acromegaly?

If

the cause is a pituitary adenoma, this tumor can press on the optic chiasm
What are the main functions of growth hormone (GH)?

1) Human

growth and development\n2) Stress response to starvation


How does growth hormone (GH) respond to starvation?

GH is released

in response to hypoglycemia & acts directly to REDUCE glucose uptake by


cells & to increase lipolysis inorder to INCREASE blood sugar levels
How is growth hormone (GH) used in the Dx of acromegaly?

In a

healthy person, a glucose tolerance test should depress GH levels. \n


However, in acromegaly, the GH levels would not be affected
What is the growth hormone (GH) response to obesity?

it's secretion is

reduced
What is the growth hormone (GH) response to pregnancy? GH secretion
decreases
What are somatomedins and when are they released?

1)

Somatomedins = insulinlike growth factors\n2) Mediate metabolic


changes necessary for growth & development\n3) Act on hypothalamus &
anterior pituitary via negative feedback to reduce growth hormone (GH)
secretion
What is somatomedin's effect on GH secretion

1) It is part of the

negative feedback system that regulates GH secretion\n2) Synthetic forms


can be used to treat acromegaly
What are the primary findings in acromegaly?

1) Cause: excess GH in

adults\n2) Features:\n Large tongue with deep furrows\n Deep voice\n

Lg. hands & feet\n Coarse facial features\n Impaired glucose tolerance
(insulin resistance)
What does excess GH cause in adults? In children? In adults:
acromegaly\nIn kids: gigantism
What is the Tx for acromegaly? Pituitary adenoma resection followed by
octreotide administration Octreotide: mimics natural somatostatin
pharmacologically; a more potent inhibitor of growth hormone, glucagon,
& insulin than the natural hormone.
How is acromegaly diagnosed? 1) Increased serum IGF1\n2) Failure to
suppress serum GH following oral glucose tolerance test
When is growth hormone (GH) normally increased? Stress, exercise, &
hypoglycemia
How does acetazolamide work? Inhibits carbonic anhydrase in the PCT
(luminal/basolateral membrane & cytoplasm) \n t/f decreases kidney
reabsorption of bicarbonate in the proximal tubule & increases K+, Na+, &
Cl loss\n ALKALINIZES Urine

ACIDazolamide causes ACIDosis

What is acetazolamide used for?ACE family:\n Altitude sickness\n


metabolic Alkalosis\n Alkalinizing urine \n glauComa\n Epilpsy\n
Familial periodic paralysis
What is the potential toxicity of acetazolamide?

PACE:\n Potassion

depletion (hypokalemia)\n Acidosis (Hyperchloremic metabolic) due to


excretion of HCO3\n Allergy (SULFAbased)\n Ca nephrolithiasis\n
Encephalopathy in patients with hepatic cirrhosis due to decreased NH4+
excretion
What drugs increase urine NaCl?

ALL diuretics:\n Carbonic

anhydrase inhibitors\n Loops\n Thiazides\n K+sparing


Which drugs increase urine K+ All diuretics, except K+ sparing
Which diuretics cause acidemia?1) Carbonic anhydrase inhibitors\n
reduced HCO3 reabsorbtion\n2) K+ sparing\n hyperkalemia leads to K+
entering all cells (via H+/K+ exhcanger) & H+ exiting cells

Which diuretics cause alkalemia?

Loops & Thaizides:\n1) "Contraction

Alkalosis": Volume contraction > increased ATII > Na+/H+ exchange in


proximal tubule> increased HCO3 \n2) K+ loss leads to: K+ exiting all
cells (via H+/K+ exchanger) & H+ enters cells\n3) "Paradoxical aciduria":
In a low K+ state, H+ (rather than K+) is exchanged for Na+ in principle
cells, leading to alkalosis
Which diuretics cause an increase in urine Ca2+?

1) Loops\n2)

Mechanism:\n Abolish lumenpositive potential in thick ascending limb of


loop of Henle > decrease in paracellular Ca+ reabsorption >
hypocalcemia
Which diuretics cause a decrease in urine Ca2+?

1) Thiazides\n2)

Mechanism:\n Block luminal Na+/Cl cotransport in DCT > increased


Na+ gradient > increased interstitial Na+/Ca2+ exchange >
hypercalcemia
In acute renal failure, what is the indication for dialysis?

1) Severe

hyperkalemia (elevated blood level of potassium) \n2) A rapidlyrising


potassium value\n3) Severe acidosis\n4) BUN >80 mg/dl or S/S uremia\n5)
Na Concentration <120mEq/L or >155mEq/L\n6) Toxins: lithium or
salicylates
What is a broad definition of acute renal failure?

Abrupt decline in renal

function + increased creatinine and BUN over a short period of several


days
What are the 3 stages of acute renal failure? 1) prerenal azotemia\n2)
intrinsic renal\n3) postrenal
What are the characteristics of prerenal azotemia? 1) Decreased RBF (e.g.
hypotension)\n2) Reduced GFR\n3) Na+/H2O & urea retained by the
kidney
What are the characteristic lab values of prerenal azotemia?

1) Urine

osmolality: >500\n2) Urine Na: <10\n3) Fe of Na: <1%\n***4) BUN/Cr


ratio: >20

What are the characteristic lab values of the intrinsic renal stage of acute
renal failure?1) Urine osmolality: <350\n2) Urine Na: >20\n3) Fe of Na:
>2%\n4) BUN/Cr ratio: <15
What are the characteristic lab values of the postrenal stage of acute renal
failure?

1) Urine osmolality: <350\n2) Urine Na: >40\n3) Fe of Na:

>4%\n4) BUN/Cr ratio: >15


What are the characteristics of the intrinsic renal stage of acute renal
failure?

1) Causes: acute tubular necrosis or ischemia/toxins. \n2)

Patchy necrosis leads to debris obstructing tubule & fluid backflow across
necrotic tubule leads to a decrease in GFR\n3) Urine has
epithelial/granular casts
What are the characteristics of the postrenal stage of acute renal failure?
1) Cause: outflow obstruction (stones, BPH, neoplasia)\n2) NB: ONLY
develops with bilatral obstruction
What are the 2 forms of renal failure? 1) Acute: usually due to acute
tubular necrosis\n2) Chronic: usually caused by hypertension & diabetes
What are the 8 possible consequences of renal failure?

1) Anemia:

failure of erthropoietin production\n2) Renal osteodystophy: no vit. D


production\n3) Hyperkalemia: possible cardia arrhythmias\n4) Metabolic
acidosis: decreased acid secretion & reduced HCO3 production\n5)
Uremic encephalopathy\n6) Na & H20 excess: CHF & pulmonary
edema\n7) Chronic pyelonephritis\n8) Hypertension
What is uremia?

A clinical syndrome marked by increased BUN &

creatinine & associated symptoms


What type of exchange limitation is caused by emphysema?

1)

Diffusionlimited exchange\n2) Destruction of alveoli leads to a decreased


surface area for gas exchange\n3) T/F increased perfusion will not improve
the exchange
In a diffusionlimited situation, will O2 fully equilibrate along the length of
the pulmonary capillary? 1) No.\n2) As blood is oxygenated, the gradient
driving O2 into the blood decreases.\n3) T/F further equilibration would

take more time &, in a diffusionlimited situation, there is not enough time
& O2 will not equilibrate by the time it reaches the end of the capillary.
What gasses are perfusionlimited?

1) O2 (in normal health)\n2)

CO2\n3) N2O
What does it mean for a gas to be perfusionlimited?

1) Gas

equilibrates early along the length of a pulmonary capillary\n2) Diffusion


can be increased ONLY with an indrease in blood flow
What does it mean for a gas to be diffusionlimited?Gas does not
equilibrate by the time the blood reaches the end of the pulmonary
capillary
What gasses are diffusionlimited?

1) O2 (emphysema & fibrosis)\n2)

CO
What is the most severe consequence of pulmonary hypertension?

Cor

pulmonale & RV failure (JV distention, edema, hepatomegaly)


How is the diffusion equation affected by emphysema?

Decrease in

Area
How is the diffusion equation affected by pulmonary fibrosis?

Increase

in Thickness
Give the diffusion equation.

Vgas = A/T x Dk(P1P2)\n\nA = area\nT =

thickness\nDk(P1P2) = difference in partial pressures


What are the pulmonary function findings in a case of COPD?
Obstructive Pattern:\nIncrease in RV\nDecrease in FVC\nDecrease in
FEV1\nDecrease FVC\nDecrease FVC1/FVC ration (**)\nV/Q mismatch
What is the pathology of Chronic Bronchitis (Blue Bloater)?

1)

Hypertrophy of mucussecreting glands in the bronchioles (small


airways)\n2) Reid index >50\n3) Productive cough for > 3 consecutive
months in 2(+) years\n4) Findings: wheezing, crackles, cyanosis (early
onset hypoxemia due to shunting), lateonset dyspnea
gland depth / total thickness of bronchial wall

Reid index =

What are the types of emphysema?

1) Centriacinar: smoking\n2)

Panacinar: alpha1antitrypsin deficiency (or liver cirrhosis)\n3)


Paraseptal: bullae that rupture causing pneumothorax (often = otherwise
healthy, young males)
What is the pathology of Emphysema (Pink Puffer with barrelshaped
chest)?

1) Enlargement of air spaces and decrease in recoil resulting

from destruction of alveolar walls\n2) Increased elastase activity\n3)


Increased compliance due to loss of elastic fibers\n4) Px exhale through
pursed lips to increase airway pressure and prevent airway collapse during
exhalation
What are the clinical findings in Emphysema? Dyspnea\n Decreased
breath sounds\n Tachycardia\n Lateonset hypoxemia due to eventual
loss of capillary beds (loss of alveolar walls)\n Earlyonset dyspnea
What is the pathology of asthma?

1) Bronchial hyperresponsiveness

causes reversible bronchoconstriction\n2) Smooth muscle hypertrophy &


Curschmann's spirals (shed epithelium from mucous plugs)
What triggers asthma?

Viral URIs\n Allergens\n Stress

What are the clinical findings of asthma?

Cough, wheezing,

dyspnea\n Tachypnea\n Hypoxemia\n Reduced I/E ratio\n Pulsus


paradoxus\n Mucus plugging
What is the pathology of Bronchiectasis?

Chronic necrotizing infection

of bronchi leads to permanently dilated airways, purulent sputum,


recurrent infections, and hemoptysis
What is Bronchiectasis associated with?

Bronchial obstruction\n

CF\n Poor ciliary motility\n Kartagener's Syndrome\n Development of


Aspergillosis
What does an isolated prolonged PTT indicate?

One of the

following:\n1) Factor XI, IX, & VIII deficiency\n2) Presence of factor VIII
inhibitor\n3) Use of Heparin\n4) Presence of lupus anticoagulant

If the PT test is prolonged, what other test will automatically be abnormal?


the INR will be prolonged as well (that is, what is monitored in
Warfarin Therapy
What does a prolonged PT (& INR) indicate?

One of the following:\n1)

Warfarin therapy\n2) Liver Disease\n3) Factor VII deficiency\n4) Vitamin K


deficiency
What does a combination of prolonged PT, INR, and bleeding time
indicate?

One of the following:\n1) Warfarin therapy\n2) Liver

Disease\n3) Factor VII deficiency\n4) Vitamin K deficiency\n5) vWF


deficiency\n\nNB: the same as PT & INR, + vWF for platelet
What does prolongation of all coagulation parameters indicate? vWF
deficiency or DIC
What are the lab test results that indicate thrombocytopenia?

1)

Reduced Platlet Count\n2) Increased Bleeding time\n3) NORMAL PT & PTT


What are the lab test results that indicate Hemophilia A or B?

All

normal, except increased PTT


What are the lab test results that indicate von Willebrand disease?

1)

Increased Bleeding Time\n2) Increased PTT


What are the lab test results that indicate DIC?

1) Decreased: platlet

count \n2) Increased: PT, PTT, & Bleeding Time


What are the lab test results that indicate Vitamin K deficiency? Increased
PT & PTT
What are the lab test results that indicate BernardSoulier disease?
Decreased platelet count\n2) Increased bleeding time

1)

BernardSoulier

Disease: defect of platelet adhesion (reduced GP Ib)


What are the lab test results that indicate Glanzmann's thrombasthenia?
1) Increased bleeding time

Glanzmann's Thrombasthenia =

defect of platelet aGgredation (reduced GP IIbIIIa)


What factors are tested by PT? Extrinsic factors: II, V, VII, & X
What factors are tested by PTT? Intrinsic: ALL, except VII

What are Lewy bodies?

1) Pink inclusions (aggregation of alphasynclein

proteins) in neurons of the substantia nigra \n2) Patients with Parkinson's


or Lewy body dementia
What are the clinical characteristics of Parkinson's? TRAP\n1) Tremor (At
rest)\n2) cogwheel Rigidity\n3) Postural instability\n\n4) Bradykinesia
bradykinesia: "slow movement", a slowness in the execution of
movement.
What disease classically presents with myoclonic fasciculations?
CreutzfeldJacob

1) Myoclonus: sudden, involuntary jerking of a

muscle or group of muscles. \n2) Fasciculation (or "muscle twitch"): a


small, local, involuntary muscle contraction (twitching) visible under the
skin arising from the spontaneous discharge of a bundle of skeletal muscle
fibers.
What type of muscle weakness is seen in GuillainBarre syndrome and
amyotrophic lateral sclerosis?

symmetrical

What types of diseases present with tongue fasciculations?


degenerative neurological diseases (e.g. spinal muscular atrophy &
amyotrophic lateral sclerosis)
What is the pathology of Parkinson's disease?

1) A degenerative

disorder of the CNS\n2) Histo: Lewy bodies (alphasynuclein)\n3) Gross:


depigmentation of the substantia nigra pars compacta (loss of
dopaminergic neurons)\n4) Rare cases linked with exposure to MPTP (a
contaminnant in illicit street drugs)
Define "aphasia"

Higherorder inability to speak

Define "dysarthria" motor inability to speak


Define Broca's aphasia.

Broca's Broken Boca:\n\n1) Nonfluent aphasia

with intact comprehension\n2) Broca's area: infereior frontal gyrus


Define Wernicke's aphasia.

Wernicke's is Wordy buy makes no sense:

Wernicke's = "What?"\n\n1) Fluent aphasia with impaired


comprehension\n2) Wernicke's area: superior temporal gyrus

Define Global aphasia.

1) Nonfluent aphasia with impaired

comprehension\n2) Both Broca's and Wernicke's areas are affected


Define Conduction aphasia.

1) Poor repetition, but fluent speech\n2)

Intact comprehension\n3) Arcuate fasciculus: connects Broca's &


Wernicke's areas
Where is a lumbar puncture taken?

Between L4 & L5 (Level of iliac

crests)
What structures are pierced in a lumbar puncture? 1) Skin/Superficial
fascia\n2) Ligaments (Supraspinous, interspinous, ligamentum flavum)\n3)
Epidural space\n4) Dura mater\n5) Subdural space\n6) Arachnoid\n\nNOT
PIA: Pia is not Pierced
What is the likely tumor at the epiphyseal end of the femur in a 30year
old woman? Giant cell: \n1) Benign, primary bone tumor, aggressive\n2)
Radiography: double bubble or soap bubble sign\n3) Histo: oval or spindle
shaped cells along with scattered multinucleated giant cells\n4)
Epidemiology: 2040 year old, usually women
What patients are most susceptible to chondrosarcoma?

1) Older men

(3060)\n2) Malignant cartilaginous tumor of the pelvis, spine, scapula,


humerus, tibia, or femur.\n3) May be primary bone or from an
osteochrondroma \n4) Gross: expansile glistening mass within the
medullary cavity
What is the most common benign bone tumor?

osteochondroma: it

occurs in men less than 25 years old, in the metaphysis of long tubular
bones
What is Ewing's sarcoma? Going out for Ewings & onion rings!\n\n1)
Aggressive malignant bone tumor; early met but responsive to chemo\n2)
Epidemiology: boys less than 15 years old\n3) Histo: lamellated, "onion
skin" periosteal reaction: uniform sheets of anaplastic, small blue cells\n4)
Gross: DIAPHYSIS of bones, pelvis, scapula, ribs\n5) 11:22 translocation
What is osteosarcoma?

2) Epidemiology: Males, 1020 years old\n3)

Gross: METAPHYSIS of long bones\n4) Radiography: 2 growth patterns:\n

Codman's Triangle: elevated periosteum \n Sunburst growth pattern:


reactive bone formation
List the 6 benign bone tumors. 1) Osteoma\n2) Osteoid Osteoma\n3)
Osteoblastoma\n4) Giant Cell (Osteoclastoma)\n5) Osteochondroma
(exostosis)\n6) Enchondroma
List the 3 malignant bone tumors.

1) Osteosarcoma (osteogenic

sarcoma)\n2) Ewing's Sarcoma\n3) Chondrosarcoma


What is an osteoma?

1) Benign bone tumor\n2) New pieces of bone

grows on another piece of bone, often in the SKULL\n3) Associated with


Gardner's syndrome (FAP)
What is an osteoid osteoma?

1) Benign bone tumor\n2) Interlacing

trabeculae of woven bone are surrounded by osteoblasts\n3) <2cm;


proximal tibia or femur\n4) Men <25 years old
What is an osteoblastoma?

1) Benign bone tumor\n2) Interlacing

trabeculae of woven bone are surrounded by osteoblasts\n3) can be large;


vertebral column\n4) 2040 year old Px
What is a Giant Cell tumor?

1) Locally aggressive benign tumor often

around the distal femur, proximal tibial region (KNEE)\n2) Radiology:


"double bubble" or "soap bubble"\n3) Spindleshaped cells with
mutinucleated giant cells\n4) 2040 year old Px
What is a Osteochondroma?

1) MOST COMMON BENIGN bone

tumor\n2) Mature bone with cartilaginous cap\n3) Males <25 years of


age\n4) Origin: long metaphysis
What is an enchondroma?1) Benign cartilaginous neoplasm found in
intramedullary bone\n2) Usually DISTAL extremities
What is an osteosarcoma?

1) 2nd most common primary malignant

tumor of bone (after multiple myeloma)\n2) Peat incidence: males 1020


years old\n3) METAPHYSIS of long bones: distal femur, proximal tibial
region (knee)\n4) Predisposing factors: Paget's disease of bone, bone
infarcts, radiation, familial retinoblastoma\n5) Radiography: Codman's
triangle or sunburst pattern \n6) Poor prognosis

What is the mechanism of albuterol?

Increases conversion of cAMP by

adenylate cyclase:\n activates B2adrenergic receptors on smooth


muscle cells to activate adenylate cyclase. \n Adenylate cyclase then
converts ATP to cAMP\n SMOOTH MUSCLES RELAX
What lymphoma presents with aggressive onset of thrombocytopenia,
anemia, and a lg. mediastinal mass?

ALL (Acute Lymphoblastic

Lymphoma):\n lymphoblasts in blood and bone marrow\n Epidemiology:


children < 15 years old
What makes a pneumonia "atypical"? 1) insidious onset\n2)
Nonproductive cough\n3) streaky interstitial infiltrate on chest
radiograph\n4) Most common cause: Mycoplasma (confirm with cold
agglutinin test): contains cholesterol in membrane\n5) Rx: Erythromycin or
tetracyclin (b/c NO CELL WALL)
What is the pathogenesis of acute intermittent porphyria (AIP)? 1)
Deficiency is porphobilinogen deaminase\n leads to a deficiency in heme
synthesis & buildup of intermediates like aminolevulinic acid &
porphobilinogen\n2) Acute attacks of GI, neurological/psychiatric, & heart
symptoms\n3) Onset after puberty, more common in women\n4)
Symptoms can be brought on by drugs that INDUCE cytochrome P450
What drug prevents growth, survival, and division of Estrogenpositive
cancer cells and is thereby prescribed to certain breast cancer patients?
Raloxifene: it also increases bone mass
What is the expected respiratory carbon dioxide change expected for
every 1mEq/L bicarbonate increase? The PCO2 should increase by
0.7mm Hg per 1mEq/L increase in bicarbonate. \n A normal bicarbonate
is 24 mEq/L & a normal PCO2 is 40 mmHG.
What are the causes of secondary cardiac tamponade?

1) Trauma\n2)

hypothyroidism\n3) Myocardial rupture\n3) Complication of pericarditis


(esp. due to malignancy or uremia)
What is Beck's Triad?

Signs of cardiac tamponade:\n1) Distant heart

sounds\n2) Increased JVP\n3) Hypotension\n\nAlso, pulsus paradoxus:


systolic Bp drops by >10mmHg on inspiration

What is the best treatment for cardiac tamponade? 1) First: Immediate


volume resuscitation (IV fluids) to maintain cardiac output & increase
preload\n2) 2nd: Positive iontropes (dobutamine & pericardiocentesis)
What drug has the side effect of a brown darkening of the iris?
Latanoprost: prostaglandin prodrug used to increase the outflow of
aqueous humor in glaucoma or increased intraocular pressure\n side
effect occurs in 10% of patients
What is Wallenberg's syndrome?AKA: lateral medullary syndrome:\n1)
Stroke: dissection & thrombosis of vertebral artery & posterior inferior
cerebellar artery
What symptoms indicate an infarction of the lateral medulla?

1) Spinal

trigeminal nucleus: loss of pinprick sensation on the right face\n2)


Spinothalamic track: loss of pinprick sensation on the left below the
neck\n3) Inferior cerebellar peduncle: ataxia\n4) Vestibular nucleus:
nystagmus\n5) Dorsal motor nucleus of vagus: hoarsness\n6) Descending
sympathetic fibers: Horner's syndrome (ptosis, miosis, anhidrosis)
Where in the medulla do vibratory & proprioceptive info course? Medial:
medial lemniscus
What paraneoplastic syndrome is produced by small cell carcinoma?
Cushing's syndrome (ectopic ACTH)
What paraneoplastic syndrome is caused by squamous cell carcinoma of
the lungs?

Hypercalcemia: cells produce parathyroid homonerelated

peptide.
What type of drugs for CHF cause a urinalysis with the following
findings:\n pH 5.5\n increased NaCl, K, Ca concentrations

Loops

(like furosemide): Loops Lose calcium, stimulate acidification by increasing


distal Na delivery & transport (this causes a favorable electric gradient for
K & H+ secretion)
Can there be "carriers" of AD diseases?

No.

In diseases that cause a decrease in intestinal cells, what step of lipid


digestiong & absorbtion is impaired?

Chylomicron formation, this step

occurs within the intestinal cells (reestrification)


What cause croup? Parainfluenza virus is a common cause
What is arcus lipoides

Opaque rings found on the endge of the iris that

indicate type IIa familial dyslipidemia (familial hypercholesterolemia)


What is the difference in prognosis for those heterozygous and
homozygous patients with familial hypercholesterolemia? This disease is
AD:\n1) Homozygous: MI in 1st decade of life\n2) Heterozygous: elevated
LDL that may manifest in middle age
What brain tumor is a feature of vonHippelLindau disease?
Hemangioblastoma (benign) of the cerebellum or retina:\n Patients
present with headache\n Findings include papilledema, nystagmus, &
ataxia (& 2ndary polycytemia due to epo secretion by the tumor)\n Tx:
surgical resection
What is the cause of von HippelLindau disease?

AD disease that

results in a deletion of a tumor suppressor gene on Chromosome 3\n


Features: cysts of kidney, pancrease, polycytemia, hemangioblastoma of
cerebellum or retina
What are the signs of chronic vitamin A (retinol toxicity)? Ataxia\n
Alopeica\n Headache\n Fatigue\n Bone & joint pain\n Muscle pain\n
Visual disturbances\n Hyperlipidemia\n Hepatotoxicity\n Dry mucous
membranes\n Lip Fissures
What are the signs of acute vitamin A (retinol toxicity)?

1) Nausea\n2)

Vomiting\n3) Vertigo\n4) Blurry vision\n\nSevere: drowsiness & altered


mental status
What is the pathogenesis of the allergic reaction to aspirin increase
leukotriene syntheses leads to bronchoconstriction: b/c the
cyclooxygenasecatalyzed prostaglandin pathway is blocked

What drug can stop the bronchoconstriction due to aspirin allergy?


Zileuton: Inhibits the 5lipoxygenase pathway to block the
conversion of arachidonic acid to leukotrienes
What are the most common arteries targed by peripheral atherosclerosis?
1) abdominal aorta\n2) Iliac\n3) Femoral\n4) Popliteal (can present
with intermittent claudication)\n5) tibial\n6) popliteal
What is the mechanism of action of zanamivir?

Inhibits neuraminidase

enzyme that is critical to the influenza life cycle (A & B) & blocks the
release of progeny virons as there is no cleavage of neuraminic acid to
release the viron with host cell membrane cover.
What cell types and cytokines drive asthma? 1) hyperresponsive T
lymphocytes in the lung (esp. Th2)\n2) Il4, Il5, Il3
What are the main causes of eosinophilia?

Asthma\n Neoplasms\n

Parasites\n Collagen vascular diseases\n Allergic processes


What is the microscopic pathology of hypertrophic obstructive
cardiomyopathy (or asymmetric septal hypertrophy and idiopathic
hypertrophic subaortic stenosis) AD in 50% of cases:\nmyofiber
hypertrophy & disarray
What triad indicates classic carcinoid syndrome?

1) Chronic

diarrhea\n2) Intermittent facial flushing\n3) Tricuspid stenosis murmur


Under what conditions are carcinoid syndrome present?

When there is a

liver metastases of a GI carcinoid (b/c then the liver cannot metabolize the
serotonin produced within the liver)
What is the pathogenesis of the right heart valvular disease of cardinoid
syndrome?

Serotoninmediated fibroelastosis

What urine test finding is diagnostic of carcinoid syndrome?

Elevated

5HIAA
Which viruses establish latency in the dorsal root gangila? 1) Herpes
simplex 1 & 2\n2) Varicellazoster

What is one of the 1st signs seen in a postMI biopsy within 12 hours of
the infarction?

Contraction bands @ the margin of the necrotic area

due to elevated intracellular calcium & the resultant hypercontraction.


What is the principle function of the anterior cruciate ligament? To
prevent anterior subluxation (misalignment) of the knee relative to the
femur anterior in the fully extended position.
How does hydroxyurea help sickle cell patients?

It increases fetal

hemoglobin production in place of hemoglobin S & thereby reduces RBC


sickling and complications there of.
What infects does the spleen clear? What if the spleen is gone? 1)
Encapsulated bacteria\n2) Prophylaxis with fluorquinolones and macrolides
What are the 2 major clinical features of nephrotic syndrome?

1)

Edema: low plasma oncotic pressue due to protein wasting & sodium
retention from the collecting tubules\n2) Massive proteinuria,
hypoalbuminemia, hypercholesterolemia
What is Ortner's syndrome?

An enlarged left atrium compresses the

left recurrent laryngeal nerve:\n Causes hoarseness\n Is a branch of the


vagus & innervates on "glossus", the palatoglossus
What muscles does the hypoglossal nerve innervate?

All intrinsic &

extrinsic tongue muscles (except the palatoglossus, which is innervated by


the vagus & helps to raise the back of the tongue)
Explain the process of serum sickness? What type of immune reaction is
it?

Type III: immune complex deposition (in skin: rash, in kidney:

glomerulonephritis) \n Humoral immune response against foreign protein:


710 days later as antibodies must be produced (requires Thelper cells)
What is the toxicity of oligomycin?

This antibiotic blocks ATP synthesis

by binding to the stalk of the ATP synthases. T/F it blocks protein reentry
to the matrix.
What is the toxicity of 2.4 Dinitrophenol?
that results in decreased ATP

It is an uncoupling protein

What is the toxicity of Antimycin A?

Binds to cytochrome c reductase &

prevents further continuation of the electron transport chain in ATP


synthesis.
What is the toxicity of Cyanide? Binds the Fe3+ of cytochrome oxidase a3
& prevents the last enzyme of oxidative phosphorylation & ATP
production?
What is Rotenone's toxicity?

Inhibits transfer of electrons to ubiquinone

& thus stops ATP production


How does E. coli acquire the Shigalike toxin that can cause bloody
diarrhea & hemolyticuremic syndrome?

specialized tranduction

transfer: a lysogenic bacteriophage incorporates foreign DNA into its host.


What is conjugation?

1 bac extends its mating pili to another without

the F plasmid & transfers this plasmid to the 2nd bac (& maybe also some
adjacent DNA)
What is bacterial transformation?

DNA is taken up directly from the

environment by a bac
What is bacterial transposition? DNA segments can jumpt to various
locations within a genome as well as in a plasmid & can then by passed
on.
What does the trochlear nerve innervate?

IV: superior oblique muscle.

Injury leads to CN IV palsy: diplopia that worsens when they look down and
away from the affected side
What is the first line drug choice for BPH & what is that drug's mechanism
of action?

Tamsulosin:\n Smooth muscle tension is mediated by alpha

1adrenergic receptors and these are selectively blocked.


How do integrases treat HIV?

Prevents the formation of a proviral

infection
What is the mechanism of the gramnegative rod Bordetella pertussis's
toxin? The A subunit inhibits membranebound Gi proteins that results in
the accumulation of cAMP. \n This results in histamine sensitization,

increased insulin synthesis, lymphocytosis, and the inhibition of


phagocytosis.\n Result: whooping cough
What is the mechanism of the alpha toxin of S. aureus and Streptolysin O
of S. pyrogens?

Binds to the membrane and allows ions and small

molecules to enter the cell. This leads to swelling & eventual cell lysis.
What is the mechanism of action of C. diptheriae's exotoxin & P.
aeruginosa's exotoxin A? Inactivates elongation factor 2, causing
pharyngitis & the pseudomembrane of the throat.\n\nRx: antitoxin (to
prevent membrane spread & airway obstruction), penicillin or
erthyromycin & DTP vaccine
What is the mechanism of action of C. botulinum's toxin? Inhibition of the
release of acetylcholine into the synaptic cleft. This leads to muscle
weakness and paralysis?
What is an endotoxin?

Lipopolysaccharidelipid A, a part of the

bacterial membrane of all gramnegative bac (except L. monocytogenes)


What is a common extraintestinal manifestation of Chron's disease?
migratory polyartheritis
Which has a higher cancer risk: UC or Crohn's?

Ulcerative colitis

Which disease almost always affects the rectum: UC or Crohn's

UC: 50%

of UC cases are even limited exclusively to the rectum


Which condition is associated with sclerosing cholangitis: UC or Crohn's UC.
This results in dilation of the biliary ducts, fibrosis of the liver & eventual
hepatic failure.
What is another name for GuillainBarre syndrome? acute inflammatory
demyelinating polyradiculopathy.\n\nNB: radiculopathy: disease of spinal
nerve roots and spinal nerves
What is the histological findings in GuillainBarre syndrome?

Segmental demyelination of areas of peripheral nerves (possibly including


inflammation of the axons themselves)\n This results in slower
conduction velocity of action potentials through peripheral nerves & spinal
nerve roots

How do you calculate Mean Corpuscular volume (MCV)?


[Hematocrit/RBC count] x 10
How can you histologically distinguish iron deficiency anemai from a
thalassemia?

Thalassemia will have target cells\n Both will be

microcytic & hypochromatic


What is the Mentzer index?

RBC MCV/RBC\n < 13 in Px with iron

deficiency anemia & <13 in patients with thalassemia


What Mentzer index values distinguish iron deficiency anemia from
thalassemia?

> 13 is iron deficiency\n< 13 is thalasemia

What is LambertEaton syndrome?

Oat (small) cell carcinoma produces

antibodies against presynaptic Ca channels @ the neuromuscular junction:


\n proximal muscle weakness\n decreased deep tendon reflexes\n
diplopia
What is another name for nonbacterial thrombotic endocarditis? Libman
Sacks endocarditis (a symptom of SLE): \n single or multiple 1 to 3mm
verrucous deposits on the edge of either surface of the valve leaflets on
any heart valve \n Usually, mitral, aortic, & tricuspid\n Histo: immune
complexes, mononuclear cells, hematoxylin bodies, fibrin, & platelet
thrombi\n Healing = fibrosis, scarring, & calcification = valve damage,
regurg, & predisposition to bac. endocarditis\n fibrin & platelet thrombi
may embolize systemically
What is the Whipple triad & what does it indicate?

1) episodic

hypoglycemia, CNA symptoms (confusion, stupor, convulsions, & coma), &


reversal of CNS symptoms with glucose\n2) insulinoma (islet; that is, B cell
tumor), either benign or malignant = elevated C peptide & insulin levels
What are the characteristic symptoms of Hodgkin's lymphoma? Night
sweats, fever, & weight loss\n Also, mediastinal lymphadenopathy\n
Histo: ReedSternberg cells: cells with large, "owl eye" binucleate cells;
and reactive lymphocytes
What is the shortterm compensation for highaltitude hypoxia? An
increase in 2,3diphosphoglycerate (2,3DPG) to release more O2 to
tissues)\n This causes an increase in the P50 (partial pressure of O2 at

50% saturation of hemoglobin) and a right shift in the hemoglobinO2


curve
What are the principle functions of thyroid hormone in development?
Bone Growth\nCNA maturation
What are the primary functions of somatostatin?

1) Inhibits all GI &

pancreatic exocrine & endocrine functions & inhibits the release of


insulin\n2) Inhibits GH & prolactin release by the pituitary
What are the side effects of octreotide (exogenous somatostatin)
administration?

Nausea\nVomiting\nDiarrhea\nGallstonerelated illness

What are the characteristics of Renal Tubular Acidosis (RTA)?

Non

anion gap, hyperchloremic, metabolic acidosis


What problem is associated with ngiotensinconverting enzyme inhibitors,
timethoprim, and heparin?

RTA IV: inability to secrete sufficient acid

into the urine due to hypoaldosteronism\n hypoaldosteronism leads to


decreased Na reuptake in the distal convoluted tubule & collecting duct\n
This decreases excretion of K into the urine = kyperkalemia\n
Hyperkalemia results into H+ ion shift to the extracellular fluid to maintain
a K balance = metabolic acidosis\n The alkalotic environment of the renal
tubule cells = low ammonia & H+ production & secretion. T/F the kidney
cannot secrete the excess bodily acid\n Low ammonia = unbuffered
urine & a low pH (below 5.5)
What are the stimuli for insulin secretion?

1) Hyperglycemia (primary

stimulus)\n2) Increase in fatty acids in the blood\n3) Increase in amino


acids in the blood\n4) Cortisol\n5) GH\n6) Gastrointestinal inhibitory
peptide
What causes a physiological rightshift of the OxygenHemoglobin curve
in exercising persons?

Right shift = decrease affinity of hemoglobin for

O2 & more tissueunloading of O2:\n\n1) Decrease: tissue pH (up H+


concentration)\n\n2) Increase: Temperature & partial pressure of CO2
Why are excercising patients not loosing too much CO2 due to increased
respiration? Tissue CO2 production due to aerobic respiration

What is the formula for physiologic dead space (Vd)?

tidal vlume x

([arterial CO2 pressure expired air CO2 pressure] divided by the arterial
CO2 pressure\n\nor\n\nVt x ([PaCO2 PeCO2] / PaCO2)
What is the equation for renal blood flow? What can be done to increase
flow? 1) Flow = (change in pressure) / resistance\n2) Decreasing
resistance will increase flow:\n low levels of dpamine dilate renal
arterioles\n bradykinin induces vasodilation of arterioles
What is the equation for renal blood flow? What can be done to decrease
flow? 1) Flow = (change in pressure) / resistance\n2) Increasing resistance
will decrease flow:\n Stimulating afferant alpha1adrenergic receptors will
vasoconstrict the arteriole\n Stimulating afferant angiotension II receptors
will vasoconstrict the arteriole
What is the iodine status of a pregnant woman?

Relative deficiency:

due to increased renal clearance


What lab values indicate an isolated respiratory alkalosis? low CO2 and
low HCO3 levels
What can cause an isolated respiratory alkalosis?

hyperventilation due

to anxiety/panic/pain (sympathetic stimulation) or high altitude or low O2


What type of limited gas is O2, diffusion or perfusion?

Perfusion: gas

exchange can only increase or decrease with an increase or decrease in


blood flow.
What hormone maintains lactation?

Prolactin from the anterior pituitary

How does lactation prevent ovulation? Inhibits secretion of GnRH


(Gonadotropinreleasing hormone) from the hypothalamus.\n This inhibits
secretion of LH and FSH from the anterior pituitary
What are 2 common causes of hyperprolactinemia? 1) Tumors\n2)
Antipsychotic drugs (dopamine antagonists = increase prolactin as
dopamine usually inhibits prolactin release)
Is pepsin a high or low pH active enzyme?

Low; if antacids or used

protein digestion can be impaired and patients will often have constipation

What are the steps to DNA binding from circulation that a steroid hormone
undergoes? 1) Enters cell membrane (lipophilic)\n2) Binds to an
intracellular receptor in the cytoplasm or within the nucleus\n3) The
hormonereceptor complex transforms to reveal the hormone's DNA
binding domain\n4) The hormone binds to the DNA enhancer element &
generates gene transcription.
What respiratory cause of hypoxemia cannot be overcome by supplimental
O2?

A shunt, e.g. alveolar collapse, where blood flows through some

capillaries without any gas exchange


Which respiration problems result in a large Aa gradient? 1) Shunt\n2)
ventilationperfusion mismatch\n3) Diffusion impairment
How does imipramine change a patient's sleep pattern?

Decrease in

stage 4 or slow wave (delta: large & slow wave) sleep. \n Stage 3 &4 is
when disorders including night terrors, sleepwalking, and enuresis take
place.
How do central chemoreceptors respond to blood pH and partial gas
pressures?

1) Increase PCO2 or decreased pH = increase in total

peripheral resistance\n2) Mechanism: systemic vasoconstriction via


sympathetic outflow
How does the Na+/H+ exchanger in the proximal tubule facilitate Na &
water resorption?

1) H+ is pumped into the tubual lumen\n2) Luminal H+

then returns to the tubular cell in the process of HCO3 resorption:\n H+


& HCO3 join to form H2O via carbonic anhydrase at the brush boarder\n
H20 & CO2 diffuse into the tubular cell\n3) HCO2 is regenerated in the
tubular cell (& sent to the bloodstream) & H+ is left behind to participate
in the exchanger again.
What is the physiology of high fat food pain in patients with gallstones?
CCK (cholecystokinin) is released in the duodenum after a fatty
meal (& amino acids) to induce gallbladder contraction & thus the stone
obstruction causes pain.
Which Coombs test detects antibodies attached to RBCs & which detects
antibodies circulating in serum? Direct: antibodies on RBCs\nIndirect:
antibodies in serum

What is the correct order of MI blood test markers? 1) Troponin I (elevated


w/in 4 hours; remains for 710 days)\n2) CKMB (peaks w/in 24 hours &
falls off) best for monitoring postMI for a new infarct w/in 1 wk\n3) AST
(increases over 1st 2 days; slowly declines): NOT SPECIFIC, like
myoglobin\n4) LDH (elevated 2day post MI; remains elevated for 7 days)
What is contraction alkalosis?

Loss of volume without losing

bicarbonate\n E.G. Excessive diuresis with a loop diuretic; CF patient


outside too long on a hot day
What is the mechanism of action of Losartan?

Inhibits angiotension II

receptor: angiotension II is formed from angiotension I by the ACEnzyme in


the lung capillaries
During which interval does isovolumetric relaxation occur in the heart?
During early ventricular diastole: ventricles relax with aortic &
matral valves closed
What side effect is shared by Demeclocycline & lithium?

Nephrogenic

diabetes insipidus: these drugs inhibit the kidneys' response to ADH. \n


DDx: Exogenous ADH administration or a hold on fluids will not improve
symptoms.
Which form of diabetes insipidus will be corrected by exogenous ADH
administration?

Central DI

How is prolactin secretion regulated?

Prolactin increases dopamine

synthesis & secretion from the hypothalamus\n Dopamine itself inhibits


prolacin secretion from the anterior pituitary
In what environment is one more likely to isolate deoxyhemoglobin? What
about oxygemoglobin

1) Deoxyhemoglobin (T form): lowO2 affinity

state = peripheral capillaries\n2) Oxyhemoglobin (R form): high O2 affinity


state = pulmonary capillary bed
What does the P wave represent?

Depolarization of the atria (NOT

contraction)
What does the PR segment represent? isoelectric (flat) portion of AV
node conduction

What does the PR interval represent? Time of initial depolarization of the


atria to the initial depolarization of the ventricles (a pathological delay can
occur here)
What does the QRS complex represent?
What does the T wave represent?

Vetricular depolarization

ventricular repolarization

What does the ST segment represent? Legnth from the end of QRS to
initial deflection of T wave\n That is, the end of ventricular depolarization
to the beginning of ventricular repolarization
What does the QT interval represent? Time between the start of
ventricular depolarization and the end of ventricular repolarization\n from
the onset of the QRS complex to the end of the T wave
What is the change in menapause that brings on the usual lab results of
increased LH and FSH?

Follicular depletion with a resultant decrease in

estrogen secretion; this leads to no negative feedback on:\n


gonadotropinreleasing hormone\n folliclestimulating hormone (FSH)\n
luteinizing hormone (LH)
What is the action of infliximab? It is a chimeric monoclonal antibody. It
bings to soluble TNFalpha (secreted by macrophages & found in high [] in
Crohn's patients)\n Rx: Crohn's; a single infusion = clinical response in
65% of Px\n Side Effects: Headache, GI distress, increased upper
respiratory infections
What drugs, in combo with St. John's wort cause serotonin syndrome?
SSRs (liek sertraline)\n\nSeratonin Syndrome: mental status
changes, autonomic changes (fever, diaphoresis, tachycardia, tremor,
rigidity)
What CSF findings indicate bacterial meningitis?

Increased protein\n

Decreased glucose
What is the most common cause of meningitis in infants (03 months old)?
Lysteria monocytogenes (gram positive rod)\n E. coli (gram rod)
\n Group B strep (gram + cocci)

What is the anticancer action of etoposide? G2 phase action: inhibits


topoisomerase II (results in irreversibel ds breaks in DNA) & prevents
synthesis of components needed for mitosis
Which cells are "output" cells of the cerebellum? What neurotransmitter
do they employ?

1) Purkinje cells\n2) GABA: all these outputs are

inhibitory
What artery supplies both the head of the pancreas & the duodenum?
gastroduodenal artery (branch of the celiac trunk) & superior
mesenteric
What are the 3 ps of Multiple Endocrine neoplasia type 1? 3 tumors:
\nPancrease\nPituitary\nParathyroid
What physical diagnosis finding is characteristic of parathyroidhormone
secreting adenoma?

History of renal stones & neck nodule

What is the genetics of myotonic dystrophy? Autosomal dominate with a


CTG trinucleotide repeat with anticipation\n\nE.G. a mom with a 50100
repeat can have a symptomatic child with a 10001500 repeat
Where is the narrowing in a Px with infantile coarctation of the aorta?
Preductal: between subclavian artery & ductus arteriosus
What is the MOPP regiment & what is it used to treat?

1)

Mechlorethamine, vincristine (Oncovin), Procarbazine, & Prednisone\n2)


lymphoma, Wilm's tumor, Choriocarcinoma
What is the action of Vincristine?

Vinca alkaloid: binds to tubulin &

prevents polymerization into microtubules. T/F there is no mitotic spindle


formation & no mitosis.\n\nRx: testicular carcinoma & Hodgkin's & non
Hodgkin's lymphomas
What is the action of methotrexate?

It is a DNA and protein synthesis

inhibitor\n Folic acid antagonist: inhibits dihydrofolate reducatase during


the S phase
What is a good drug to compliment methotrexate therapy?

Leucovorin: it is folinic acid & helps to replinish the body's supply of folic
acid. \n T/F it inhibits the myelosuppressive effects of methotrexate

What is the equation for PPV?

True Positives divided by those who

tested positive\n Higher prevalence = higher PPV (increased numerator &


denominator) & lower NPV (increased denominator only)\n\n Lower
prevalence = lower PPV (increase in denominator only) & greater NPV
(increase in numerator & denominator)\n\nPPV = TP/(TP+FP)\nNPV =
TN(TN+FN)
What is a complication of neonatal meningitis?

1) Menigeal scarring:

can lead to a decrease in CSF absorption by arachnoid villi (=


nonobstructive/communicating hydrocephalus)\n2) Tx: shunt
What is a side effect of nonselective Bblockers?

Vasoconstriction of

peripheral vessels by blocking B2mediated vasodilation


How is T. gondii transmitted?

Cysts in raw meat\nCat feces

What is the treatment for toxoplasmosis?

Sulfadiazine &

pyrmethamine
What class of antiarrhythmics is Lidocaine?

Class Ib:\n it blocks sodium

channels & decrease action potential duration \n selective for ischemic or


depolarized purkinje & ventricular tissue\n\nOther Ib: mexiletine &
tocainide
Which part of the myocardium is the first to undergo necrosis due to
prolonged ischemia?

ventricular wall (last to be perfused)

Where is a Pancoast tumor located?

Superior pulmonary sulcus; grows

towards thoracic inlet & can damage structures that exit that region:\n
bracial plexus, stellate ganglion, & sympathetic chain
What are the 3 major mechanisms that limit the extent of coagulation? 1)
activity of tissue factor pathway inhibitor\n2) activity of antithrombin
III\n3) activity of activated proten C with cofactor protein S
When a Px is vitamin K deficient, why do they exhibit a coagulation
overactivation?

Protein C is not gammacarboxylated & thus does not

become active to cleave factors Va & VIIIa


Where are bile acids recirculated to the liver? Terminal ilium via a Nabile
cotransporter

Where is iron absorbed? Duodenum


What drugs cause a lupuslike syndrome?

1) quinidine\n2)

minocycline\n3) isoniazid\n4) valproate\n5) hydralazine\n6)


penicillamide\n7) sulfasalazine\n8) procainamide
What are the % risks for needlestick contraction of Hep B, C, & HIV
Hep B: 30%\nHep C: 10%\nHIV: 0.3 0.45%
What is Waldenstrom's macroglobinemia?

Bcell neoplasm secreting lg.

amounts of IgM:\n headaches & vision problems (impaired crania blood


flow)\n weakness\n weight loss\n increased bleeding (due to inhibited
clotting factors)\n monoclonal M spike on serum protein electrophoresis
(lg. spike in the gamma region)
Why are some Bblockers nephrotoxic? B/C they can induse
rhabdomyolysis & T/F excretion of myoglobin by the kidneys.\n Myoglobin
is nephrotoxic = ATN
What is an early sign of salicylate toxicity?

Tinnitus

What part(s) of the body are associated with the 2nd stage of Lyme
disease?

Skin, CNS, heart, & joins

What is damaged when a Px presents with expressive aphasia?

Broca's

area (inferior frontal gyrus)\n\nNB: this is near the primary motor cortex,
so lower arm & face paralysis can also occur.
What is the symptoms of a C. trachomatisis congenital infection? 1) Eye
inflammmation with purulent discharge & eyelid swelling at 514 days
after birth\n2) Can spread to the lungs & cause pneumonia between 411
weeks of life: rapid breathing, cough, & respiratory distress\n3) Histo: BIG
intracytoplasmic inclusions push the nucleus to the cell periphery.
What bacteria presents with abdominal cramps, fever, & bloody diarrhea?
(There are Cshaped organisms on stool analysis)

Campylobacter jejuni

(Rx: erythromycin)\n Untreated: GuillainBarre syndrome acute


symmetric ascending muscle weakness with parethesias that begins in the
lower extremities that results from an inflammation & demyelination of
peripheral nerves

How does the kidney respond to hypovolemic shock?

Increases

glomerular capillary hydrostatic pressure through intravascular fluid


retention & efferent arteriole constriction to retain GFR
What is the clinical presentation of LeschNyhan syndrome?

1)

Hyperuricemia\n2) Excessive production of uric acid = GOUT\n3) Neuro:


selfmutilation choreoathetosis, spasticity, mental retardation, aggressive
behavior
What kind of disorder is LeschNyhan and what is the pathogenesis?

1)

Genetic: Xlinked recessived\n2) Deficiency in the production of


hypoxanthine guanine phosphoribosyltransferase (HGPRTase) >
overproduction of purine & the accumulation of uric acid
Patients with what genetic disorder are at risk for anemia when treated
with isoniazid?

1) G6PD deficiency\n2) The anemia will show

histologically as Heinze bodies (dark inclusions of precipitated globin


chains) & bite cells\n3) anemia can also be produced after exposure to
sulfonamides, antimalarial drugs and oxidative stress.
What intermediate of the TCA cycle inhibits the ratelimiting enzyme of
glycolysis & activates the ratelimiting enzyme of fatty acid synthesis?
Citrate\n inhibits phosphofructokinase\n activates acetyl CoA
arboxylase
What is the mechanism of diptheria toxin?

Inhibits elongation factor

EF2alpha:\n Normally: EF2alpha complexes with guanosine triphosphate


& binds to the ribosome to initiate the translocation step of protein
synthesis.
Which antibiotics inhibit the 30S subunit of prokaryotic ribosomes?
Tetracycline\nAminoglycosides
What is the mechanism of action of the botulinum toxin?

inhibition of

acetylcholine release, which results in flaccid paralysis


What results from damaging the accessory nerve root?

winging of the

scapula (trapezius) and impaired neck rotation (sternocleidomastoid)

Damaging which nerve will result in a monotone voice?

external

laryngeal nerve (terminal branch of the superior laryngeal, a branch of the


vagus)
What does the phrenic nerve supply? diaphragm, mediastinal pleura,
pericardium\n it arises from C3C5
What nerve damage results in "waiter's tip" palsy? suprascapular nerve:
loss of lateral roation of the humerus at the shoulder (supraspinatus &
infraspinatus)
What are the symptoms of 17alphahydroxylase deficiency?

primary

amenorrhea\n lack of 2ndary sexual characteristics\n elevated


aldosterone = hypokalemia (low potassium) & hypertension
What is the inheritance pattern for hereditary spherocytosis?
Autosomal dominant defect in spectrin protein of erythrocyte
membranes\n increased mean corpuscular hemoglobin concentration\n
large RBCs w/o a central pallor\n Dx: osmotic fragility test\n late
complications: bilirubin gallstones & cholecystitis
How is autoimmune hemolytic anemia differentiated from hereditary
spherocytosis?

Autoimmune hemolytic anemia will have a positive

direct Coomb's test


What is the mechanism of HIV encephalitis? Infection of brain
macrophages & microglia:\n Histo: microglial nodule = multinucleated
giant cells (fusion of HIVinfected microglial cells
What cells are infected by the JC virus?oligodendrocytes: results in
progressive multifocal leukoencephalopathy
What is the immunofluorescent picture in Berger's disease?

AKA: IgA

nephropathy (postinfectious nephritic)\n Nonlinear IgA deposits in the


mesangium\n Rx: ACE inhibitors & corticosteroids
How does Horner's syndrome affect pupil constriction?

The pupil

cannot dilate b/c there is an absence of norepinephrine (there is a lesion in


the postganglionic neuron, which prevents neuroepinephrine from being
released) & t/f no stimulation of the ciliary muscle

How does cocaine dilate pupils? It inhibits norepinephrine reuptake and


thus keeps the ciliary muscle stimulated
What is the action of hydroxyamphetamine? It causes the release of
norepinephrine
If hydroxyamphetamine is given as an eye drop to diagnose a nerve
lesion, what would a pupilary dilation indicate? What if there is no
response?

Dilation: a problem from the preganglionic neuron:\n since

this drug initiates norepinephrine release in the postganglionic neuron, if


the pupil dilates, then this neuron is not damaged and the problem is
earlier in the pathway. \n\nNo change: the problem is with the
postganglionic neuron as it SHOULD dilate with this drug
What are the main "things" innervated by the 3neuron oculosympathetic
pathway?

Sweat glands of the forehead\nSmooth muscles of the

eyelid\nPupilary ciliary muscle\n\n Interruption of any of the 3 neurons


results in Horner's syndrome
What characterizes the optical pathology of Horner's syndrome? Abscence
of norepinephrine & no stimulation of the ciliary muscle: NO DILATION
What is the usual presentation of preeclampsia?

1) Hypertension\n2)

Proteinuria\n3) Nondependent edema (face & hands)


What are the symptoms of sever preeclampsia?

1) Headache\n2)

Blurry vision\n3) Right upper quadrant abdominal pain\n4)


Hyperreflexia\n\nRx: magnesium sulfate as a seizure prophylaxis
What type of antidepressants cause serotonin syndrome when combined?
SSRIs (e.g. paroxetine, fluoxetine, sertraline, citalopram) + MAO
inhibitors (e.g. Tanylcypromine, phenelzine)
What are the symptoms of serotonin syndrome?

1) Hyperpyrexia\n2)

Muscle rigidity\n3) Cardiovascular collapse\n4) Mental status changes


What is the common presentation of glioblastoma? headache, focal
neurologic findings, nausea, vomiting, seizure

What is the histological difference between an anaplasitc astrocytoma and


a glioblastoma?

Astrocytoma lacks necrosis and vascular or endothelial

cell proliferation
Where in the brain are anaplasic oligodendrogliomas located?

usually:

frontal lobes
Where are cystic astrocytomas found? posterior fossa of children (these
are lowgrade astrocytomas)
What is tumor lysis syndrome? complicationof lymphomas & other bulky
cancers due to cytotoxic therapy & dying tumor cell contents overwhelm
the body:\n Excess potassium = heart arrhythmias & weakness\n Excess
uric acid (DNA breakdown) = gout & renal failure\n\nRx: allopurinol (to
reduce uric acid production) & glucose/insulin to combat hyperkalemia
What are the Ann Arbor staging notation for lymphoma?

I = single lymph

node or single extralymph organ\n\nII = >/=2 sites on the same side of


the diaphragm\n\nIII = >/=2 sites on both sides of the diaphragm\n A
without constitutional symptoms\n B without constitutional
symptoms\n\nIV = disseminated disease (extralymphatic organs)
What is the Rx for chronic hepatitis B? Interferonalfa to block viral RNA &
DNA synthesis
What is Foscarnet used for?

Ganciclovirresistant cytomegalovirus:

binds to pyrophosphate binding site of viral DNA polymerase


What is Ganciclovir used for?

Cytomegalovirus

What is Ivermectin used for?

Onchocerciasis (River Blindness)

What usually causes systemic miliary TB?

Mycobacterium tuberculosis:

hematogenous spread via seeding of the pulmonary venous return to the


heart to affect the liver, bone marrow, spleen, adrenal glands, meninges...
How does cirrhosis show up in a liver biopsy? bridging fibrous septae (that
look like brad scars linking portal tracts)
What processes lead to centrolobular necrosis of the liver? hypoperfusion
+ retrograde congestion = left or rightsided heart failure or shock

What is the liver histological picture of a chronic alcoholic?Clear,


macrovesicular globules of hepatic steatosis\n Grossly: large, yellow, &
greasy\n Reversible
What is the usual triad for adult normal pressure hydrocephaus (NPH)?
"Wet, wobbly, & wacky": excessive cerebrospinal fluid production,
decreased absorption, or both\n1) Incontinence\n2) Gait difficulty
(Magnetic: feet appear to stick to the floor)\n3) Mental decline\n\nRx:
ventriuloperitoneal shunt\nDx: MRI to rule out a mass lesion
What is the most common etiology of acuteonset of pleuritic chest pain in
otherwise young, healthy men? Pneumothorax
What is bronchiectasis?

dilation of the bronchial tree secondary to

mucus plugging (usually seen in Px with a history of repeated lung


infections; e.g. CF Px)
What are the cluster C personality disorders? Anxiour or worried with
abnormal fears about relationships, separation, & control:\n1) Avoidant\n2)
Obsessivecompulsive\n3) Dependent
What are the 2 most common porphyrias?

Acute intermittent

(AIP)\nPorphyria Cutanea tarda (PCT)


What is the mechanism of AIP (Acute intermittent porphyria)?

Defect in

the enzyme PBG (3rd step in heme synthesis): porphobilinogen deaminase


(aka: uroporphyrinogen I synthase)\n Result: aberrant accumulation of
ALA (aminoleuvlinate) & PBG\n Symptoms: neurovisceral: abdominal
pain, muscle weakness, & anxiety/paranoia/depression & high PBG in urine
(severe: port wine urine)
What is the mechanism of Porphyria cutanea tarda (PCT)? Defect in the
enzyme URO decarboxylase (5th step in heme pathway):\n aberrant
accumulation of URO\n deficiency is inherited or acquired (iron, alcohol,
estrogens, Hep C or HIV)\n Symptoms: photosensitivity, chronic, blistering
lesions on sunexposed skin, no neuropsychiatric signs
What is the pathology of Xlinked sideroblastic anemia?
deficiency (the 1st & ratelimiting step in heme synthesis)

ALA synthase

What is erythropoeitic porphyria?

Deficiency of ferrochelatase (no

iron incorporation into protoheme)\n disorder begins in childhood with


marked photosensitivity
What catalyzes the oxidation of heme to biliverdin? Heme oxygenase
What genetic defect results in Marfan's?

Mutation of the fibrillin1

gene on chromosome 15 = defective microfibrils\n elastin in aorta\n


suspensory ligaments of the lens of the eye
What precipitates DeQuervain's throiditis?

Viral illness causes a

transient hypothyroidism:\n tender thyroid gland\n elevated ESR\n


fatigue, cold intolerance, cool skin, decreased delirium tremens\n\nRx:
nothing
What are the key dermatome locations for the following disc
locations:\nC2\nC3\nC4\nT4\nT7\nT10\nL1

C2 = head (covers the

poserior 1/2 of a skull cap)\nC3 = upper neck\nC4 = between neck &


shoulders\nT4 = nipples "T4 at the teat pore"\nT7 = xiphoid process\nT10
= umbilicus \nL1 = inguinal ligament (Oblique in shape)
What is the action of metformin?

Stimulates glycolysis in peripheral

tissues, decreases intestinal glucose absorption, & decreases hepatic


gluconeogensis\n Side effects (of all biguanides): lactic acidosis; t/f
always give hospitalized patients (who have to stop their metformin) IV
insulin
What is the mechanism of insul & insulin analogs?

Bind to insulin

receptor on cell membranes that activate tyrosine kinase that leads to


glucose absorption.\n Result: increased glycogen storage in the liver &
glycogen & protein synthesis in muscle
What is the mechanism of sulfonylureas (glyburide & glipizide)? Stimulate
the release of endogenous insulin stores: close potassium channels in B
cells that leads to an influx of Ca, cell, depolarization, & insulin release
What is the mechanism of glitaxones (rosiglitazone, pioglitazone)?
Thiazolidinediones: increase sensitivity to insulin\n Stimulate
peroxisome proliferatoractivated receptorgamma; this leads to increased
glucose uptake in muscle & adipose

What is undulant fever?

Infection by Brucella:\n E.G.: B. melitensis

(gram , coccobacilli, intracellular bac that stains with a modified acid


fast) \n From contaminated milk products or contact with sick livestock\n
Symptoms: undulating fever, weakness, loss of appetite
What bac is the cause of cat scratch? Bartonella henselae
How does Pasteurella infection present?

Cellulitis, high fever,

hepatosplenomegaly (septic arthritis in PEDs)\n Route of infection: Dog or


cat bite
What causes a rapidonset food poisoning with nonbloody diarrhea after
eating reheated rice?

Bacillus cereus exotoxin\n S/S: nausea,

vomiting, diarrhea\n "food poisioning from Reheaded Rice (diarrhea)? Be


Cerious!"
What causes a lateonset food poisoning with bloody diarrhea after eating
reheated rice?

Campylobacter jejuni: enterocolitis\n\nRx:

erythromycin or ciprofloxacin
What is the treatment for Cholerae?

Prompt H2O & electrolytes

replacement\n Tetracyclines to reduce the disease course


What is used to treate a severe Shigella infection (bloody diarrhrea)?
Fluroquinolones
What are the 4 major causes of food poisioning, nonbloody diarrhea?

1)

Staphylococcus aureus\n2) Clostridium perfringens\n3) Enterotoxigenic E.


coli (traveler's diarrhea)\n4) Bacillus cereus
What is the mechanism of Gemfibrozil?

1) increases the activity of

lipoprotein lipase & enhances clearance of triglycerides\n2) Fibric acid


derivative that acts on peroxisome proliferatoractivated receptoralpha
protein
What is the action of statins?

Reduce cholesterol biosynthesis in the

liver by preventing the formation of the precursor mevalonate


What is the mechanism of action of ezetimibe?

Blocks cholesterol

absorption in the small intestine brush boarder\n Usually dosed along


with statins

What is the action of resins (cholestyramine & colestipol)? Bind bile acids
and steroids in the small intestine & prevent their absorption
What is the most common cause of pneumonia in CF patients?
Pseudomonas\n\nRx: Gentamicin
What is the mechanism of action of aminoglycosides like gentamicin?
Bactericidal: inhibit formation of the initiation complex & cause
misreading of mRNA\n require O2 for uptake (ineffective against
anaerobes)\n Use: severe gram rod infections \n Tox: nephrotox, Oto,
teratogenic
What type of drug is Ampicillin & what is it used for?

Aminopenicillin\n penicillnasesensitive\n combined with clavulanic acid


to enhance spectrum\n Rx: Gram + & Gram : H. influenzae, E. coli,
Listeria, Proteus, Salmonella, enterococci, Shigella\n Tox: hypersensitivity
& pseudomenbraneous colitis
What can cause SIADH?

Cancer: lymphoma, sarcoma, carcinoma of

duodenum or pancreas\n CNS: meningitis, brain abscess, trauma\n


Pulmonary: oat cell carcinoma, TB, pneumonia, abscess
What is the key call mediator in the pathogenesis of atherosclerotic
plaque?

PlatletDerived Growth Factor: recruits platlets and smooth

muscle to intimal lining to produce an extracellular matrix that becomes a


fibrous plaque
What is the pathogenesis of a vascular fatty streak? 1) Endothelial
Injury\n2) Subendothelial space accumulates lipoproteins \n3) lipoproteins
are glycoslyated or oxidized and this recruits monocytes to the vessel
walls\n4) Monocytes enter the subendothelial space and convert to
macrophages\n5) Unregulated macrocytosis of the LDL cholestrol occurs
that yields foam cells
What does a hydrops fetalis newborn look like?

Gross

hepatosplenomegaly & Ascities\n Peripheral edema & purpura\n\n


Placenta is pale, thickened, & enlarged
What are the finding in a patien with toxic aspirin ingestion?

Metabolic Acidosis with Respiratory alkalosis or Acidosis with anion gap\n

Symptoms: coma, respiratory depression, cardiovascular collapse


\n\nAsprin Tox: increase sensitivity of respiratory centers in the brain,
uncouple oxidative phosphorylation, increase metabolism, inhibit Krebs
cycle & platelet aggregation
When does Acetaminophen become toxic?

When its metabolism is

saturated & Nacetylpbenzoquinone imine (NAPQI) is produced. This


damages hepatocytes\n NAPQI cannot be detoxified by endogenous
glutathione\n Lab: Huge elevations in aspartate aminotransferase &
alanine aminotransferase (due to centrilobar necrosis)\n\nRx: activated
charcoal within 4 hours & Nacetylcysteine
What confirms a Benzo overdose?

Reversal of symptoms (coma &

respiratory depression: both are rare) by flumazenil


What is the toxicity of Ephedra? CNS & cardiovascular stimulation,
arrhythmia, stroke, seizures
What is the toxicity of drain cleaners? NaOH or KOH = GI erosion &
metabolic alkalosis
What is the equation for clearance? (L/min)

Rate of drug elimination /

plasma drug concentration


How does spironolactone cause gynocomastia, testicular atrophy, and
decreased libido?

It binds to steroid receptors outside the kidney\n Its

therapeutic effect is as an antagonist at the renal aldosterone receptor in


the DC tubule\n It is potassiumsparing
What are the main side effects of Bblockers?
impotence\nbronchoconstriction\nhypoglycemia\ncold extremities
What is the main action (& side effects) of hydrochlorothiazide? Action:
Inhibit luminal Na & Cl entry to the DC tubule cells, Potassiumwasting\n
SE: hypokalemia, give potassium supplements
How does long term steroid use induce diabetes?

Decreased glucose

tolerance & counterregulatory action of the hormone:\n increase glucose


production by the liver by stimulating glconeogenesis & proteolysis in the
skeletal muscle to release glucogenic amino acides into the vasculature

What are teh 2 main causes of Diabetes insipidus? Pituitary


dysfunction\n Failure of kidneys to respond to ADH
Why do Cushing's disease patients develop hyperpigmentation of the skin?
Primary pituitary adenoma secretes ACTH\n ACTH stimulates
melanocytes
Does steroid use result in hyper or hypotension?

Hypertension due to

the mineralocorticoid properties of steroids (increased Na retention)


How does Cushing's syndrome manifest in muscle tissue? Wasting due to
catabolic effects of cortisol
What ADHD medication is not a controlled substance?

Atomoxetine

a selective norepinephrine reuptake inhibitor\n it has no abuse potential


What is the drug treatment for carcinoid syndrome? Octreotide:
somatostain analog that inhibits secretion from neuroendocrine tumors
(also used with VIPomas that secrete vasoactive intestinal polypeptide)
What does aflatoxin cause?

1) Hepatocellular carcinoma\n2) This toxin

is produced by Aspergillus flavus growing on peanuts & grains and


intercaltes into host DNA\n3) Contamination is a problem in Asia & Africa
How does AZT stop reverse transcriptase on a molecular level?

It is

converted in cells into a nucleoside triphosphate & becomes incorporated


into DNA. However, without a hydroxyl at the 3' carbon, it cannot form a
phosphate bond & elongate.
What is the function of zonula occludens?

Tight Junctions:\n Prevent

leakage across the intracellular space \n Found toward luminal side of


epithelial cells
What is the function of zonula adherens?

Components: actin

filaments & Ecadherins\n Connects cytoskeletal elements of neighboring


cells\n Below zonula occludens\n Allows cells to create a functional unit
What is the function of a desmosome? Aka: Spotadhering junction\n
Contents: keratin & desmoplakin (desmoglein), actin, & Ecadherin\n
Function: joins small, discrete sections of neighboring cells

What is the function of Gap junctions? Function: Cell communication\n


Structure: connexins with central channel
What are the causes of eosinophilia?

NAACP: (release Major Basic

Protein)\n1) Neoplasia\n2) Asthma\n3) Allergies\n4) Collagen vascular


diseases\n5) Parasitic infection
What are the main infections that can result in GillainBarre?

1)

Herpesvirus, CMV, EBV, HIV\n2) Campylobacter jejuni gastroenteritis\n\n+


stress
How does E. histolytica infection present?

It is an amoebal dysentery

associated with liver abscess formation


What foods can be contaminated with L. monocytogenes? Delicatessen
meats & soft cheese\n Gram + rod with tumbling motility
What is prophylthiouracil used to treat?

hyperthyroidism: inhibits

the synthesis of T4 & the peripheral conversion of T4 to


triiodothyronine.\n (rare) Tox: agranulocytosis, rash, edema
What is the clinical use of Leuprolide? Infertility, prostate cancer, uterine
fibroids\n MOA: Gonadotropinreleasing hormone analog = dosage
suppresses release of LH & downregulates the hypothalamicpituitary
gonad axis\n SE: gynecomastia, decreased libido, nausea, vomiting
Which anticoagulant does not require any laboratory monitoring?
LMWH = lowmolecularweight heparin\n activates antithrombin
III\n administer subcutaneously
What is the MOA of spironolactone?

PotassiumSparing diuretic:\n

Competitive aldosterone receptor antagonest: Na reabsorption & K


secretion are inhibited @ the cortical collecting duct\n it is a steroid
What is the molecular picture of nephrogenic diabetes insipidus? Loss of
arginine vasopressin receptor 2 function
Which stage of the cardiac cycle is prolonged in a patient with atrial
stenosis?

Isovolumetric contraction: it must overcome the resistance

of the valve & open it\n Sound: ejection click & cresendodecrescendo
murmur @ right sternal boarder that radiated to the neck

What is the pathogenesis of the tetralogy of Fallot? Anterosuperior


displacement of the infundibular septum during development in utero:\n
overriding aorta\n pulmonary stenosis\n right ventricular hypertrophy
What is the classic behavior of a child with tetralogy of Fallot?

They

squat during cyanotic spells to compress the femoral arteries to decrease


their righttoleft shunt
What causes venous abnormalities that can lead to impotence?
Insufficient relaxation of smooth muscle due to excessive adrenergic tone
or damaged parasympathetic innervation\n this can result in a leak into
the dorsal & saphenous veins & results in venous outflow abnormality
What is the normal physiology of an erection?

Accelerated blood flow

increases pressure within the intracavernosal spaces & this blocks penile
venous outflow.\n abnormalities in venous outflow (e.g.e leak from dorsal
vein to saphenous vein) can result in failure to attain &/or maintain an
erection
What is accumulated in LeschNyhan syndrome?

Uric Acid:\n There is a

deficiency of HGPRT in the purine salvage pathway.\n T/F guanine &


hypoxanthine cannot be recycled to IMP & GMP respectively\n Guanine &
hypoxanthine are instead converted to uric acid\n\n ALSO: de novo purine
synthesis is increased & this results in MORE uric acid
What gene fusion is characteristic in Acut promyelocytic leukemia (APL)?
t(15:17) = PML + retinoic acid receptor\n\nRx: retinoic acid to
mature the leukemic cells
What gene fusion is characteristic in Chronic myelogenous leukemia
(CML)?

Philadelphia chromosome\n t(9:22): BCRABL fusion

protein
What genetic translocation results in Burkitt's lymphoma? t(8:14): Cmyc
(Ch. 8) & immunoglobulin heavy chain (Ch. 14) locus
What is Ras? a plasma membrane protein that has GTP binding and
GTPase abilities.\n its activity is increased in some cancers BUT, it is not
associated with translocations

Which antiinflammatory drug is best to avoid gastric side effects?


Celecoxib: selective COX2 inhibitor\n decreases conversion of
arachiodonic acid to prostaglandin G2\n Prescribe with caution due to
heart side effects
How does COX1 inhibition lead to ulcers?

Removes the prostaglandin

syntheses that is protective to gastric mucosa


What is the mechanism of primary hemochromatosis?

AR inheritance:

mutation in HFE gene\n body over absorbs iron in the GI


What are some causes of sideroblastic anemia?

incomplete synthesis

of heme:\n Drug interactions (ethanol, isoniazid, chloramphenicol)\n


Lead poisoning\n Nutritional deficiencies of pyridoxine/copper\n genetic
disorders
What is the toxicity of tricyclic antidepressants?

MOA: inhibit neuronal

reuptake of norepinephrine, seratonin, & other amines\n Mental status


changes\n Hypotension\n Anticholinergic tox: flushing, hyperthermia,
dilated pupils, intestinal ileus, urinary retention\n Ventricular arrhythmias
What day post MI is a patient at risk for ventricular wall rupture? 47:
severe hypotension & results in cardiogenic shock
What is the preferred antibiotic for communityacquired lobal pneumonia
(high fever, productive cough, acute onset)? ceftriaxone: broad spectrum,
3rdgeneration cephalosporin against:\n S. aureus\n H. influenzae\n S.
pneumoniae
What is clindamycin used for?

Anaerobics: (often combined with an

aminoglycoside)\n penetrating wound infections of abdomen\n UTIs in


women
What is doxycycline particularly useful for?

Px with renal failure needing

an antibiotic:\n it binds to the 30S subunit & prevents attachment of


aminoacylttRNA\n Rx: atypical pneumonia (Chlamydia & Mycoplasma)\n
SE: GI distress, photosensitivity, rash

What is the 1st line therapy for P. jiroveci pneumonia?

Trimethoprim

sulfamethoxazole
Which anemia is caused by lead poisoning?

sideroblastic:\n Lead binds

to sulfhydryl groups of hemesynthesis enzymes: alphaaminolevulinic


acid dehydratase & ferroketolase)
What are the 4 toxicities of lead?

1) inhibits heme synthesis (binds to

alphaaminolevulinic acid dehydratase & ferroketolase)\n2) competes with


Ca ions for binding to other molecules\n3) inhibits membrane enzymes
(e.g. 5'nucleotidase & NaK pumps) causing hemolysis, renal cell damage
& hypertension\n4) inhibits active vit. D production
What drugs cause aplastic anemia?
Chloramphenicol\nSulfonamides\nAlkylating agents
What lab test can distinguish between iron deficiency anemia and lead
poisoning anemia? They both present with hypochromatic, microcytic
anemia

Transferrin saturation:\n In irondeficiency, it is increased

(b/c there is no iron to saturate)\n in lead poisoning, it is elevated or


normal (b/c there is no iron loss)
What is elevated in Vit. B12 deficiency?

1) Homocysteine = No B12

coenzyme for its conversion to methionine\n2) Methylmalonic acid = No


B12 for methylmalonylCoA mutase to convert this to succinylCoA &
metabolize oddchain fatty acid
What are the clinical symptoms of B12 deficiency? 1) Fatigue\n2)
Peripheral neuropathy\n3) Macrocytic anemia\n\nLab: elevated
homocysteine & Methylmalonic Acid
What is the most common serum antibody seen in Multiple Myeloma?

IgG

(55%)\nIga (25%)
In what cancer will a pathologist be able to see Auer rods? Acute
myelogenous leukemia\n These are in myeloblast cytoplasm
What are the characteristics of Waldenstrom's macroglobulinemia?
Monoclonal IgM\n Epistaxis\n Bleeding Gums

What are the common symptoms of Multiple Myeloma?


CRAB:\nhyperCalcemia\nRenal
insufficiency/failure\nAnemia\nBone/Back pain (lytic lesions)
What are the common physical findings in Rickets? Areas of bone
growth: \n wrists (widened)\n ankles (widened) \n costrochondral
junctions (rachitic rosary)\n\nThese are all unmineralized, soft oseoid
What are the common lab findings in Rickets?

Decreased: Ca, PO3,

vit D, alkaline phosphatase\n\nIncreased: PTH


What is vitamin D's main function?

Promotes Ca & PO3 absorption from

the GI
What genetic mutation renders a patient immune to infection with certain
strains of HIV?

Mutated CCR5 (homozygous):\n CCR5 receptor is a

chemokine receptor on macrophages that facilitates viral entry into cells.


What is injured in the "straddle injury"?

When the urethra below

the urogenital diaphragam (at the junction of the membranous & penile)
ruptures due to trauma\n Result: urine flows into the scrotum & perineal
region.
What are the 3 parts of the male urethra?

1) prostatic\n2) membranous

through the urogenital diaphragm\n3) penile


What will cause the anterior bladder wall to rupture?

Fractured

Pelvis\n Urine wile flow into the retropubic space


What would cause a penile urethra rupture? Crush Injury\n Urine will
flow into the deep fascia of Buck within the penis
What will cause the superior bladder wall to rupture (dome rupture)?

Forceful compression of a full bladder\n Urine will flow into the peritoneal
cavity
What will cause a urethral rupture above the urogenital diaphragm (@ the
junction of the prostatic & membranous urethra)?

Fractured pelvis or

improper catheter insertion\n Urine will flow into the retropubic space

What is the toxicity of polymyxins?

Nephro & Neuro: dizziness,

drowsiness, confusion, nystagmus, blurred vision


What is the MOA of polymxins? Bind to Gram bacterial cell membrane
phospholipids & act like a detergent
What drugs commonly cause granulocytopenia (neutrophils, eosinophis, &
basophils)? Clozapine\n Ticlopidine\n Sulfasalazine\n Antithyroid
drugs\n Trimethoprim\n Dapsone
What drugs, especially in combination, cause ototoxicity?
Aminoglycosides\nVancomycin
What drugs ill precipitate a severe anemia with jaundice, pallor, dark
urine, & back pain in patients with G6PD?

Primaquine\nSulfonamides

What is the treatment for ulcerative colitis?

Sulfasalazine: compo of

sulfapyridine (antibacerial) & mesalamine (antiinflammatory)\n SE:


malaise, nausea, sulfonamide tox, reversible oligospermia\n\nAlso, 6
mercatopurine & methotrexate can be used
What is Sucralfate used for?

Ulcers & traveler's diarrhea: it binds to an

ulcer base & provides protection


What is Infliximab used for?

Monoclonal antibody to tumor necrosis

factor alpha\n Crohn's & a 2nd line for UC


What is Nizatidine used for?

Peptic ulcer, gastritis, mild GERD\n

Reversible histamine2 blocker


What is Omeprazole used for?

HYPERsecretory states:Peptic ulcer,

gastritis, esophageal reflux, ZE syndrome\n Proton pump (H+/K+ATPase)


of stomach parietal cells inhibitor\n SE: headache, GI distrubances
What is Ondansetron used for? Antiemetic\n 6hydroxytryptamine3
agonist that is centrallyacting
What are the symptoms of atropine poisoning?

Blind as a bat:

mydriasis\n Red as a beet: flushed skin\n Mad as a hatter: delirium &


disorientation\n Hot as a hare: fever from inhibited sweat glands\n Dry

as a bone: lacrimal gland inhibition no tears or saliva\n Plugged Up:


constipation & urinary retension
What is the antidote for atropine poisoning? Physostigmine: reversibly
inhibits acetylcholinesterases to increase the amount of acetylcholine at
the cholinergic receptor\n ONLY use in SEVER overdose\n\nLight
poisoning: supportive care, activated charcoal, benzos
What is Clonidine used for?

Hypertension: it is an alpha2receptor

agonist
What type of drug is Lisinopril? ACE inhibitor used for hypertensive Px.
What type of drug is Metoprolol?Hypertension: a cardioselective Bblocker
What are natural sources of atropine & scopolamine?

Poisonous

plants: deadly nighshade & jimson weed


Which part of the nervous system controls accomodation or cycloplegia?
parasympathtic control of the ciliary muscle via cholinergic
receptors
What action does Acetylcholine have at muscarinic receptors in the eye?
Miosis (pupil constriction)\n Ciliary muscle contraction
(accommodation)\n\nNB: atropine (a muscarinic agonist) has the oppose
effect: mydriasis & cyloplegia. Physostigmine restores the parasympathtic
response in atropine poisoning
What is the MOA of Pilocarpine? It is an acetylcholine mimetic causes
miosis & contraction of the ciliary muscle\n Rx: open (wideangle) &
closed (narrowangle) glaucoma
How can you calculate the PPV if given the sensitivity, specificity and pre
test probability?

PPV = \n1) Convert the pretest probability to an odds

ratio\n2) Calculate the the test's positive likelihood ratio (LR+) =


sensitivity / (1specificity)\n3) Multiply #1 x #2
How does etoposide induct apoptosis? It stabilizes topoisomerase IIDNA
complex during the G2 phase & prevents G2 completion & DNA strands
break. This triggers apoptosis.

What is the MOA of paclitaxel?

Hyperstabilizes microtubules and

directly prevent anaphase (M1 arrest)\n Rx: ovarian & adjuvant to breast
cancer
What is the MOA of Beomycin? Causes the formation of free radicals after
binding DNA, which causes single & doublestrand breaks within the DNA
Which TB drug induces p450?

Rifampin

What are the main drug types that induce the cytochrome p450 system?
Rifampin\nFluconazole\nProtease
inhibitors\nDigoxin\nWarfarin\nOral Controceptives
What is the MOA of ethanbutol? It inhibits arabinsyl transferase enzymes
needed for mycobacterial cell wall formation\n\nTox: hyperuricemia, GI
distress, optic neuritis, visual loss
What is the toxicity of isoniazid? Hepatitis\nhemolysis in G6PD Px\nlupus
like syndrome\nNeurotoxicity
What is the MOA of streptomycin?

Aminoglycoside that binds the 30S

subunit & inhibits protein synthesis.\n\nTox: neuromuscular blockade &


ototoxicity
Which cancer causes parathyroid hormonerelated peptide problems?
Squamous Cell\n Results in hypercalcemia & suppressed levels of PTH
What causes central diabetes insipidus?

decreased ADH

production\n hypernatremia & increased plasma osmolaity


What newborn umbilical cord abnormality can indicated another
congenital abnormality (usually cardiovascular)?

A single umbilical

artery (there should be 2: to take deoxygenated blood from the fetus to


the placenta)
What is the MOA of Methotrexate?

It is an analog of folic acid &

inhibits dihydrofolate reducatase & thus prevents tetrahydrofolate


regeneration (metabolically active folic acid\n Therefore DNA, RNA, &
protein synthesis is decreased

What is pyridoxine used for?

Pyridoxine = vit. B6\n cofactor in amino

acid catabolism, heme synthesis, & glycogen breakdown\n\nDeficiency:


angular stomatitis, glossitis, dermatitis, & seizures
What does cobalamin deficiency result in?

Vit B12 deficiency: decreased

methionine synthesis from homocysteine\n megaloblastic anemia (no


folate derivatives for DNA synthesis are recycled b/c
methyltetrahydrofolate is not converted to tetrahydrofolate)\n
paresthesias & ataxia due to subacute combined degeneration (of the
dorsal & lateral spinal columns)\n Reduced epinephrine, creatine, &
melatonin b/c methionine is a precursor
What is the treatment for a benzodiazepam overdose?

Flumazenil: a

competitive antagonist of the GABAergic receptor


What is the treatment for opiate overdose?

Naloxone

What can be given to counter a Bblocker overdose?

Glucagon

What happens after vasopressin binds to the V2 receptor? Activation of a


Gs to activate adenylyl cyclase\n Increased AMP then leads to the
insertion of aquaporin proteins into the apical membrane of the collecting
duct epithelium\n mutations in the V2 receptor can cause diabetes
insipidus
How do fibrates increase HDL?

Induce the synthesis of the major HDL

cholesterol apolipoproteins (apoA1 & apoA2)


How do fibrates reduce lipids?

Increase lipoprotein lipasemediated

lipolysis & this removes triglycerides from the peripheral circulation\n


E.G. Gemfibrozil & fenofribrate
What is the mechanism of statins?

Inhibit HMGCoA reductase (3

Hydroxyl3methylglutarylCoA) the ratelimiting step of cholesterol


biosynthesis
What is the enzyme that converts arachidonic acid to leukotrienes?

lipoxygenase
What is the 1st committed step of fatty acid synthesis?
malyonylCoa via AcetylCoA carboxylase

The synthesis of

What is the MOA of thrombolytics?

Indirect or direct assistance of the

conversion of plasminogen to plasmin \n plasmin cleaves thrombin &


fibrin clots
What extra problem is associated with ovarian teratomas? Can contain
thyroid tissue (struma ovarii) & cause a hyperthyroid state
What lab value indicates that a fetus will have Down's?

AFP reduction:

"AFP goes down in Down's Syndrome"


What lab value indicates a neural tube defect in a fetus?

Dramatic

increase in AFP (reduce risk with prenatal folate)


What is the MOA of abciximab

Monoclonal antibody against the

glycoprotein IIb/IIIa receptor on the surface of platelets\n It impairs


platelet aggregation\n\nRx: acute coronary syndrome
What is the MOA of heparin?

Complexes with Antithrombin II and

increases its activity by 2000fold


What is the treatment for Enterobacter species?

Carbapenems (i.e.

imipenem & meropenem) when resistant to penicillin/penicillinase via B


lactamase
What is the mechanism of C. difficile infection?

Pseudomembranous

colitis: Exotoxin that results in enterocyte death & watery diarrhea\n


Often follows ampicillin or clindamycin treatment
What is the toxicity of C. perfringens? Lecithinase exotoxin that results in
myonecrosis, gas gangrene, & hemolysis
What are the signs of nephrotic syndrome?

1) Azotemia\n2)

Hematuria\n3) Oliguria\n4) Hypertension


What is the initial treatment for esophageal dysmotility?
Metoclopramide: promotility agent
What is Ondansetron used for? 5HT3 receptor antagonist\n Used to
treat refractory or severe nausea & vomiting

What are the two types of diabetes insipidus (DI)

Central (reduced

ability to produce ADH)\n Nephrogenic (lack renal response to ADH) & can
be a rare toxicity of Lithium
What is the toxicity of valproic acid

Liver & pancreatic toxicity along

with neural tube defects if the patient is pregnant.\n This drug is an


antiepileptic
What is the risk factor for developing Ebstein's anomaly? Lithium
exposure during gestation:\n apical displacement of the septal &
posterior tricuspid valve leaflets = atrialization of the right ventricle &
displacement of the anterior leaflet
What is fetal hydantoin syndrome?

Phenytoinassociated fetal

defects:\n growth deficiencies\n skull & facial feature abnormalities\n


mental retardation\n underdevelopment of nails in the fingers & toes
What are the risk factors for the Tetralogy of Fallot? Phenytoin
\nAlcohol\n\nAlso: 22q11 syndrome association
What physical symptoms characterize ALS?

Upper & lower motor

neuron signs: fasciculations, hyperreflexia in lower extremities and


areflexia in the upper limbs\n Biopsy of muscle: muscular atrophy due to
lack of neuronal input
What type of viruses are capable of genetic/antigenic shift?

Those

with segmented genomes that can exchange segments (e.g. influenza:


negativestranded RNA virus) & can result in periodic changes int he
surface antigen of the virus:\n H antigens: hemaglutinin\n N antigens:
neuraminidase
What family of viruses is Influenza a member?

Orthomyxovirus

What viruses are a member of the paramyxovirus family?


Parainfluenza\nRSV\nMumps\nMeasles
What is the MOA of amphetamines for ADD treatment?

Promote the

release of catecholamines (dopamine & norepinephrine):\n increases the


concentration in the synaptic cleft \n Dopaminergic activity in the CNS
stimulates & leads to increased attention

When is atropine indicated for heart therapy?Advanced cardiac life


support to block parasympathetic input to the heart:\n Increases heart
rate
What is the common MOA of cocains and tricyclic antidepressants?
Block reuptake of catecholamines into the presynaptic terminals
What is the MOA of Reserpine? Blocks catecholamine storage within the
presyaptic neuron by preventing storage withing the presynaptic vesicles
What tract is responsible for the dopaminergic control of prolactin?
tuberoinfundibular: antagonists to D2receptors in this region will
cause hyperprolactinemia.\n\nE.G. Chlorpromazine (antipsychotic) is such
a drug that antagonizes this receptor.
What tract is antagonized by typical antipsychotics and thought to
exacerbate negative symptoms of schizophrenia?

Mesocortical

(cognition pathway) D2receptors


What results in D2receptor antagonism by chlorpromazine in the
mesolimbic tract? Decrease in positive symptoms of schizophrenia
(hallucination, delusions, & frank psychosis)
What is the function of the nigrostriatal tract?

Basal ganglia involved

in movement production:\n antagonism of D2Receptors results in


Parkinsonlike movement & tardive dyskinesia (typical antipsychotics act
here)
What is the DDx for organisms causing long branching filaments?
Fungi\nNocardia asteroides\nActinomyces israelii
What is the treatment for a Nocardia pneumonia?

Sulfabased antibiotic,

IV, high dose: sulfamethoxazole


What are the catalase, hemolytic, and coagulase properties of S. aureus?
Catalasepositive\nCoagulasepositive\nBhemolytic
What are the catalase and coagulase properties of S. epidermis? Catalase
positive\nCoagulasenegative

What bacterial cause of pneumonia is a grampositive cocci that grows in


chains and possesses a glycoprotein capsule?

S. pneumoniae; it is

also bile soluble


Which type of ulcer results in weight loss & pain with food?
gastric\n\nRx: 14day triple therapy\n Bismuth\n Omeprazole\n
Amoxicillin or tetracycline
What are the symptoms of acut gastritis?
Nausea\nVomiting\nHematemesis\n\nHisto: diffuse erythematous
patches on the lumen of the stomach
What is the MOA of Sildenafil?

Inhibits cGMP phosphdiesterase (PDE5)

which increases cGMP leading to smooth muscle relaxation in the corpus


cavernosum to increase blood flow to the corpus vascularity
What is Clomiphene used for?

Female infertility:\n it is a selective

estrogen receptor modulator and partial agonist of the estrogen binding


site on the hypothalamus & pituitary\n it increases secretion of
gonadotropinreleasing hormone
What is the MOA of Alprostadil? Intraurethral prostaglandin pellet that
increases arterial inflow & decreases venous outflow via binding G proteins
& stimulating adenylyl cylase\n\nTox: priapism if given as an
intracavernosal injection
What structures run through the tarsal tunnel (posterior to the medial
malleolus)? Superior to Inferior: "Tom, Dick, And Very Nervous Harry":\n
Tibialis posterior tendon: plantar flexion & inversion\n flexor Digitorum
longus tendon: plantar flexion of foot & flexion of the 2nd5th toes\n
posterior tibial Artery \n posterior tibial Vein\n tibial Nerve\n flexor
Hallucis longus tendon
What are fibromuscular dysplasias of the kidney?

Inherent structural

defects of the renal artery wall that impair perfusion:\n leads to


stimulation of the reninangiotensionaldosterone axis that leads to
vasoconstriction & volume expansion\n Rx: surgery
What is a catecholaminescreting tumor of the adrenal medulla?
Pheochromocytoma:\n increased plasma epinephrine &

norepinephrine\n Symptoms: hypertension, diaphoresis, headaches, heart


palpitations
What are the symptoms of Conn's syndrome (primary
hyperaldosteronism)? What is the treatment? Newonset hypertension\n
Alkalosis & hypokalemia (can manifest as muscle weakness or
paralysis)\n Low plasma renin\n\nRx: Aldosterone antagonist (e.g.
spironolactone)
What antidepressants have the best cardiac profile?1st Choice:
SSRIs\n\n2nd Choice: Monoamine oxidase inhibitors (MAOs)\n also good
for BPH patients b/c there are no anticholinergic activity
What causes "red man" syndrome (RMS)?

Too rapid infusion of

vancomycin\n\nRx: slow or pretreat with antihistamine


What is a common sequela of Rotavirus infection?

Transient lactose

intolerance due to brush border of the small intestine being shed.


Where can patients contract Chagas disease?

Central & South

America via the reduvidd bug\n chagomas (dermal granuloma)\n


myocarditis\n CHF if not treated\n\nChronic: arrhythmias, dilated
cardiomyopathy, megacolon, megaesophagus
Where do G. lamblia trophozoites colonize?

Duodenum & jejunum: cause

malabsorption & diarrhea\n\nRx: Metronidazole


What is Oral Rehydration Therapy? as used for Cholera?

Glucose

containing sodium solution in a ration not exceeding 2 glucose molecules


per 1 sodoum molecules\n prevents dehydration by shifting fluid from the
intestinal lumen into the circulation as a 2ndary effect of glucosecoupled
Na transport into the mucosal cells
What is Gerstmann's syndrome Damage to the visual association cortex
(angular gyrus):\n1) Inability to distinguish right from left\n2) Inability to
identify fingers\n3) Agraphia\n4) Acalculia or dyscalcuia
What results from injury to the left posterior inferior frontal lobe? Broca's
(expressive aphasia) & (sometimes) brachiofacial weakness

What results from injury to the left posterior superior temporal gyrus?
Wernicke's (fluent/receptive) aphasia & may involve visual dificits
What results from injury to the left sylvian region?

global aphasia,

hemianopia, hemiplegia
What results from injury to the left temporoparetooccipital junction?
Transcoritical sensory aphasia: \n1) poor comprehension\n2) good
repetition\n3) nonfluent speech
What cause of sudden death does malignant hypertension predispose to?
Thoracic aortic dissection: pain radiates to the back & ECG will
appear normal!
What does the Achilles tendon reflex test?

S12 nerve roots\n a

diminished deep tendon reflex = LMN injury\n\nNB: "Count up"\nAchilles


reflex (S1, S2)\nKnee jerk (L3, L4) \nBiceps (C5, C6) \nTriceps (C7, C8)
What infections does the complement pathway clear?

gram negative

What are the most common causes of otitis & sinusitis?

H. influenzae

(gram )\nMoraxella catarrhalis (gram )


How do IgG & IgM bind to gram back in the classical complement
pathway?

with the Fc region

What is C7 radiculopathies?

Damage (usually via herniation) to the C7

nerve root:\n Tricep weakness: elbow extension limited\n Weakness in


extension of wrist & fingers\n Sensory: pain & paresthesia of the middle
finger
Do cervical roots emerge on top or below corresponding vertebrae?
ABOVE
What would a C4C5 disc herniation result in?

Impingement upon the

C5 root: limited shoulder abduction\n Weak deltoid\n Weak


infraspinatus\n Sensory: shoulder & upper, lateral arm
What would a C5C6 herniation result in?

C6 nerve root problems:\n

weak wrist extensors & biceps\n Absence of right bicep & brachioradialis
relex\n Sensory: 1st & 2nd fingers

Why is alkalosis coupled with potassium wasting?

Kidneys avidly take up

H+ to restore the pH and these are exchanged with potassium


What is the DDx for target cells on a peripheral blood spear?
HALT:\nHemoglobin C disease\nAsplenia\nLiver
disease\nThalassemia
What is within the omental foramen?

hepatoduodenal ligament that

contains the common bile duct, hepatic artery proper & hepatic portal vein
What is the MOA of isoniazid?

Inhibits mycobacterial cell wall formation

by inhibiting enzymes required for mycolic acid synthesis.


What is the MOA of Ethambutol? It inhibits arabinosyl transferase to
decrease arabinogalactan (a component of mycobacterial cell walls)
What is the MOA of pyrazinamide?

Unknown; however, its activity

requires conversion by pyrazinamidase


What is the presentation of a patient with pheochromocytoma?

Neoplasm

of the chromaffin cells (neural crest derivatives that synthesize & release
catecholamines):\n Hypertension (bursts or chronic)\n Palpitations\n
Headache\n Tremor\n Sweating\n Sense of apprehension
What syndrome develops rapidly (days to weeks) with adrenocortical
insufficiency and an overwhelming bacterial infection (N. meningitidis),
rapidly progressive hypotension, shock, DIC, & widespread purpura?
WaterhouseFriderichsen syndroma
What is bronchiectasis?

abnormal dilation of the proximal & medium

sized bronchi (>2mm in diameter) caused by destruction of the muscular


and elastic components of the bonchial walls
What Reid index value indicates chronic bronchitis and hyperplasia of
submucosal glands?

>0.4

How can you definitively diagnose a B12 deficiency?

Check

methylmalonic acid levels:\n B12 is a cofactor for the conversion of of


Methylmalonyl CoA to succinyl CoA\n Without B13, Methylmalonyl CoA is
converted to methylmalonic acid

What causes the "penisat12" phenomenon?

5alphareducatase

deficiency:\n genetic males with testes & ambiguous or female external


genitalia\n @ puberty, increased testosterone causes masculinization of
the external genitalia\n5alpha reductase converts testosterone to
dihydrotestosterone that is required in utero to develop external genitalia
What does 21hydroxylase do? It converts progesterone to
deoxycorticosterone on the way to aldosterone\n without this, there is a
virilization of the female genitalia at birth\n without this, there is no
feedback inhibition on renin & Px have elevated Bp
What does 17alphahydroxylase do?

Converts pregnenolone &

progesterone to 17alphahydroxypregnenolne & 17alpha


hydroxyprogesterone respectively\n Deficiency leads to increased sex
hormone levels, decreased corisol levels & increased mineralocorticoid
levels
What deficiency causes chronic granulomatous disease?

NADPH oxidase:

no superoxides from molecular O2


What is pregnenolone?

an intermediate in the conversion of cholesterol

into adrenal steroids


What ovarian tumor presents with symptoms of hyperthyroidism in 515%
of patients? Struma ovarii:\n monodermal teratoma that contains thyroid
tissue\n Increased risk of malignant thyroid carcinoma arising from this
tumor
What can cross the vilous epithelial cell membrane of the small intestine
via simple diffusion?

free fatty acids & monoglycerides: breakdown

product of triglycerides by pancreatic lipase & bile salts


How are carbohydrates absorbed across the apical membranes? SGLT1:
glucose & galactose\nGLUT5: fructose\n\nPortal circulation access is
through GLUT2 cotransporter
What is the histological sign of extramedulllary hematopoiesis?
dacryocytes or "teardrop cells"

What is idopathic myelofibrosis? myeloproliferative syndrome in which


bone marrow is gradually replaced by collagen fibrosis\n Lab: anemia,
mild thromocytosis, mild neutrophilia\n Gradual bone marrow failure
results in extramedullary hematopoiesis in spleen
What is the DDx for target cells 1) asplenia\n2) alpha & beta thalassemia
What is another term for hemoglobin clumping?

Heinz Bodies (e.g.

G6PD deficiency)
What is the histological appearance of intravascular hemolysis? helmet
cells & schistocytes
What is the lab finding in a child with ITP (idiopathic thrombocytopenia
purpura)?

Antiplatelet antibodies\n Thrombocytopenia\n Blood

smear: large platelets due to increased production


What is the genetics of Hunters & Hurlers?

Xlinked recessive

What is the pathology of Hunter's & its clinical presentation?

1)

Pathology: deficiency in lysosomal iduronate sulfatase; accumulation of


dermatan & heparan sulfate\n2) Clinical: Mild mental retardation, hearing
loss, coarse facies, hepatosplenomegaly, agressive behavior, airway
obstuction, but NO corneal clouding
What is the clinical presentation of Job's syndrome? 1) Recurrent staph
abscesses\n2) Pruritic eczema (papulovesicular dermatitis)\n3) Multiple
"cold" (i.e. noninflamed) skin lesions\n\nLAB: High levels of IgE: \n T
helper cells cannot produce IFNgamma & thus do not activate phagocytic
cells\n T/F > a failure of neutrophil chemotatic response
What are the lab values & symptoms of a patient with WiskottAldrich
MALE (xlinked)\n\nSymptoms: (TRIAD)\n Recurrent pyrogenic
infections\n Eczema (itchy erythematous rash)\n
Thrombocytopenia\n\n\nLAB:\n Elevated IgA, IgE\n Normal IgG\n Low
IgM = pathology b/c there is no IgM response
When is Amiodarone indicated & what are its toxicities?

Indication:

Vfib\n\nTOX:\nhypotension, PULMONARY FIbrosis, thyroid dysfunction,


hepatotox, ocular changes, bradyarrhythmia & torsades

When are Class IC agents (flecainide & encainide) indicated?

Vtach or

intractable supraventricular tachycardia\n\nMOA: phase 3 of the action


potential\nTox: increased postMI mortality
When is Procainamide indicated?

1) Reentrant & ectopic

suptraventricular & ventricular tachycardia\n2) MOA: (Class 1a Nachannel


blocker) increase action potential duration & increased QT interval =
decrease in myocardial excitability & conduction velocity
What are the digoxin toxicities? Color vision disturbances (esp. yellow
green)\n Nausea\n Vomiting\n Diarrhea\n Arrhythmias
What is the MOA of Digoxin?

Directly inhibits Na+/K+ATPase, this

indirectly inhibits the Na+/Ca2+ exchanger & increases intercellular


calcium. \n POSITIVE INOTROPY\n Rx: CHF & atrial fib
What is a toxicity of Quinine?

torsades de pointes\n This drug is an

isomer of quinidine, an antiarrhythmic\n Antiprotozoal action: prevents


separation of dsDNA & thus inhibits DNA replication & transcription
What is the treatment for Chagas?

nifurtimox

What are the 6 most common causes of DIC? 1) Gram sepsis\n2)


Malignancy\n3) Acute pancreatitis\n4) Trauma\n5) Transfusion
reactions\n6) Obstetric complications
What infections are sickle cells, asplenic patients more at risk for?
Encapsulated Organisms: \n Pneumococci\n Meningiococci\n
Haemophilus\n\n& Salmonella osteomyelitis
@ what level of saturation does glucose stop being reabsorbed in the
proximal tubule via the Nagluclose cotransport?

200 mg/dl

What is the pathogenesis of Bruton's? Xlinked agammaglobulinemia:\n


NO signaling molecule (Bruton's tyrosine kinase) to tell B cells to develop
= NO immunoglobulins (low IgG & IgM, NO IgA) b/c of a mutation on the X
chromosome (Xq21.22)\n Susceptible to extracellular pyogenic bacterial
infections: H. influenzea, S. pyogenes, S. aureus, S. pneumoniae
If a patient has no T lymphocytes, what infections are they susceptible to?
Viral & intracellular bacteria

What cell type should be stimulated when a patient is at risk for


neutropenia & throbocytopenia (e.g. because of cancer chemo)? Meloid
stem cell: progenitor of neutrophils, basophils, eosinophils, macs, &
platelets
What enterococcus infection treatment can lead to pseudomembrane
formation?

ampiclin

What anaerobic bacteria infection can result in pseudomembranous colitis


when treated with Clindamycin? Bacteroides Fragilis
What metabolic disease can mimic Parkinson's?

Wilson's: copper builds

up in the putamen\n Dx: low leves of serum ceruloplasmin, increased


heaptic copper content, & increased urinary copper excretion\n Rx: D
penicillamine (copper chelator)
Why do patients on repeated broad spectrum antibiotics develop
spontaneous bleeds?

Gut flora are wiped out & thus Vit. K is not

produced & there is a deficiency in the Vit. K clotting factors: II, VII, IX, X, C
& S. \n VII is deficient 1st b/c it has the shortest 1/2 life & thus
prothrombin time is prolonged 1st.
What vein is the source of the bleed when an alcoholic bleeds from the
mouth?

Left gastric (b/c of the anastomoses between the left gastric

and the azygos)


What is the MOA of Gemcitabine?

Inhibits DNA polymerase &

ribonucleotide reductase when phosphorylated intracellularly\n It inhibits


nucleotide synthesis by inhibiting the formation of deoxyribonucleotides
What is the MOA of Cyclophosphamide?

Alkylating agent: alkylates

DNA (esp. at the N7 position of guanine) that results in interstrand


linkages\n This leads to DNA strand breaks & abnormal base pairing.\n
T/F Interferes with DNA replication
What is the MOA of Dactinomycin?

Binds to & intercalates with dsDNA

to prevent RNA synthesis (particularly rRNA)


What is the MOA of Etoposide? Inhibits topoisomerase II & t/f causes
dsDNA breaks & interferes with cell division at the lateS to G2phase

What drugs should not be combined with cephalosporins (e.g. cefoxitin)?


Aminoglycosides (Tobamycin, gentamicin, streptomycin).\n The
combination results in ATN which reduces GFR & increases serum
creatinine. Can be reversed with discontinuation of the drugs
What is the scope of Azithromycin and what are it's SEs?

Gram +

organisms\n Allergic hepatitis & Thrombophlebitis


What is the main SE of piperacilin?

Hypersensitivity in 10% of

patiensts\n Effective against Klebsiella


What is the difference in cough between croup & epiglottitis?

Croup:

barking (leads to inspiratory stridor)\n Epiglottitis: stridor & hoarseness


(leads to respiratory distress & can require surgery to establish an airway)
What two diseases can causes a dilation of the aorta & dilation of the
aortic ring that results in aortic regurgitation?

1) T. pallidum

(syphilis)\n2) Coarctation of the aorta


What heart murmur is associated with congenital bicuspid aortic valves?
Crescendodecrescendo sytolic ejection following an ejction click:
aortic stenosis (resulting from valvular calcification)
What heart pathology is associated with EhlersDanlos syndrome?
Mitral valve prolapse
What is the most common heart pathology after chronic rheumatic heart
disease?

mitral stenosis

What does an STelevation indicate? What about an STdepression? Q


wave? STElevation = transmural\nSTDepression = subendocardial\nQ
wave = transmural
What is the MOA of the cholera toxin

ADP ribosylation of the GTP

binding protein \n Keeps adenylate cyclate in the active state & thereby
increases chloride secretion
What is the most common nonHodgkin's lymphoma?
t(14:18), bcl2 protein (antiapoptotic) overproduced

Follicular

What translocation is associated with APL version of AML? t(15:17)\n Rx:


alltrans retinoic acid to restore differentiation & conventional chemo
What receptors do Heroin & other opiates target?

u stimulation in brain

and GI:\n Brain = pleasure\n GI = decreased motility & constipation


How does Clofibrate increase the risk of gallstones? It up regulates
lipoprotein lipase (to increase triglyceride clearance from the bloodstream)
but down regulates the rate of bile acid secretion.\n Bile is
supersaturated with cholesterol & this increases the risk for stones
What is the fucntion of Transforming growth factorBeta? Dampens the
immune response
What is the function of Clara cells?

1) Location: terminal bronchiole\n2)

Produce the protein component of surfactant\n3) Transport Cl ions\n4) Site


of cytochrome p450dependent mixedfunction oxidase in the lung
What are Bronchial cells of Kulchitsky? Endocrine cells of the trachea:\n
vesicles contain ADH, serotonin, calcitonin, somatostatin
Where do gram + organisms inhibit penicillin?

Those with B

lactamaseincluding some S. aureus strainssecrete it into the


surrounding space (there is no space between the cytoplasmic membrane
& the peptidoglycan layer of the cell wall)
What acne drug is absolutely contraindicated for pregnant women?
isotretinoin
What are the microbio risk factors for B12 deficiency?

H. pylori\nD.

latum infection\n\n(both can affect directly/indirectly absorption in the


terminal ilium)
What areas of the CNS are affected by a B12 deficiency?

1) Dorsal

Columns: loss of propioception, pressure, vibration, & light touch\n2)


Lateral corticospinal tract: descending fibers to control limb movement\n3)
Spinocerebellar tract: modulation of movement & gait
What is the triad of a molar pregnancy?
bleeding\n3) Hyperthyroidism

1) Hyperemesis\n2) Vaginal

Why does a molar pregnancy present with hyperthyroidism?

BhCG

has a alpha subunit shared with TSH & can sometimes crossreact with the
TSH receptor
What Bunyavirus can cause ARDS?

Hantavirus transmitted via rodent

droppings & saliva


What are the symptoms of PMR associated with giant cell arteritis?
Aching of proximal muscles & girdle stiffness
What is the MOA of Vancomycin?

Gram +s: prevents the cross

linking of DalaDala subunits of the bacterial cell wall


What is the MOA of Doxycycline?

Binds the 30S ribosomal subunit

and prevents aminoacyltRNA attachment


What is the definitive test for chronic granulomatous disease?

Nitroblue tetrazolium dyn reduction: negative result


What is the pathogenesis & presentation of Ataxiatelangiectasia?

Defect in DNA repair enzyme (Ch. 11q2223 mutation) & IgA deficiency\n
Presentation: cerebellar problems (ataxia) & spider angiomas
(telagiectasia) @ a young age\n Also: impaired organ maturation, xray
hypersensitivity, predisposition to malignancy
What is the most common selective immunoglobulin deficiency? IgA: sinus
& lung infections
What is the classic presentation of Bruton's agammaglobulinemia?

Xlinked affected Male\n Normal Tcell count\n Recurrent bac infections


after 6m = . influenzae, S. pneumo, S. aureus
What is neuroleptic malignant syndrome?

Lifethreatening

extrapyramidal side effect of antipsychotic agents:\n hyperpyrexia\n


autonomic instability\n severe rigidity\n\nRx: Dantrolene (muscle
relaxant), discontinuation of all neuroeptics, & supportive care
What is treatment for a benzo overdose?

Flumazenil (competative

antagonist at the yaminobuteric acid receptor

What is the period of maximum susceptibility to teratogens?


Organogenesis: between weeks 3 & 8 of development
What is the fetal period of resistance? Week 1 & 2: either the fetus will be
survive the teratogen or it will fail to survive at all. "allornone" response
What are possible squalae of celiac sprue?

1) Tcell lymphoma\n2)

Dermatitis herpetiformis (pruritic papular lesions located on the trunk &


extensor surfaces)
What is the electron & light microscopic picture for membranoproliferative
glomerulonephritis (MPGN)?

Electron: subendothelial "humplike"

immune deposits & mesangial interposition into the capillary wall\n Light:
tramtracking of capillaries\n\n Young Px (830 years old)
What is the difference between primary and 2ndary Membranoproliferative
glomerulonephritis?

1) Primary = idiopathic\n2) 2ndary = hep C & B,

cryoglobulinemia, lupus, rheumatoid arthritis, maligancy cause\n\n In


either case, 20 years to endstage renal disease
What is Berger's disease? IgA nephropathy: \n IgA in the mesangium\n
most common cause of nephritic syndrome worldwide\n Concurrent with
an upper respiratory infection
How is Warfarin monitored?

Prothrombin time (PT): prevents synthesis

& gammacarboxylation of vitamen Kdepenent factors (II, VII, IX, & X,


proteins C & S) via vit. K antagonism\n\nMneumonic: WEPT: Warfairin,
Extrinsic, PT
What muscle lowers the jaw?

Lateral pterygoids: innervated by

mandibular branch of the trigeminal\n\n"Lateral Lowers, M's Munch"\nM =


Masseter, teMporalis, Medial pterygoid
What are the classic lab findings for obstructive jaundice? Elevation in
alk phosphatase & Direct bilirubin \n Decreased Urine
urobilnogen\n\nLight stools
What is the treatment for Gilbert's disease?

Indirect hyperbilirubinemia:

mild decrease in glucoronyltransferase activity\n\nRx: phenobarbital

What is the difference between direct and indirect bilirubin?

Direct:

conjugated\nIndirect: unconjugated
What are the 2 main risk factors for hepatocellular carcinoma?

1)

Chronic hepatitis\n2) Exposure to toxins (aflatoxin)


What is the histological pattern of CreutzfeldJakob disease (CJD)?
spongiform degeneration:\n vaculated appearance of the cortex
due to deposition of abnormal PrPsc prtoein along with reactive
astrocytosis \n decreased number of neurons in the cortex in the absence
of inflammatory changes
What is the clinical presentation of CreutzfieldJakob disease (CJD)?

Rapidly progressive demintia & ataxia (cerebellar involvement) &


involuntary muscle jerks "startle myoclonus)\n Die within 6 m., onset in
the 7th decade
What disease causes hemosiderosis & gliosis of the mamillary bodies?
WernickeKorsakoff syndrome = secondary to vitamin B1 (thiamine)
deficiency in chronic alcoholics \n\n Hemosiderin is due to repeated
hemorrhages
What are the 3 phases of posterior pituitary acute trauma?

1)

Ceasation of secretion of ADH\n2) Death of pituitary: release of all stored


ADH\n3) Persistent diabetes insipidus dues to injury to posterior pituitary
What does a mild B6 deficiency cause? Reduced decarboxylation
reactions\n Inhibited neurotransmitter production: symptoms of
depression, irritability, confusion, convulsions\n Cheliosis & glossitis\n
Microcytic anemia\n\n Always supplement with isoniazid treatment b/c
isoniazid inactivates pyridoxine by binding with B6
What distinguishes a case of phyelonephritis from a UTI? Presence of
WBC casts in the urine. Indicates renal parenchyma involvement
What causes bilateral acoustic neuromas?

Aka: vestibular

schwannomas\n\nCause: Neurofibromatosis Type 2\n Mutation of the


summor suppressor on Chromosome 22q\n\nSymptoms: hearing loss,
headache, tinnitus, vertigo

What is the mutation involved with Wilms' tumor?

Loss of WT1 tumor

suppressor gene on chromosome 11q


What genetic mutation ca result in reintoblastoma and osteosarcoma? Rb
tumor suppressor gene on chromosome 13q
What chromosomal abnormality results in LiFraumeni syndrome?

AD: Loss of p53 tumor suppressor on Chromosome 17p\n Leaves patients


at a significant rick for malignancy as children or young adults:
osteosarcoma, softtissue sarcomas, earlyonset breast cancers,
adrenocortical tumors, leukemias
What genetic mutation can cause colorectal cancer?

Loss of the APC

tumor suppressor gene on chromosome 5q


What is the role of the pylorus? limit the rate of gastric emptying: it
contracts in response to each peristalic wave
How does swallowing work?

1) Initiation: touch receptors near the

pharyngeal opening are stimulated\n2) Signals are sent to the swallowing


center in the medulla & lower pons\n3) Swallowing center sends impulses
to the muscles of the pharynx & esophagus via cranial nerves
What nerve ends mediate pressure, coarse touch, vibration, & tension
Pacinan (large, ovid, 12 mm by 0.10.7 mm)
What nerve structures are involved in proprioception?

Golgi tendon

organs & muscle spindles (encapsulated mechanoreceptors)


What nerve structures are responsible for light discriminatory touch?
Meissner's (small, encapsulated, on dermal papillae of
glabrous/hairless skin)
What nerve receptors are sensitive to pain caused by mechanical stress,
extreme temperatures, & cytokines such as bradykinin & histamine?
nociceptors
What nerve structures respond to temperature changes?
Thermoreceptors (naked, small, nonmyelinated nerve fibers)

What nerve structures respond to static touch?

Merkel's disks (cup

shaped, encapsulated)
What can help DDx a chest pain to be a pulmonary embolism?

Low O2

saturation
What type of familial hypercholesterolemia is associated with a defect of
LDL cholesterol receptors?

Type IIa

What syndrome is associated with a defect in collagen type III?

Ehlers

Danlos\n\nTriad: Loose joins, decreased skin elasticity, genu recurvatum of


the knee (knee is fixed in hyperextension)
What causes scurvy?

Vit. C deficiency resulting in a defect in

hydroxylation of proline & lysine residues of collagen


What does a P. aeruginosa ear infection present with?

(Swimmer's Ear)

External or malignant otitis externa: ear pain & itchiness\n\nMOA:


produces exotoxin A that ADP ribosylates elongation factor 2 & impairs
host cell protein synthesis\n\nP. aerugoinoa: gram , nonlactose
fermenting rod that forms bluegreen colonies & has a fruity odor
What is the MOA of lipopolysaccharidea (LPS)?

Binds & activates toll

like receptor 4
What is the MOA of Shiga toxin? cleaves rRNA of the host cell
What is the MOA of the superantigen that causes TSST1 (toxic shock
syndrome) by S. aureus? Nonspecificallly binds & activates lympocytes:
fever, rash, shock
What is the primary defect in diabetes type 2?

Peripheral insulin

resistance due to decreased glucose uptake in muscle & adipose\n


Possible cause: defective downstream signaling pathways causing either a
decreased number of insulin receptors on the cell surface or signaling
errors after insulin has bound to the receptor\n\nHyperglycemia presents
with polyuria, polydipsia
Why do obese patients often have hyperestrogenism?

Adipose

contains aromatase that converts androgens into estrogens

Why is there an increase in hepatic glucose production in type 2 DM?


insulin resistance results in a decreased ability to suppress
gluconeogensis in the liver
What is the pathogenesis of paroxysmal nocturnal hemoglobinuria?
There is an insufficient synthesis of glycosylphosphatidylinositol
anchors, which stabilize proteins that protect RBCs from complement
mediated cell lysis
What is the pathogenesis of throbotic thrombocytopenic purpura?
Absences of von Willebrand factor protease & there is hemolysis
due to shear trauma in the vessels
What causes RBC lysis in autoimmune hemolytic anemai? IgG targets RBC
antigens
How is a PDA maintained in an infant & how is it medically closed?
Prostaglandin I2 \n\nRx: indomethacin inhibits PGI2 formation
What are the 3 congenital heart lesions that cause late cyanosis (L to R
shunt)?

VSD\nASD\nPDA

What does Thromboxane A2 cause?

Platelet aggregation &

vasoconstriction\n Inhibited by aspirin


What results from prostaglandin E2 activation?

increase in body

temperature
What results from prostaglandin F2alpha activation?

Uterine

contractions & bronchoconstriction


What are the symptoms of Kawasaki's? What is the treatment?

1) Px < 5

years old\n2) Fever (40C/104F)\n3) Red eyes = injected conjunctivae\n4)


Erythematous rash on palms & soles + edema\n5) Tachycardia & mucosal
inflammation\n6) Lymphademopathy \n7) Lab: Normocytic anemia &
neutrophilic leukocytosis & elevated ESR\n\nRx: Aspirin & IV Ig (prevent
coronary artery aneurysm due to necrotizing vasculitis of sm/m muscular
arteries)
What is Busulfan used for?

Alkylator used to treat CML

What is cylophosphamide used for?

Nitrogen mustard alkylating

agent\n\nRx: cancer, Wegener's granulomatosis & polyarteritis nodosa


What are the main uses of penicillin G?

Syphilis\n Gram & gram

+s that are not penicillin resistant


For what autoimmune disease is prednisone contraindicated?
Kawasaki's b/c it might increase the risk for coronary aneurysm
What amino acid can donate methyl groups? Methionine (SadenosylL
methionine)\n\nE.G. used in the synthesis of phosphatidylcholine
What are the lab results that indicates Edwards' syndrome?

Low:

maternal AFP, estradiol, Bhuman chorionic gonadotropin


What genetic mutation results in DiGeorge's? What are the characteristic
features of this disease? 22q11 microdeletion\n\nCATCH22:\nCleft
palate\nAbnormal facies\nThymic aplasia\nCardiac
defects\nHypocalcemia\n22 deletion
What lab value distinguishes Edwards' from Downs? 1) bhCG: elevated in
Down's and decreased in Edwards\n\n2) SAME: low AFP & low estradiol
What is the picture of Pneumocystis infection?

S/S: dry cough,

dyspnea, hazy bilateral hilar infiltrate on chest Xray\n\nRx:


Trimethoprim/Sulfamethoxzaole
What type of fracture is most likely due to child abuse from bone twisting?
Spiral fracture
What type of fracture is caused by compression and results in a bulging or
buckling of the periosteum?

Buckle: usually caused by accidents

What catalyzes the production of testosterone by Leydig cells?

17alpha

hydroxylase (& 17betahydroxysteroid dehydrogenase)


What does 11betahydroxylase assist in the formation of? Cortisol &
corticosterone in the adrenal cortex\n without this, biological males will
be born with female external genitalia that will be virilized @ puberty
What is type 1 5alphareductactase for?
dihydrotestosterone in the skin

Conversion of testosterone to

What is type 2 5aphareductase for?

Conversion of testosterone to

dihydrotestosterone in the prostate & genital target tissues


What is the maximum point value for APGAR & what is each area
evaluated?

10: \n 7(+) = good survival\n 4(<) = mortality risk\n\n02

for each:\nAppearance\nPulse\nRespiration\nGrimace\nActivity
What is the treatment for mycoplasma pneumoniae infection?
Macrolides (e.g. azithromycin)
What is a common cause of nosocomial UTI & subacute endocarditis?
Enterococcus\n\nRx: ampicillin or vancomycin (if resistant)
What is the MOA & use of piperacillin? Inhibits bacterial cell wall
synthesis\n\nRx: gram +, P. aeruginose, Enterobacteriaceae\n\nUse in
combo with Blactamase inhibitor such as tazobactam
What is female pseudohermaphroditism?

Ovaries are present &

external genitalia are virilized/ambiguous


What is required for development of the penis & scrotum during
embryogenesis?

Dihydrotestosterone via functional 5alphareductase to

convert testosterone to dihydro...\n\nDeficiency causes phenotypical


female with bilateral testes & male internal urogenital tract. @ puberty,
external genitalia will be masculinized
What are the symptoms of rhabdomyolysis? muscle pain, weakness,
"cocacola colored" urine
What is the musclerelated complication of statin use?

myalgia or

rhabdomylosis\n\nEspecially when combined with any inhibitor of


cytochrome P450 enzyme CYP3A4 (that metabolizes statins)\n E.G.
erythromycin
What is the pathogenesis of pyomyositis?

S. aureus infection of skeletal

muscle\n due to bacteremia from IV drug use or dental work)\n often


large leg muscles\n S/S: muscle pain, swelling, fever
What genetic disorder predisposes patients to thromboses in unusual
locations (e.g. mesenteric veins) at an early age (<50)?

Factor V Leiden

thrombophilia\n heterozygous have a higher risk for thrombotic events

What does Fanconi's anemia predispose patients to?

These patients

have pancytopenia and have an increased risk of infection, anemia,


fatigue, & bleeding
How is von HippelLindau disease characterized?

abnormal blood vessel

growth leading to angiomas and hemangioblastomas in the:\n Retina\n


Brain\n Spinal Cord\n & elsewhere
How does von Willebrand's present?

Prolonged bleeding time after

trauma, surgery, nosebleeds, & hematomas


How does Giant Cell arteritis present histologically? Nodular thickening
that reduces the size of the lumen\n Granulomatous inflammation with
mononuclear & giant cells\n Fragmentation of the internal elastic
membrane
What infection causes tabes (i.e. shriveled) dorsalis?

T. pallidum =

neurosyphilis progressively demyelinates & scleroses the posterior nerve


roots\n reduced reflexed, decreased pain, decreased proprioception (will
show a positive Romberg test)\n\nRx: doxycycline
What is Pott's disease?

When TB invades the spinal column & causes

bone destruction & compresses region of the spinal cord\n 1st symptom
is pain\n\nRx: isonizaid + rifampin
Where does poliovirus affect the spinal cord? Motor neurons in the anterior
horn\n\nRx: prevent via vaccine
How does alpha1antitrypsin deficiency harm the liver?

intrahepatic

accumulation of abnormal alpha1antitrypsin protein


What are the estertype local anesthetics?
Tetracaine\nChloroprocaine\nProcaine\nNovacaine\nPropoxycaine\n\nIf a
Patient is allergic to one, avoid all
What are the amidetype local anesthetics?
Bupivacaine\nEtidocaine\nLidocaine\nPrilocaine
What vessel prohibits a horseshoe kidney from ascending?
mesenteric artery

inferior

What is WaterhouseFriderichsen syndrome? Bilateral necrosis &


hemorrhage of the adrenals cause by infection with N. meningiditis\n\nDx:
high potassium level, low Na, & no increase in serum cortisol level in
response to ACTH administration \n\nRx: cephalosporin, penicillin,
chloramphenicol & glucocoritcoid replacemtn therapy with hydrocortisone
What is the MOA of statininducted ATN?

secondary to obstruction or

via toxic effect on renal tubules


What color is a patient's urine when they have myoglobinuria?

Tea

colored
What are causes of interstitial nephritis?

Allergic reaction to drugs

(NSAIDS, diuretics, antibiotics)\n Infections\n Infiltrative process


What is the typical presentation of a Becker's (BMD) patient?

Muscle

weakness &/or cardiac disease a mid adolescence\n Cause: an inframe


deletion in the dystrophin gene: partially functional protein is produced
(vs. DMD = out of frame that results in truncated or absent dystrophin)
What is the most common cause of mitral stenosis? Rheumatic heart
disease\n delayed, rumbling, late diastolic murmor
What is the most common valvular lesion?

mitral prolapse\n late

systolic murmor with a midsystolic click\n predisposes to infective


endocarditis
What are the main causes of aortic stenosis? Calcified valve\nCongenital
bicuspid aortic valve\nRheumatic heart disease
What are the physical exam finding that indicate aortic regurg? high
pitched blowing diastolic murmur with widened pulse pressure
What are the physical exam finding that indicate aortic stenosis?
Crescendodecrescendo systolic ejection murmur that radiates to the
carotid \n Weak peripheral pulse (pulsus parvus et tardus)
What are the physical exam finding that indicate mitral regurg?
holosytoci highpitched blowing murmor @ the base of the heart
with a radiation to the left axilla

What are the physical exam finding that indicate mitral stenosis?Opening
snap with a delayed, rumbling latediastolic murmur
How does the kidney respond to a high sodium diet?

Volume

expansion will increase stretch in baroreceptors located in the afferent


arteriole\n This leads to a reduced sympathetic activity\n This leads to
vasodilation of the glomerular afferent arterioles\n Increases GFR &
decreases Na reabsorption
When is atrial natriuretic peptide secreted?

By the atria in response to

increased extracellular fluid volume & causes dilation of the glomerular


afferent arterioles
What are the toxicities of cisplatin?

Nephrotoxicity & acoustic nerve

damage\n\n It is a cell cycle nonspecific drug that alkylates DNA (cross


links and inhibits DNA synthesis)
What are the side effects of Busulfan? DNA alkylator \n\n PULMONARY
FIBROSIS\n hyperpigmentation\n bone marrow suppression\n GI & CNS:
insomnia, anxiety, headache
What is the toxicity of Doxorubicin?

DNA intercalator that leads to

strand breaks (Rx: breast cancer)\n\n Cardiotox (dilated


cardiomyopathy)\n Myelosuppression & alopecia
What are the side effects of etoposide plant alkaloid that acts in the late S
and early G2 & inhibits topoisomerase II\n\n Tachycardia\n Nausea,
vomiting\n Shortness of breath\n hematuria\n melena\n
myelosuppression\n alopecia
What is methotrexate used for? Antimetabolite that inhibits dihydrfolate
reductase in the S phase b/c it is structurally similar to folic acid:
decreases purine nucleotide
synthesis\n\ncarcinoma\nleukemia\ntrophoblastic disease\nrheumatoid
arthritis\nectopic pregnancy\npsoriasis\nabortifactant
What are the side effects of methotrexate?
myelosuppression\nmucositis\nelevated liver enzymes\nrenal
toxicty

What is the use of ondansetron? antiemetic: selective serotonin receptor


antagonist\n\nSE: headache, malaise, constipation
What is the MOA of Vinblastine? binds tubulin and blocks microtubule
polymerization & mitotic spindle assembly\n\nSE: bone marrow
suppression, alopecia, SIADH
What is the MOA of acyclovir, ganciclovir, and famciclovir? Inhibits viral
DNA polymerase
What 5 types of antibiotics inhibit protein synthesis?

1)

chloramphenicol & clindamycin\n2) linezoid\n3) erythromycin\n4)


tetracycline & doxycycline\n5) aminoglycosides
What is the treatment for Rocky Mountain spotted fever (R. rickettsii) &
Lyme disease (B. burgdorferi) infection?

Doxycycline (inhibits protein

synthesis)
What is the treatment for ectopic pregnancy? laparoscopic rescection
What is the frequency of the slow waves of the stomach and small
intestine?

Stomach: 3Hz\nSmall intestine: 12 Hz

What is Whipple's triad for an insulinoma?

1) Hypoglycemia\n2)

Hypoglycemic symptoms (headache/irritability)\n3) Relief of symptoms


with calorie intake\n\nLAB: high Cpeptide (indicates taht serum insulin is
endogenous)
How is Pneumocystis jiroveci pneumonia diagnosed?

Methenamine

silver stain of lung biopsy tissue\n\nRx: \n Sulfamethoxazole


trimethoprim\n pentamidine\n dapsone
What is the MOA of sulfamthoxayoletrimethoprim & trimethoprim?
Inhibit folate synthesis: pteridine & PABA are incorporated into folic
acid\n\n= inhibition of nucleic aci & amino acid formation
What is the MOA of antifungals like fluconazole & terbinafine?
of ergosterol synthesis
What antibiotics inhibit the 50S subunit?
Chloramphenicol\nErythromycin\nClindamycin\nLinezolid

Inhibition

How is N. gonorrhoeae cultured?ThayerMartin agar (chocolate agar plate


suffused with vancomycin, colistin, & nystatin (VCN) to suppress
endogenous growth)
What is BordetGengou (aka potato) agar or ReganLowe medium used to
culture

Bordetella pertussis (gram + rod: whooping cough)\n Also

collect with calcium alginate swabs


What agar is used to cuture Legionella pneumophilia?

Charcoal yeast

extract with increased iron & cysteine


What agar is used to culture TB LowensteinJensen (no growth of gram
positive respiratory flora)
What is MacConkey's agar used to culture?

Lactosefermenting

enterics:\nE.coli\nKlebsiella
What drugs can cause SIADH?

1) Highdose IV cyclophosphamide\n2)

Carbamazepine\n3) Vincristine\n4) Vinblastine\n5) Amitriptyline\n6)


Amiodarone\n7) MAO inhibitors
What are the clinical features of SIADH?
nausea\nlethargy\nseizures\ncoma\n\nLab: \n hyponatremia\n
serum hypoosmolality\n urine hyperosmolarity\n decreased hemocrit
(2ndary to dilution
What are 2 drugs that can be used to treat nephrogenic DI?
Indomethacin\nHCTZ (hydrochlorothiazide)
What is a direct Coomb's test?

1) Coombs reagent is added to the

patient's blood\n2) Binds to autimmune Ig bound to several RBCs & causes


agglutination
What can IgM cold agglutinin immune hemolytic anemia occur
concuurently with? Mononucleosis\n Mycoplasma pneumoniae\n
Lymphoid neoplasm (chronic anemia only)
What is an indirect Coombs' used to assess

Blood compatibility (before a

transfusion): detects unbound antibodies that can bind to RBCs from a


normal, healthy doner.

What is a neural complication of overaggressive treatment of


hyponatremia (correction of the Na level at a rate faster than 1mEq/L/Hr)?
Central pontine myelinolysis (Osmotic demyelination): rate of
osmolarity outstrips the brain's ability to adapt to the rising osmolarity >
neuronal shrinkage & death\n\nClinical: dysarthria, dysphagia, flaccid
quadriparesis (can lead to spastic or lockedin syndrome)
What is lockedin syndrome?

When a patient is fully aware, but can

move only the extraocular muscles


What is the treatment for bulimia?

SSRIs (e.g. fluoxetine)

What are the clinical signs of bulimia? Calluses/Scars on the dorsum of


the hand (abrasion by teeth during induced gaggin)\n Parotid
enlargement\n Erosion of tooth surfaces from enamel dissolution
What is Lorazepam used to treat?

Anxiety disorders

What are amphetamines used to treat (main 2)?


ADD/ADHD\nNarcolepsy\n\nModulate dopamine, serotonin, & NE
What is Thioridazine used to treat?

Schizophrenia (as a last resort b/c

of SE)\n\nSE: neuroleptic malignant syndrome, cardiotoxicity, retinopathy


At what point in meiosis does trisomy 21 occur (i.e. the nondisjunction)?
Anaphase I or II (mitotic spindle unequally separates 21, usually in
the mother's gamete)
Where is a Robertsonian translocation?A translocation between
chromosome 21 & long arm of a second acrocentric chromosome (usually
14 or 22)
When does crossover or recombination occur in meiosis? Phrophase I
What is characteristic of MS (anatomical changes & symptoms)? CNS
demyelination with neurologic lesions within the white matter\n S/S that
are separated in time & anatomic location\n Can have difficulty urinating
or incomplete voiding\n IMPORTANT: episodic symptoms last >24 hours;
MRI: periventricular plaques in >90% of Pxs

What do intention tremors & ataxia indicated?

Cerebellar disease

(e.g. with MS)


What is the triad for Huntington's?

Chorea (forceful, spastic

involuntary movements of the limbs &/or facial muscles with gradual onset
but become permanent)\n Muscle rigidity\n Dementia\n\nCause:
nucleotide repeat expansion on the short arm of Ch. 4
What happens in latestage amyotrophic lateral sclerosis? Paralysis of the
respiratory muscles
What are the 4 main manifestations of NF2 (neurofibromatosis type 2)?
Bilateral schwannoma (neoplasm at cerebellopontine angle (CN VII & VIII
can be affected)\n Eye lesions (juvenile cataracts)\n Ependymomas\n
Meningiomas\n\n NF2 tmorsuppressor on Ch.22 that encodes a
membrane cytoskeletal protein
What are the manifestations NF1 (Neurofibromatosis type 1)?

Cafe

aulat spots\n neurofibromas\niris hamartomas\n optic pathway


gliomas\n pheochromocytoma\n scoliosis (or other skeletal disorders)\n
UNILATERAL schwannomas)
What is the origin of neurofibromas

Schwann cells + perineural &

endoneural components
What is the classical presentation of CharcotMarieTooth? Distal muscle
(esp. calve) atrophy\n Clumsiness\n\nGenetic disorder that affects
peripheral nerves via mutations in various myelin genes
What are the characteristic facial features of fragile X?

Large Ears &

Jaws, & MacroOrchidism in males \n\nEVERYTHING (chromosome, face...)


is BIG
What is the treatment for Bruton's agammaglobulinemia? IV Ig\n\nNo
tyrosine kinase gene = no B lymphocyte differentiation
What lab results show elevated mineralocorticoids (along with a lack of
other cortical steroids)?

Hypertension\n Hypokalemia\n Female w/o

sexual maturation\n\n17alphaHydroxylase deficiency

What 2 enzymes are needed to produce cortisol? sex hormones?


Cortisol:\n21betahydroxylase\n17alpha
hydroxylase\n\nSex:\n17alpha hydroxylase
What are the ocular manifestations of a Berry aneurysm? Cranial nerve
(CN III) compression due to the rupture of a thinwalled outpouching at a
cerebral arterial branch point (anterior cerebral & communicating) \n
parasympathic fibers are compressed and this results in dilated,
unreactive pupils (palsy)
How does blockage of a penetrating artery in the internal capsule present?
pure motor or sensory deficits
How does the blockage of an anterior cerebral artery present?

Stroke:

contralateral motor & sensory deficits in the leg & foot


How do subdural hematomas present on CT? "crescentshaped" density
bridging veins (between dura & arachnoid) rupture do to shearing forces
What is the usual cause & presentation of a middle meningeal artery
rupture?

Epidural hematoma due to a lateral blow to the head\n\nCT:

"lensshaped" density
How does Ketoconazole or metyrapone inhibit ACTH effects?

Both

inhibit cytochrome P450 in the adrenal glands (& ketoconazole inhibits


ACTH)\n\n Can be used to treat SIADH from small cell lung cancer
What is the MOA of Desmopressing acetate (DDAVP)?

It is a synthetic

analog of ADH & binds to V2 receptors in the collecting ducts to increase


water resorption\n\n Used to treat diabets insipidus\n Stimulates release
of vonWillebrand factor & factor VIII by stimulating the V1a receptor (used
to treat coagulation disorders: vW disease, mild Hemophilia A,
thrombocytopenia)
What is the MOA & use for Finasteride? MOA: Decreases conversion of
testosterone to dihydrotestosterone\n\n Rx: BPH hypertrophy
What is Pamidrone used to treat?
Hypercalcemia

(Bisphosphonate)\n\nRx:

What is spironolactone used to treat (besides hypertension)?

Can be

used to treat Conn's (primary hyperaldosteronism)\n\n It is a potassium


sparing diuretic & also competitively inhibits the mineralcorticoid
receptor\n It is a steroid
What is commonly used to treat neuroleptic malignant syndrome?
Bromocriptine (a dopamine agonist to restore the depletion of
dopamine signaling caused by drugs like haloperidol that blocks dopamine
receptors)
What is Benztropine used for?

It is an antimuscarinic that improves the

tremor & rigidy in patients with Parkinson's


What 2 antipsychotics have the highest rist of neuroleptic malignant
syndrome?

Chlorpromazine\nHaloperidol

What is the MOA of Clozapine?

Atypical antipsychotic that blocks

serotonin & dopamine receptors (D1 & D2)\n\nSE: agranulocytosis &


seizures in nonepileptic Px
What is the MOA of Fluoxetine? Serotoninspecific reuptake inhibitor
What is the pathogenesis of midgut volvulus?Complication of intestinal
malrotation in the 1st month of life when the midgut rotates less that 270
degrees (between week 10 & 12)
How does midgut volvulus present on Xray? Clinically? What are possible
complications?

Xray: Twisted & overdistened colon with a coffeebean

shape & lack of septa & haustra \n\nClinical: vomiting &


constipation\n\nComplications: compromised blood supply > ischemic
necrosis & perforation
Describe the Measles rash.

Blanching red that starts on the face &

then moves down the trunk & limbs \n\n+ fever


What is a rare complication of measles?

SSPE: subacute sclerosing

panencephalitis \n 79 years after infection\n personality changes\n


lethargy, difficulty in school, odd behavior\n Late changes: dementia,
severe myoclonic jerking, eventyal flaccidity & decorticate rigitiy > rip

What is the presentation of pediatric roseola?High fever (35


days)\nconjunctivits\notitis media\nmalaise\n\n Blanching macular rash
on the neck and trunk that spreads to face & extremities
What is the Rubella of rash like? Descending: starts on the face\n\n fever
before (& resolves) rash, lymphadenopathy
What is the presentation of Scarlet fever?

Strep pyrogenes: fever, soar

throat, strawberry tongue\n\nRash: trunk, neck, limbs, NO palms/soles.


\n\nPost rash: desquamation of palms & soles
What is the MOA of fluoroquinolones (such as ciprofloxacin)?

Inhibit

DNA gyrase & topoisomerase of gramnegative bac.


What nerve roots contribute to the axillary nerve?

C56 (posterior cord

branch)
What is the clinical presentation of a cocaine user? Tachycardia\nDilated
pupils\nAngina\nNosebleed\n\nMOA: sympathomimetic that inhibits
neurotransmitter reuptake (e.g. NE)
What causes prerenal azotemia?Reduction of GFR due to a decreased
vascular supply to the kidney\n Heart failure\n Sepsis\n Renal artery
stenosis
What are the classic lab values of prerenal azotemia?

Accumulation of

BUN & Cr in blood:\n BUN rises out of proportion to Cr (Cr is filtered, no


reabsorbed) = elevated BUN: Cr ratio
How does AIN (acute interstitial nephritis) present? eosinophilic casts in
the urine\nelevated fractional excretion of Na
What distinguishes postrenal from prerenal azotemia?

Post: normal

BUN: Cr ratio, lack of highly concentrated urine\n\nPre: increased BUN:Cr


ration, highly concentrated urine
What level are the nerves that control erection?

S24 (sympathetic

fibers)
What level are the nerves that control erection?
fibers)

S24 (sympathetic

When does fibrinous pericarditis occur post MI?

35 days\n\nS/S:

sharp, pleuritic chest pain, fever, pericardial friction rub


When is a thrombotic emboli a risk post MI? weeks to months later
When is a heart rupture a risk post MI? 410 days
What are the main functions of smooth ER?

steroid

biosynthesis\nmembrane synthesis\nrepair\ndetoxification (liver has a lot)


What are the S/S of Rocky Mountain spotted fever? fever\nsever
headache\nred conjunctivae\nrash on palms and soles\n\nRx: tetracycline
(e.g. doxycycline) or chloramphenicol FAST (or death)
What lab test can confirm a rickettsial infection (rash on palms & soles)?
WeilFelix reaction for this gram bac\n Serologic fluorescent
antibody screen \n Positive: crossreacting antigens from Proteus vulgaris
What type of infection is cold antibody agglutination used to detect?
Mycoplasma
What are the vectors for Rickettsia rickettsii? Dermacentor andersoni & D.
variabilis (ticks)
What is the tumor marker for breast cancer? CA153
In what cancers is alphafetoprotein elevated?

hepatocellular

carcinoma and nonseminomatous germ cell tumors (yolk sac tumor)


What is Bombesin a tumor marker for? neuroblastoma, lung, & gastric
cancer
What tumors can CA125 mark? Ovarian & malignant epithelial tumors
What is S100 a tumor marker for?

melanoma\nneural

tumors\nastrocytoma
What tumors is Tartrateresistant acid phosphatase a marker for?
Hariy cell leukemia (a Bcell tumor)
What is volvulus?

A twisting of a portion of the bowel around its

mesentery\n\n PEDs & elderly

How doe spyloric stenosis present?

S/S: nausea & vomiting at 3 to 6

weeks post birth; oliveshaped mass on physical exam\n\n Patho: pyloric


muscle hypertrophies & causes gastric outlet obstruction\n\n Rx: surgical
pyloroplasty
What is the pathogenesis of Meckel's diverticulum? Incomplete
obliteration of the vitelline duct (yolk stalk)\n\n2% of population & 2 ft.
from ileocecal junction; usually asymptomatic\n\nComplication: bleeding
What is intussusception & how does it present?

Invagination of

intesting upon itself (usually at ileocecal junction) @ 636 m. old\n\nS/S:


intermittent & crampy abdominal pain with currentjelly stool due to rectal
bleeding
What is cystinuria? Increased levels of lysine & arginine in the urine due to
a defect in amino acid reabsorption in the proximal tubule leading to a
wasting of:\n lysine, arginine, cystine\n\nElevated cystine leads to
recurrent stone formation (S/S severe pelvic pain & elevated creatinine
due to postrenal failure caused by the stone)\n\nRx: hydration &
alkalization of urine
What level are the nerves that control erection?

S24 (sympathetic

fibers)
What is the classic presentation for prolactinoma (most common pituitary
adenoma) in a woman?

Headaches \nGalactorrhea\nBitemporal

hemianopsia
How does prolactin suppress menstration?

inhibits FSH and GRH &

promotes breast milk production


What is thyroid dermopathy?

Pretibial myxedema (nonpitting &

thickening of skin)\n Characteristic of hyperthyroidism due to Graves


When there is truncal obesity due to a pituitary adeonma, what is the
cause of the obesity?

ACTH hypersecretion

What are the neurofibrillary tangles of Alzheimer's composed of?


intracellular inclusions of microtubule binding protein tau

What is the most common cardiovascular abnormality seen in congenital


rubella syndrome? PDA
What is the presentation of congenital CMV infection?

petechial

rashes\njaundice\nhepatosplenomegaly\nsensorineural hearing loss


How does congenital herpes infection present?

skin (vesicles), eyes

(keratoconjunctivitis), and mouth (vesicles) or localized CNS infection, all


within 4 weeks of birth\n CNA: temp. instability, RDS, poor feeding,
letharcy
What is the triad for congenital toxoplasmosis?
chorioretinitis\nhydrocephalus\nintracrainal calcifications\n\nEarly:
maculopapular rash, jaundice, hepatomegaly
How does congenital syphilis present? asymptomatic at birth\n\n Cranial
VIII defect (hearing problems)\n Cutaneous lesions (palms & soles)\n
Anemia, jaundice, hepatomegaly
What spinal nerve controls the inguinal region dermatome?

L1: also

includes the femoral triangle and superolateral quadrant of the buttock


What spinal nerve controls the anterior & lateral thigh dermatome?

L2

What spinal nerve controls the dermatome at the level of the umbilicus?
T10
What spinal nerve controls the dermatome directly inferior to the
umbilicus?

T11

What spinal nerve controls the dermatome region interior to the anterior
iliac crest?

T12

What lab values indicate an inability to dilute urine? Urine osmolaity >
plasma osmolality\n Normal specific gravity\n Urine Na >40mEq/L
What ECG is indicative of WPW (WolffParkinsonWhite) syndrome?
Wide QRS with relatively short PR invervals & slurring of the initial
parts of the QRS complex\n Cause: congenital accessory pathway leads
to preexcitation of the ventricle; the slurred upstroke is the delta wave\n
Result: suptraventricular tachycardia: palpitations followed by syncopy

What causes an S4 heart sound?

stiffness of hypertrophied ventricle

What heart sounds indicate hypertrophic cardiomyopathy?

S4 (due

to stiff walls)\nEjection murmur (mild outflow tract obstruction due to


septal hypertrophy)\n\nRx: bblocker or nondyhydropyrine Ca channel
blocker (verapamil)
Which rotator cuff muscle medially rotates the arm?subscapularis
What is the action of the infraspinatus?

adducts the arm and laterally

rotates the glenohumoral joint


Which rotator cuff muscle is most often injured?

supraspinatus:

initiator of abduction (first 15 degrees of the arc; deltoid does the rest)
What is the action of teres minor?

adduct & laterally rotates the arm

What is the key component of fungal endospores that allow them to resist
dehydration, heat, & chemicals? dipicolinic acid (e.g. present in
Coccidioides immitis: causes coccidioidomycosis in the southwest US)
Which virus is known for its hemagglutinin antigen influenza
What bacteria has a capsule made of Dglutamate Bacillus anthracis
What is the action of Teichoic acid on gram + bacteria?

induces tumor

necrosis factor alpha and interleukin1 production


What is the ECG finding in tamponade?

Sinus tachycardia & low

QRS voltages\n Elecrical alternans (alternations of QRS height with each


beat) is pathognomonic
What does an ECG result of PR elongation indicate? 1st degree
atrioventricular block
What does a peaked or hyperacute T wave indicate?

hyperkalemia

(or, early MI or myocaridal ischemia)


What are the causes of QT prolongation?

1) Drugs:\n Class IA or III

antiarrhytmics\n Phenothizines\n TCAs\n2) Hypocalcemia\n3)


Hypothermia\n4) Cerebrovascular diseases\n5) Ischemic heart disease\n6)
Idiopathic

What is the site of lymph drainage of the liver?

Space of Diss

(between sinusoidal endothelial cells & hepatocytes)


What heaptic vein carries deoxygenated blood from the liver back to the
heart?hepatic vein
What proteins are affected by tuberous sclerosis?

AD (Ch. ( & 16):

tuberin & hamarin, they regulated cellular growth &


differentiation\n\nTumors: cortical tubers, renal angiomyolipomas, cardiac
rhadbdomyomas, astrocytomas, pulmonary
lymphangioleiomyomatosis\n\nPhysical: ashleaf spot, Shagreen patch,
facial angiofibromas\n\nPresentation: mental retardation & epilepsy
What is the pathogenesis of Leber's hereditary optic neuropathy?

Mitochondrial mutation that causes degeneration of the optic nerve with


rapid loss of central vision, leading to a permanent central scotoma\n
Males are more affectwith & symptoms usually start in the 3rd decade
What is the bac most often responsible for osteomyelitis in diabetic
patients?

pseudomonas aeruginosa

What is the cellular lining of the epididymis (where sperm acquire


motility)?

pseudostratified columnar epithelium with stereocilia

What is the cellular lining of the vas deferens (where the sperm travel
after maturing in the epididymis)?

pseudostratified columnar

epithelium
What is the cellular lining of the rete testes (@ the testicular hilum &
where the sperm is transfered from the sminiferous tubules to the ductuli
efferentes to the epididymis)?

simple cuboidal cells

What cell type lines the bladder, calyces, ureters, & upper urethra?
transitional cell epithelium
What is the treatment for Clozapineinduced agranulocytosis?

1)

Discontinuation of clozapine & \n2) Administration of Granulocyte colony


stimulating factor (GCSF)
What lab results indicate primary hyperparathyroidism?

Hypercalcemia

> metastatic calcification & duodenal peptic ulcers\nhypophosphatemia

\nincreased alk phos\nincreased PTH levels > osteitis fibrosa cystic


(excessive bone resoption & fibrous replacement of the marrow leaving
cystic spaces (brown tumors)
What is Osteitis fibrosa cystica and what causes it? Excessive bone
resoption & fibrous replacement of the marrow leaving cystic spaces
(brown tumors)\n Increased PTH levels\n Presentation: bone pain &
fractures
On what chromosome is the FAP (familial adenomatous polyposis) genetic
mutation found?

5: APC locus, loss of tumor suppression & increased

risk of:\n duodenal\n gastric\n liver\n thyroid\n CNS


What is the DOC for H. ypylori infection?

Clarithromycin, amoxicillin, &

PPI \n 14 day regiment\n\nOR\n\nMetronidazole, tetracycline, bismuth


subsalicylate, & PPI
What is the most common cause of 2ndary hyperparathyroidism?

Renal failure\n Symptoms: diffuse bone pain & proximal muscle weakness
(+ pruritis if there is a deposition of excess Ca)
What is renal osteodystrophy?

1) Nephron damage (Lab: increased

creatine) reduces Ca reabsorption, impaired phosphate excretion &


impaired vit. D activation.\n2) Result: decreased serum Ca & increased
phosphate = hypocalcemia\n3) PTH increases & bone is reabsorbed:
increased alk phos.
What are the symptoms of hypocalcemia?

tetanny & incrased

neuromuscular excitability + decreased serum PTH


How does maligancy cause hypercalcemia?

1) Lytic metastases to

bone \n up serum alk phos & hyperphosphatemia\n2) PTHrelated


peptide production (with hypophosphatemia)
What are the lab results that indicated Vitamin D intoxication?
hypercalcemia\nhyperphosphatemia\ndecreased serum intact PTH
levels

When do extrapryamidal adverse effects of typical antipsychotic set in?1)


acute dystonia (1st hours after dose)\n2) akinesia (1st few days after
dose)\n3) tardive dyskinesia (several months)
What renal disease often presents in otherwise healthy young males with
hemoptysis? Goodpasture's: Dx with antiGBM antibodies in blood &
kidney biopsy\n\nRx: plasmapheresis & airway protection + steroids &
cyclophosphamide
Which strands of strep tend to cause poststreptococcal
glomerulonephritis?

M type \n Autoimmune attack begins 26 wks.

after skin infection & 13 weeks after pharyngitis


What drug can be used to "rescue" bone marrow after methotrexate
administration?

It delivers folinic acid that can be reduced to

methylene tetrahydrofolate & will thereby bypass the dihydrofolate


reducate enzyme that is inhibited by methotrexate
What is Mesna used for? Prevent hemorrhagic cystitis in patients taking
cyclophosphamide:\n supplies a free thiol group to bind to & inactivate
the toxic metabolite acrolein in the kidney
What is compartment syndrome?

when a lacerated artery bleeds into

a closed space:\n greater than expected pain\n decreased blood flow @


the site of injury
What are the HLA b27 diseases? PAIR:\nPsoriasis\nAnkylosing
spondylitis\nIBD\nReiter's syndrome
Disorders of peroxisomes result in what 2 diseases? 1) Refsum's\n2)
Adrenoleukodystrophy\n\n peroxisomes can no longer metabolize very
longchain fatty acids & this results in problems with making myelin &
hydrogen peroxide formation = progressive neurological damage
What test approaches 100% sensitivity for detecting syphilis?

Veneral

Disease Research Laboratory (VDRL) test: FTAABS absorption test


Describe the 3 stages of syphilis?

1) painless chancre (localized)\n2)

SYSTEMIC (6 wks later): maculopapular rash (palms & soles), condylomata


lata\n3) Gummas (chronic graulomas), aoritis (vasa vasorum destruction),

neurosphyilis (tabes dorsalis), Argyll Robertson pupil. SIGNS: broadbased


ataxia, positive Romberg, Charcot joints, stroke w/o hypertension
What are the signs of congenital syphilis?

Broadbased ataxia\nPositive

Romberg\nCharcot joints\nStroke w/o hypertension


What is the Tzanck preparation with Giemsa stain used for?

1) HSV:

reveals multinucleated giant cells \n2) Vesicular stomatitis virus infection


When can an individual possess the capacity for awareness of a full
rectum/bladder & are able to control bowel & urinary tract sphincters?
1936 months
When is the oedipal/phallic phase?

366 years: strong attachment to

parent of the opposite sex


When is the oral stage of life?

1st year: passive dependence on eating &

sucking
When is the latency phase of life?

6 years puberty: children do not

fixate
What infection & CD markers are associated with Hodgkin's disease?
EBV\nCD45,CD30+, CD15+/
What are the risk factors for Hodgkin's lymphoma? Male Gender\n Age
1540 or 50+\n EBV infection (history, not current)\n
Immunodeficiency/AIDS\n Prolonged use of growth hormone
How are the lymph nodes changed in Bartonella heniselae infection?
stellate abscesses & tender\n\nAlso, there can be sterile,
suppurative papules at the site of inoculation
What does mucosis fungoides initially present as?

Eczematous,

psoriaticlike rash\n\n It is a cutaneous Tlymphocyte lymphoma


What H2antagonist is an inhibitor of the cytochrome p450 system?
cimetidine (Rx: ulcers; SE: headache, confusion, gyncomastia,
thrombocytopenia)
What FEV1/FVC ratio is indicative of a restrictive disease? >80

What can be a severe compication of bleomycin therapy? pulmonary


fibrosis (~1 month after initiation of treatment) & ARDS
What prostaglandin E1 analog is contraindicated in pregnancy (& why)?
Misoprostol: increases producing & secretion of gastric mucous
barrier\n it is an abortifactant & can cause diarrhea
What are the most common causes of IUGR (intrauterine growth
restriction)? Poor maternal nutrition\n Hypertension\n Infections\n
Congenital anomalies\n Smoking
What are the risks of placenta previa? (Placenta covers the cervical
os):\n Prior cesarean section\n Multiparity\n Advanced maternal age\n
Multiple gestations\n Prior placenta previa
Where does Triatoma cruzi infect?

Heart: myocarditis (ventricular

dilation & congestive heart failure 2ndary to myocyte necrosis &


fibrosis)\n Nerve cells of the myenteric plexus: dysmotility of hollow
organs (esophagus, colon, ureter) = acquired achalasia (distal 1/3) &
megacolon/ureter
How does psoriatic arthritis present?

psoriasis (nonpuritic scaly or silvery

erthematous plaques with welldefined borders) + joint symptoms with


acute onset in 1/3 of patients\n Joint swelling asymmetrically in distal
interphalangeal joints\n some patients have sausagelike finger from
inflammation of the digital tendon sheaths
What skin disorder goes along with celiac disease? dermatitis
herpetiformis: pruritic papules & vesicles
What is deposited in joints in pseudogout?

Ca pyrophosphate dihydrate

crystals in CONNECTIVE tissues: weakly bifringent & rhomboidal in shape


What is the most common selective Ig deficiency syndrome & how does it
present?

IgA: mucosal deficiency leads to sinus & lung infections

What is the most common type of primary malignant tumor found in the
liver? HCC: hepatocellular carcinoma
What causes a bicornuate uterus?

Failure of complete fusion of the

paramesonephric ducts at there superior end.

What causes uterus didelphys? Complete failure of fusion of the


paramesonephric (Mullerian) ducts\n Results in 2 seperate uterine
cavities that each have a cervix & vagina
What cause a arcuate uterus?

(aka septate uterus)\nFailure of resorption

of the midline uterine septum


Where is Schistosoma hematobium endemic? What does it cause?

North Africa & Middle East (parasitic flatworm)\n Infection via snails;
enters bladder can cause squamous cell carcinoma of the bladder\n Rx:
Praziquantel
What does Dracunculus medinensis cause?

(aka Guinea worm)\n

Cutaneous nodules & ulceration: resides in subcutaneous tissue & the tail
of adult female can protrude to from a small ulcer (usually found on the
Patient's lower limb)
Where is Necator americanus found (in the world)? (aka hookworm)\n
Americas\n subSaharan Africa\n Southeast Asia
What does Schistosoma masoni cause?

intestinal schistosomiasis\n

Intestinal cancer\n Liver infection\n Katayama fever\n\n(subSaharan


Africa, South America)
What is used to treat endometriosis?

Danazol (androgen): to decrease

GRH to reduce luteinizing hormone & FSH secretion (decrease growth of


endometrium)\n\nSE: menstrual irregularities, thrombotic events, benign
hepatic adenomas, benign intracranial hypertension (psuedotumor
cerebri), weight gain, & acne
What antiantrogen is used to treat BPH & male pattern balness?
finasteride (5alphareductase inhibitor)
What antiantrogen is used to treat prostate cancer? flutamide (competitive
androgen inhibitor @ the testosterone receptor)
What antiantrogen is used to treat hirsutism & polycystic ovarian
syndrome?
synthesis)

ketoconazole (inhibits adrenal desmolase needed for steroid

What antiantrogen is used to treat class III/IV congestive heart failure,


primary hyperaldosteronism, diureticinduced hypokalemia)?
sironolactone (steroid binding inhibitor)
What differentiates clomiphene & tamoxifen? Tamoxifen: has agonist
activity & promotes endometrial hyperplasia\n Cloomiphenes: has
antagonist activity: inhibits endometrial hyperplasia
In which tissues does clomiphene & tamoxifen act as an antagonist?

1)

CNS = vasomotor symptoms & sleep irregularities\n2) Breast = breast


cancer protection\n3) bone PTH hormone = bone pain, hypercalcemia,
arthralgias\n4) Lipid profile = incrases HDL & triglycerids and decreased
total & LDL cholesterol\n4) Liver = increased protein synthesis, including
clotting factors
What drug often given to treat breast cancer in postmenopausal woman
can cause StevenJohnson syndrome? Anastrozole (aromatase inhibitor)
What is a patient at risk for postthyroidectomies if not given
levothyroxine?

Reduced:\n Glycogenolysis\n Gluconeogenesis\n

Lipolysis\n\nB/C thyroid receptors are located on bones in both the CNA &
autonomic nervous systems
What is the only human glycerophospholipid that is antigenic?
Cardiolipin (Diphosphatidylglycerol): it is recognized by antibodies
against Treponema pallidum
What is the most common presentation of Burkitt's in North America?
Loss of appetite\nSever Constipation\nAbdominal pain or
distention\n\nCause of symptoms: enlarged lymph nodes in the abdomen
What catalyses the prodrug prednisone into prednisolone? 11beta
hydroxysteroid dehydrogenase
What is the MOA of imatinib mesylate (Gleevec)?

Tyrosine receptor

kinase inhibior that acts on Philadelphia chromosom fusion protein product


BcrABl in CML. \n Stops autophosphorylation & the unchecked
proliferation of granulocytes in CML.

What do glomeruli of diabetic patients with ESRD (EndStage Renal


Disease) look like histologically? Nodular glomerulosclerosis
What is the most common congenital heart disease in adults? How can it
present?

Patent foramen ovale (PFO)\n Migraine with aura\n Right

toleft cardiac shunts\n Stroke in a young patient (venus thrombi bypass


pulmonary circulation and cause paradoxical emboli)
What is the treatment for a suspected rapies infection?

1) Administer

human rabies immune globulin to provide passive immunity\n2) Followed


with a series of 5 injections of killed rabies virus vaccination to develop
active immunity
What 2 steps make bilirubin soluable? 1) Noncovalent binding to albumin
(in liver)\n2) Conjugation with glucuronate (in hepatocytes via UDP
glucuronyl transferase) to form bilirubin diglucuronide
Where is urobilinogen producted?

Gut: bilirubin diglucuronide is

reduced by gut bacteria


What is the typical presentation for MEN1?

Hyperparathyroidism

(hypercalcemia & hypophosphatemia)


Besides hyperparathyroidism, what 2 signs can be indicative of MEN1? 1)
Pituitary prolacinoma (galactorrhea)\n2) Duodenal ulcers (gastrinoma in
the pancreas causing SollingerEllison syndrome)
What nerve roots is the pudendal nerve derived from?
What is the karyotype of a complete mole?

S24

46 XX (haploid sperm

replicates within an empty egg)


What is the karyotype of a partial mole?

69 XXX (2 haploid sperm

fertilize a haploid egg)


What "essential" amino acids can be synthesized by a healthy adult?
histidine & arginine
What is the embryological source of cranial nerve VII & muscles of facial
expression

2nd branchial arch

Which test for syphilis is the most specific?

FTAABS: dlurescent

treponemal antibody absorption


What are the early cellular changes due to ischemia? late ones? Early:
cellular swelling, organelle swelling, ribosomal disaggregation\nLate:
organelle membrane damage & morphologic changes (cell blebs), cell
surface deformities caused by cyoskeletal dysfunction, & nuclear changes
(pyknosis, karyolysis, & karyorrhexis)
How do renin levels distinguish 1 from 2ndary hyperaldosteronism?
Primary: high renin\nSecondary: low renin
What is the hemodynamic picture for septic shock? Increased CO\n
Decreased SVR (Systemic Vascular Resistance) & PCWP (Pulmonary
Capillary Wedge Pressure)
What is the hemodynamic picture of neurogenic shock?

Loss of

vasomotor tone\n Decreased CO\n Decreased PCWP (Pulmonary


Capillary Wedge Pressure)
What hemodynamic picture indicates hypovolemic shock? Increased
SVR\n Decreased CO\n Decreased PCWP (Pulmonary Capillary Wedge
Pressure)
What hemodynamic picture is seen with cardiac shock?

Increased

SVR\n Decreased Co \n Increased PCWP (Pulmonary Capillary Wedge


Pressure)
What bacteria cause skin abscesses & are most likely transmitted via
dog/cat bite?

Pasteurella multocida (gram rod)

What does infection by Brucella canis cause? fever, malaise,


hepatosplenomegaly
What are the HACEK organisms?
Haemophilus\nActinobacillus\nActinomycetemcomitans\nCardiobacterium
hominis\nEikenella corrodens\nKingella kingae\n\nCan all cause culture
negative endocarditis

What bacterium can be transmitted to humans via handling of rabbit


tissue (or a tick or deerfly bite)? Francisella tularensis\n Welldemarcated
skin lesion with a black base!!\n Later: systemic symptoms (fever,
headache, malaise)
What is the leading theory about how vaccine use has increased the
number of allergies/autoimmune diseases?

Low stimulation of Th1

lymphocytes (b/c they are stimulated by bac/viruses)\n This leads to an


overproduction of Th2 lymphocytes and allergenspecific IgE
What drug is used to treat narcolepsy daytime sleepiness>

Modafinil: increases monoamines in the brain somehow (MOA unknown)\n


Safer than amphetamines & methylphenidate
What nerve innervates the adductor muscles of the thigh? What patients
are at risk for damage?

Obturator (athletes: can be entrapped between

muscles)
What nerve innervates the anterior compartment of the thigh?

femoral

(flexes the thigh & extends the knee)


What nerve innervates the gluteus maximus?Inferior gluteal: extends &
laterally rotates the thigh
What nerve innervates the obturator internus & superior gemellus?
Obturator internus: laterally rotates & flexes the thigh
What muscles does the sciatic nerve (tibial & common peroneal divisions)
innervate?

Hamstrings (posterior thigh muscles): extend the thigh & flex

the leg
How is the kidney changed in hepatorenal syndrome?

It is not. It

remains of normal shape & size. \n There change is only physiological:


vasoconstriction in the vascular beds due to the activation of the renin
angiotensin system.
In what disease do the kidneys appear grossly fleabitten? Malignant
hypertension
What is the MOA of hydralazine?increases cGMP & causes smooth muscle
relaxation (greater effect on arterioles than veins) to reduce afterload\n

Rx: essentail hypertension and CHF\n\nSE: compensatory tachycardia


(angina), Up fluid retention, & lupuslike syndrome with longterm use
What is a rare complication of longterm heparin therapy? What's the
treatment?

HIT: heaprininduced thrombocytopenia\n Heparinspecific

antibodies form complexes with heparin & platelets\n Results in platelet


activation, clot formation, & steadily dropping platelet levels\n\nRx: Direct
thrombin inhibitors (Argatroban & Lepirudin)
What are the direct thrombin inhibitors & why aren't they 1st line therapy?
Argatroban & Lepirudin\n Effects are not readily reversible
What are the physical findings that indicate Icell disease? Coarse facial
features \nDevelopmental delay\nRestricted joint movement\n\nHisto:
intracytoplasmic inclusions in mesenchymalorigin cells.
What is the pathogenesis of Icell disease?

Improper intracellular

trafficing: \n Failure to add a mannose6phosphate residue to proteins


that should be directed to lysosomes\n Result: membranebound
intracytoplasmic vacuoles that are filled with fibrillogranular material
(lipids, mucopolysaccharides, oligosaccharides)
How is congenital rubella infection characterized?
Cataracts\nGlaucoma\nPigmented retinopathy\nCardiac
malformations\nDeafness
What is the MOA of bacterial/viral superantigens?

Proteins that

nonspecifically crosslink MHC class II molecules with certain Tcell


receptor (TCR) subsets \n Inappropriate activation of these Tcells (220%
of all Tcells)\n Supraphysiologic production of cytokins: IL1, TNFalpha,
IL6, IL12, interferongamma.
What is oculocutaneous albinism?

A mutation in tyrosinase (converts

tyrosine into dopa) that is the key, ratelimiting step in melanin production
from phenylalanine
What enzyme is deficient in Maple syrup urine disease?

alpha

ketodehydrogenase: \n inability to degrade branched amino acids


(isoleucine, leucine, valine)\n\nI LoV Maple Syrup

What enzyme deficiency results in galactosemia & galactosuria?


galactokinase: converts galactose into glucose
What disorder is characterized by black urine (especially after long periods
of standing) & arthralgias?

Alkaptonuria (deficiency of homogentisic

acid oxygenase)
What 3 different enzyme deficiencies can result in homocystinuria?

1)

Methionine synthase\n2) Cystathionin synthases (converts homocysteine


into cysteine): REQUIRES B6!!\n3) Decreased affinity of cystathionine
synthase for pyridoxal phosphate\n\nResult: mental retardation,
osteoporosis, tall statue, kyphosis, lens subluxation
What is the presentation of giardiasis? Bloating\n Flatulence\n Foul
smelling, nonbloody, diarrhea\n Lightcolored fatty stools
What infection presents with macrophages containing amastigotes and
clinical symptoms of hepatosplenomegaly, malasie, anemia, & weight
loss? Leishmainia donovani (sandfly transmission)\n\nRx: Sodium
stibogluconate
What 2 important cystic infections are acidfast staining?
Cryptosporidium (water: fecaloral): severe/mild, nonbloody diarrhea
(severe = HIV Px)\n Toxoplasma (raw meat or cat feces): brain abscesses
in HIV Px & birth defects if congenital
What is the treatment for T. gondii?

Sulfadizine & pyrimethamine

What are RBCs containing schizonts characteristic of?

Plasmodium

infection (late stage of the asexual form of the parasite) transmitted by


the female Anopheles mosquito: flulike symptoms
What endogenous hormone inhibits HCl secretion? somatostatin
How do NSAIDS cause peptic ulcer disease? Disrupt the mucus barrier by
inhibiting prostaglandin synthesis
What cells secrete cholecystokinin?

I cells: promotes gallbladder

contraction after ingestion of fatty acids or amino acids

What is the action of VIP (vasoactive intestinal polypeptide)?

Increases

intestinal water & electrolyte secretion


What is the 1st line therapy of ulcerative colitis?

Sulfasalazine:\n

Metabolized to 5aminosalicylic acid in GI tract & decreases local


inflammation\n SE: Renal insufficiency & increased risk of a bleed
When is ciprofloxacin used to treat UC?

When it is complicated by

strictures & infections of the GI (NOT 1st line Rx)


What do the acidophils of the pituitary secrete?

Growth

hormone\nProlactin
What do the basophils of the pituitary secrete?

FSH \nLH\nACTH\nTSH

What are oxyphils? Cells of the parathyroid gland


@ what receptors does ADH act?

V2 (basolateral side of principal

cells) \n activates Gsmediated cyclic AMP \n This mobilizes aquaporin


transmembrane channels to the luminal surface\n Net water moves from
the collecting duct lumen to the hyperosmolar interstitium
What is the course of the ureters in the female pelvis?

Cross iliac

artery anteriorly @ its bifurcation\n Continues into the pelvis, where it


passes posterior to the uterine artery\n"water under the bridge"
What presents in 10% of patients with pancreatic adenocarcinoma?
Trousseau's Syndrome: migratory thrombophlebitis\n\n+ normal
signs of acute weight loss, steatorrhea, & atypical DM (increase thirst &
urination)
What is the blood supply of the liver? 75%: portal vein\n25%: hepatic
artery\n\nBUT, they give equal O2
How does BrownSequard present?

(spinal cord hemisection)\n

ipsilateral loss of motor function & vibration/proprioception\n


contralateral loss of pain & temperature sensation
How does Syringomyelia present?

bilateral sensory defects in the

upper extremities dues to enlargement of the central canal of the spinal


cord

What is the most commonly obstructed artery in the gut & what does it
supply?

SMA (superior mesenteric artery): embryonic midgut

(duodenum to the proximal 2/3 of the transverse colon)


What artery supplies the gallbladder, stomach to proximal duodenum,
pancrease & the liver?

celiac: embryonic foregut

What infection causes undulating fever (peaks in the evening & falls at
nighttime)? Brucella (contaminated milk or direct livestock contact)
Where are Yersinia pestis buboes located?

Buboes (painful, enlarged

lymph nodes) in the groin & armpits. \n then can enter bloodstream and
cause sepsis, DIC, pneumonia or meningitis
How is renal osteodystrophy treated? Ca\n Phosphate binders\n
calcitriol (synthetic vitamin D3) supplimentation
What lung parameter is changed equally in obstructive & restrictive lung
disease?

decrease in vital capacity (insufficient amount of air exhaled

or inhaled)
What are the 2 ways that glucose can enter a cell? 1) Naindependent
facilitated diffusion with glucose transporters\n2) Namonosaccharide
cotransporter system (intestinal cells, renal tubules, choroids plexus)
SGLT1
Where is the GLUT2 receptor? Basal surface of intestinal cells &
transmits glucose into the blood
Where is the GLUT4 receptor? Muscle & adipose tissues
Where is the GLUT5 receptor? Lumen of the gut: Nadependent
absorbtion of fructose
What is the MOA of betablockers that allows them to help with coronary
artery disease?

Slows phase 4 depolarization & prolong's diastole:

increase coronary artery filling\n Decreased sympathetic tone decreases


the inward Na+ current

What characterizes the 2nd phase of Lyme disease? (Borrelia burgdorferi


via Ixodes tick)\n Transient arthralgias\n Bell's palsy\n Atrioventricular
nodal block\n 2ndary annular rashes\n Carditis
What characterizes the 3rd phase of Lyme disease? Migratory joint
pain\nChronic arthritis\nEncephalopathy\nAcrodermatitis
What infection is characterized by relapsing fevers (~5 days of fever with
~ 1 week between episodes)?

Borrelia burgdorferi (spirochete): fever

pattern is due to its ability to undergo antigenic variation


How can lymphoblasts be distinguished from mature lymphocytes?
lymphoblasts have fine, homogenous chromatin, irregular nuclear
boarders, & scant cytoplasm\n Found in ALL (Dx: peripheral blood smear)
What heart murmur indicates a VSD?

IIIII/VI holosystolic murmur \n

Best heard @ the left sternal boarder


What 3 defects can result in a VSD?

1) Septum of the atrioventricular

canal\n2) Muscular septum\n3) Distal conal septum


What are the GRAIN lab results that indicated sarcoidosis? gamma
Globulinemia\nRheumatoid arthritis\nelevated ACEnzyme
levels\nInterstitial fibrosis\nNoncaseating granulomas
Where are MalloryWeiss tears found? gastroesophageal junction: a
nonpenetrating mucosal tear
What is Boerhaave's perforation?

Transmural perforation of the

esophagus\n Presents with Mackler triad: vomiting, lower thoracic pain,


subcutaneous emphysema\n Dx: endoscopy & Rx: surgery
What is the major distinguishing factor of hepatitis E?

Severe, often

fatal disease course in pregnant women: fulminate (20% mortality)\n Hep


E = RNA herpesvirus (nonenveloped, ssRNA)
What type of virus is Hep C?

RNA flavivirus (Enveloped RNA)\n Blood

borne
What type of virus is Hep B?
DNA)

DNA hepadnavirus (enveloped, circular

What type of virus is Hep D?

Defective RNA deltavirus (requires Hep B

to serve as an envelope)
How does FSH induce the conversion of testosterone to 17betaestradiol
in the ovaries?

Stimulates the enzyme aromatase within the graulosa

cells to convert testosterone to 17betaestradiol in the overian follicle.\n


Testosterone gets to the granulosa cells via diffusion from the theca cells
where it is produced
What are the 4 causes of Parkinsonism?

1) Neurodegeneration of the

substantia nigra (idiopathic)\n2) MPTP exposure\n3) Adverse effects of


neuroleptic meds\n4) Ischemic damage to the basal ganglia
What causes hemiballism of the limbs?lesion of the contralateral
subthalamic nucleus cause uncontrollable flailing
Describe the (INHIBITORY) indirect pathway in the brain.

1) Substantia

nigra pars compacta sense INHIBITORY neurons to the striatum.\n2)


Striatum sense INHIBITORY output to the external segment of the globus
pallidus; this inhibits the subthalamic nucleus\n3) Subthalamic nucleus
sends EXCITATORY neurons to the internal segment of the globus pallidus:
this increases firing of GABA neurons that run from the GPi to the
thalamus\n4) Movement on the contralateral side of the body is inhibited
What brain lesion would cause a spastic paralysis of the arm & leg on one
side of the body?

On the CONTRALATERAL side of the brain either:\n1)

primary motor cortex\n2) medullary pyramid\n3) lateral white matter tract


of the spinal cord above C5
What does paralysis in general indicate?

A problem in the motor

neurons themselves (i.e. not a ganglion)

Describe the pathway of an upper motor neuron.

1) Origination: primary

motor cortex\n2) Pass through posterior limb of the internal capsule,


midbrain cerebral peduncles, ventral pons, & medullary pyramids\n3)
Majority decussate @ the caudal medulla\n4) Run in the lateral white
matter tracts in the spinal cord\n5) Synapse on the lower motor neuron @
he ventral horn, in the gray matter

What is the ABVD regimen?

Adriamycin (aka doxorubicin: DNA

intercalator; cardiotoxic)\n Bleomycin (DNA strand breaker )\n


Vinblastine\n Dacarbazine\n\nRx: Hodgkin's lymphoma
What change in gram + structure results in vancomycin resistance?

ala Dala to Dala Dlac aminoacid change for the cell wall inhibitor that
vancomycin targets
What does a right or left shift in the O2 dissociation curve indicate?
Right: decreased affinity for O2 (lower % of bound O2 @ a given
partial pressure)\n\nLeft: increased affininity for O2 (greater % of bound
O2 @ a given partial pressure)
What drug is used both as an antitumor agent and for sickle cell?
Hydroxyurea: antitumor (Rx: CML) & also raised fetal hemoglobin
levels
What is 2.3BPG's relationship with O2 affinity?

Inverse: high levels

decreases O2 affinity
What are the classic symptoms of PraderWilli?

Mental retardation\n

Short stature\n Hypotonia\n Hyperphagia\n Obesity\n Small hands &


feet\n Hypogonadism\n\nDeletion of q12 on paternally derived
chromosome 15
What are the classical symptoms of AngleMan's syndrome?

Mental

Retardation\n Seizures\n Ataxia\n Inappropriate Laughter\n\n"Happy


Puppets"\n Deletion of q12 on the maternally derived chromosome 15
What characterizes Paget's disease of bone & what lab values go along
with it?

Extensive remodeling of bone that drastically incrases bone

mass (yet, with increased fracture risk).\n Up risk for sarcomas\n LAB:
elevated/normal Ca, normal phosphate, elevated alk phosphate
How does class V "lupus" nephritis present histologically? Membranous
nephropathy: diffuse thickening of the glomerular basement membrane
due to immune complex deposition\n Nephrotic: edema, proteinuria,
hyperlipidemia\n EM: sup epithelial deposits (spike & dome"\n Immuno:
granular

What type of nephritis results in eosinophilia of the kidney interstitium?


druginduced interstitial nephritis
How does class 1 lupus nephritis present on light microscopy? clinically no
apparent pathology on LM; nephrotic syndrome
What disease presents with proliferation of the mesangial cells in the
glomerulus? membranoproliferative glomerulonephritis or (slight
proliferation) in class II lupus nephritis
What kidney disease is characterized by subepithelial humps on EM?
subepithelial humps
What are smudge cells and what do they indicate? Fragile lymphocytes
(arrested between between preB & mature B cells) that are destroyed
during slide prep\n CLL: chronic lymphocytic leukemia
What are the kidneytargeted drugs used to treat gout? What is their
MOA? Probenecid & sulfinpyrazone: compete with uric acid for
reaborbtion in the kidney
What is the MOA of Allopurinol? Blocks xanthine oxidase & prevents
formation of uric acid from purines\n Rx: chronic gout or tumor lysis
syndrome
What OTC drug is contraindicated in gout patients? ASPIRIN\n b/c, at low
doses it competes with uric acid for excretion
What drug is used to treat acute gout? Colchine: depolymerizes microtules
to limit the inflammatory response to urate crystals
What is the 1st line DOC for acute gout?

Indomethacin (NSAID):

blocks prostaglandin synthesis


What diuretic is counter indicated in gout patients? Furosemide
(sulfonamide loop) b/c it raises serum uric acid levels\n Inhibits NaK2Cl
cotransporter to increase Na, K, & Cl excretion\n Ototoxic
What other cancers are kids with retinoblastoma at risk for?
Osteosarcomas in their teens:\n Rb gene mutation on ch. 13: no
E2F regulation and cells easily move into S phase

What causes metabolic acidosis?

Increase in acid:\n Ion gap\n

Hypovolemic shock via lactic acid\nDecrease in base:\n Diarrhea\n


Kidney failures
What is non ion gap vs. ion gap acidosis?

Ion gap acidosis: caused by

an increase of anions (such as lacate)\n\nNon ion gap: caused by a loss of


bicarb with an increase in Cl\n\nAnion Gap = [Na+] [Cl]
[HCO3]\nNormal anion Gap = 1016 mEq/L
Unpack the mneumoic "MUDPILES" in regards to anion gap metabolic
acidosis?

Methanol\nUremia\nDiabetic/ethanol

ketocaidosis\nParaldehyde\nIsoniazid/Iron toxicity\nLactic
acidosis\nEthylene glycol\nSalicylates\n\n(& Rhabdomyolysis)
What are the 2 causes of respiratory acidosis?

1) Inadequate

ventilation (drugs or neurologic injury)\n2) Impaired gas exchange (e.g.


pulmonary edema)
What is metabolic alkalosis?

High pH + increased plasma

bicarbonate\n\nCause: addition of alkaline compounds (antacid) or loss of


acid (vomiting)
What causes respiratory alkalosis?

Increase in ventialtion leading to a

decrease in serum CO2 & excess bicarb. Causes:\n Increased drive (drugs
or CNS disorders)\n Anxiety/Fear
What are fibrates used to treat? hypertriglyceridemia\n\nE.G. Genfibrozil
What does Niacin do to cholesterol levels?

@ high doses:\n Raises

HDL\n Reduces VLDL


What is the MOA of phytosterols?

Inhibit the incorporation of

cholesterol into micelles in the GI tract \n Decreases the overall amount


of cholesterol absorbed.
What is the chromosomal source of von HippelLindau disease? Defect in
the VHL gene on Chromosome 3
Where is the Rb gene?

Ch. 13

Where is the NF1 gene? Ch. 17

Where is the NF2 gene? Ch. 22: merlin tumor suppressor gene is
mutated
Where is the APC gene?

Ch. 5

What can hypersecretion of ectopic thyroid tissue cause? menstrual


abnormalities: amenorrhea & oligomenorrhea\n\nLocation of (congenital)
ectopic thyroid tissue: anywhere along the course of the throglossal duct
beginning beneath the the tongue @ the foramen cecum
How does Addison's result in diffuse integumentary hyperpigmentation?
NO ACTH secretion feedback inhibition from cortisol\n T/F the
POMC gene (which codes for ACTH & melanocytestimulating hormone) is
upregulated
What is the DOC for Bblocker overdose?

Glugagon: \n positive

inotropic agent: increases intracellular cAMP independently of adrenergic


receptor signaling (thus, it bypasses any adrenergic blockage)
Unpack the mneumonic "BFLAT Major" for the cells of the pituitary gland
cells. Basophilic:\nFSH\nLH\nACTH\nTSH\nMSH
What is the MOA of exemestane?

Inhibits aromatase: no conversion

of testosterone to estradiol & androstenedione to estrone\n forms an


irreversible bon with aromatase
What is the difference between the site of estrogen production in pre &
postmenopausal women? Pre: produced in ovaries\nPost: adrenal gland via
aromatase
What 2 drugs inhibit aromatase via reversible competition?
anastrozole & letrozole
What is the MOA of finasteride when used to treat BPH, prostate cancer, &
male pattern baldness?

inhibits 5alpha reductase (in prostate,

epididymis, seminal vesicles, & skin) from converting testosterone to


dihydrotestosterone\n Can increase levels of estradiol\n Dutasteride has
the same MOA
How do GnRH agnonists like Leuprolide act to reduce FSH & LH release in
hormoneresponsive prostate or breast cancer?

B/C the drug is not

giving the physiological GnRH pulses, FSH & LH are downregulated after
an initial burst.
What cell population defect causes Hirschsprung's disease?
craniocaudal neural crest cell migration failure to distal colon: no
parasympathetic ganglion cells & no coordinated peristalsis (functional
obstruction of the colon)\n\nSymptoms: constipation, distended abdomen,
bilious vomiting\n\nRx: resecetion of the aganglionic segment of the colon
What types of drug can cause hyperprolactinemia? Drugs that suppress
dopamine's effects (& its inhibitory effect on prolactin): e.g. Haloperidol, a
typical antipsychotic
What face nervous modalities pass through the VPN of the thalamus?

All sensory: light touch, proprioception, vibration, pain & temperature\n


Blood to this structure is supplied by the posterior cerebral artery
What information does the lateral geniculate nucleus of the thalamus
receive?

optic tract

What information does the medial geniculate nucleus of the thalamus


receive?

auditory info from the inferior colliculi

Through what nucleus of the thalamus do all sensory modalities of the


body (including dorsal column & spinothalamic tract) pass through?
Ventral posterolateral nucleus
What result on Hektoen enteric agar will show that the infection is
Salmonella and not Shigella?

Green with black centers = a colony of

Salmonella (produces hydrogen sulfate) & are flagellated\n Both will


appear clear or white on MacConkey agar
What are the symptoms & history that indicate an infection iwth
Entamoeba histolytica?

Bloody mucoid diarrhea\n Hepatic abscess (up

alk phos with normal AST, ALT & bilirubin)\n Recent travel to a developing
country\n\nRx: metronidazole
For what infection would triple therapy with Ampicillin, gentamicin, &
clindamycin be prescribed?

Gramnegative GI infection

Why do injection drug users have a higher risk for hepatocellular


carcinoma (HCC)? Concomitant increased risk for Hep B & C, which are
the direct cancer risk factors
What is deficient in Pompe's?

Lysosomal alpha1,4glucosidase:

necessary for the hydrolysis of the outer branches of glycogen\n glycogen


is deposited in the myocardium & thus, "Pompe's trashes the Pump" by
the 6th month of life
What defect results in Fabry's?

lysosomal alphagalactosidase A: T/F

ceramide trihexoside accumulates, resulting in:\n peripheral neuropathies


of the hands & feet\n angiokeratomas, heart, & renal disease\n\n X
linked recessive
What is defective in Gaucher's

Lysosomal betaglucocerebrosidase:\n

Hepatosplenomegaly\n Aseptic necrosis of the femus, bone crises\n


Gaucher's cells
What is the defect in VonGierke's?

glucose6phosphate deficiency:\n

severe fasting hypoglycemia with large stores of glycogen in the liver\n


hepatomegaly\n increased blood lactate levels
What deficiency results in McArdle's?

gycogen phosphorylase: glycogen

in skeletal muscle cannot be broken down\n cramping \n myoglobinuria


with strenuous exercise
What is the MOA of trimethoprimsulfamethoxazole?

blocks the

synthesis of folate:\n Trimethoprim inhibits bacterial dkhydrofolate


reductatase\n Sulfamethoxazole inhibits dihydropteroate synthase
What is the #1 & #2 cause of UTIs in young women?

1) E. coli\n2)

Staph saprophyticus
What are the symptoms of HSV1 encephalitis?

Amnesia \n

Behavior changes (e.g. hypomania)\n KluverBucy syndrome (loss of


anger/fear response + hypersexuality\n Late: coma & death
How do patients with HIV meningoencephalitis present?
of slowly worsening dementia

Gradual onset

How does progressive multifocal leukoencephalopathy present? Cause:


JC virus\n Altered mental status, motor deficits (hemiparesis or limb
ataxia)\n Visual symptoms: e.g. hemianopia
What is characteristic of dermatomyositis & what does it indicate?

Immunemediated disorder of skin & skeletal muscles: rash is violet


discoloration of the upper eyelids with periorbital edema; gradual,
symmetric muscle weakness of proximal muscles, sometimes
dysphagia\n Gottron's lesions: erythematous patches over knuckles,
elbows, & knees\n\n 645% of patients have underlying visceral cancer
What is Zenker's diverticulum? Pharyngeoesophageal diverticulum:\n
Outpouching of the esophageal wall above the level of the upper
esophageal sphincter\n Pathogenesis: herniation of mucosa though
defective muscular layer
What will a physician find upon physical exam of a spontaneous
pneumothorax?

Lung sound: decreased sound\n Percussion:

Hyperresonance b/c the space is filled with air instead of tissue.\n Tactile
fremitus: absent\n Tracheal deviation: toward side of lesion (opposite for
tension pneumothorax)
What is the MOA of Norwalk virus infection? Direct, noninvasive toxin
mediated damage of the microvilli of enterocytes\n Disease course:
gasteroenteritis (nausea, vomiting, diarrhead) that resolves within 1224
hrs.
What is the MOA of Shigella toxin?

Induces cytokine release & causes

hemolytic uremic syndrome:\nbloody & mucusrich diarrhea


What is lacking in Krabbe's disease & how does it present?
galactosylceramide Bgalactosidase deficiency:
galactocerebroside \n optic atrophy\n spasticity\n Early death via
deposition in intermediary of the brain
What is deficient in NiemannPick disease & how does it progress?
Sphingomyelinase: sphingomyelin & cholesterol buildup in
reticuloendothelial & parenchymal cells/tissue\n Patients die by age 3

Which 5 kidney diseases are nephrItic (hematuria, azotemia, RBC casts in


urine, oliguria, hypertension, proteinuria)?

Acute proststrep\n Rapidly

progressive (crescentic)\n Diffuse proliferative (due to SLE)\n Berger's


(IgA glomerulopathy)\n Alport's

What type of cells are found in the liver's space of Disse? Hepatic stellate
cells (Ito/lipocytes):\n filled with lipid droplets containing vitamin A\n
produce collagen in response to liver injury
What is the ingested form of vit. D called?

ergosterol

What type of murmur is heard with a classic case of acute bacterial


endocarditis?

Diastolic: dues to large vegetations on the ricuspid

valve that impede flow through the valve


What is the difference between the action of Ca in smooth & skeletal
muscle?

1) Smooth: Ca signals through myosin light chain kinase \n

Ca binds/activates calmodulin that activates MLCK \n MLCK


phosphorylates myosin to allow crossbridges\n2) Skeletal: Ca binds
troponin to allow actinmyosin interaction
How does skeletal muscle respond to depolarization?

Dihydropyridine receptor directly allows extracellular Ca influx & activates


the ryanodine receptor\n Ryanodine receptor mediates release of Ca from
intracellular stores in the sarcoplasmic reticulum
What is the MOA of Class IA antiarrhythmics? Rx: atrial or ventricular
arrhythmia\nMOA:\n block Na channels and slow conduction velocity >
this slows phase 0 & t/f increases QRS duration\n block potassium
channels > increases AP duration & ERP (effective refractory period
What results from combining an ACE inhibitor with a potassiumsparing
diuretic?

A possible dangerous level of hyperkalemia:\n ACE reduces

angiotension II production and aldosterone is reduced (increases Na


excretion & potassium retention)
What GI side effects do typical antipsychotics like Haloperidol causes?
antimuscarinic: constipation

What Bp effects can typcial antipsychotics have?

hypotension (anti

alpha receptor action)


What histaminerelated effects can typical antipsychotics have? anti:
sedation
What causes the extrapyramidal adverse effects of antipsychotics?

An

imbalance in dopamin & muscarinic receptor antagonism:\n Dystonia


(abnormal postures due to sustained & prolonged contraction of
agonist/antagonist muscles)\n Akinesia: absence of movement\n
Akathisia: restlessness that is relieved by movement\n Tardive dyskinesia:
involuntary choreiform movements of the lower face (e.g. rhythmic
protrusion of the tongue, lip smaking, chewing)
What musculoskeletal problem can be due to lithium use? Ataxia:
incoordination
What culture is used to grow Corynebacterium diptheriae? Loeffler's
medium, blood agar, tellurite plate
What type of goiter do Grave's patients have?

Diffuse on

palpation\n High radionuclide uptake on thyroid scan


What type of goiter does a patient with a TSHsecreting pituitary tumor
have? Diffuse on palpation\n Low radionuclide uptake on thyroid scan
What type of goiter do patients with toxic hyperthyroidism present with?
Multinodular on palpation\n High radionuclide uptake on thyroid
scan
How does a goiter on a patient with primary hyperthyroidism due to
destructive throiditis, iodine excess or excess thyroid hormone appear?
Nodular on palpation\n Low radionuclide uptake
How does Bell's palsy present clinically?

Inability to close the eye or

seal the corner of the mouth on the affected (ipsilateral) side


How does stage 2 Lyme disease present?

Bell's palsy (LMN lesion of

facial nerve)\n New Heart block


What drugs cause ototoxicity?

aminoglycosides & vancomycin

What drugs are notorious for their nephrotoxicity?


Aminoglycosides\nVancomycin
What drug can result in PED teeth discoloration?

doxycycline

What drug class can cause acute cholestatic hepatitis?

macrolides

What is the 1st treatment for acute ICP (increased intracranial pressure)?
1) Intubate\n2) Hyperventilate: CO2 is a powerful vasodilator of the
cerebral vessels; thus, "blow it off" to cause vasoconstriction & decrease
intracrainal blood
Why is a lumbar puncture dangerous in patients with an increased ICP? ICP
drops too much and this can depress the entire ventricular system and
allow for further herniation of the brain.
The patient is a smoker and Xray revelas a large hilar mass with signs of
cavitation. What's the Dx?

Squamous cell carcinoma: also can

present with paraneoplastic production of parathyroid hormonerelated


peptide (PTHrP) that causes HYPERcalcemia (> urolithiasis)
What is tumor lysis syndrome? Uric acid stone formation due to excessive
cell turnover after chemo (esp. with lymphoma)
What paraneoplastic syndromes are associated with renal cell carcinoma?
Parathyroid hormonerelated peptide\n Erythropoietin
What paraneoplastic syndrome is associated with small cell lung
carcinoma? ADH (SIADH)\n ACTH (Cushinglike)
What infections can cause urolithiasis with ammonium magnesium
phosphate (aka struvite) stones?

Ureaseproducing orgs:\n Proteus

vulgaris\n Staphylococcus\n Klebsiella


What is the MOA of Tacrolimus & for what is it usually prescribed?

1)

Immunosuppressive: transplant recepients\n2) MOA:\n Binds FKbinding


protein & this complex inhibits the secretion of cytokines that activate
lymphocytes (e.g. IL2)\n SE: nephrotox, CNS effects, hyperglycemia
To which class of drug do busulfan, nitrosoureas, & cyclophosphamide
belong?

Alkylating agents that bind to & disrupt DNA = antineoplastic

What drug classes inhibit microtubules?

Vinca Alkaloids (Vincristine

& vinblastine)\n Rx: cancer (testicular & lymphomas)


What type of virus is Hep A?

Picornavirus: Linear ssRNA

What embryologic abnormality results in cleft lip?

Failure of the maxillary

processes or the medial nasal processes to fuse


What embyrologic abnormality results in cleft palate?

Failure of the

fusion of the lateral palatine processes: difficult to create suction needed


for proper feeding\n Result: choking/coughing/aspiration + poor weight
gain
What lab values indicate an intrarenal type of renal failure?

Decline

in GFR\n BUN & Cr increase in proportion (RATIO REMAINS BELOW


15:1)\n Faction of excreted Na >2% b/c the kidney is not effectively
reaborbing Na = LOW URINE OUTPUT
What lab values indicate postrenal causes of renal failure (e.g. prostatic
disease, urethral stones)? BUN:Cr ration of 1520:1\n Low urine output
What lab values indicate a prerenal cause of renal failure? BUN:Cr ration
of >20:1\n Fractional excretion of Na <1%\n LOW URINE
OUTPUT\n\nCause: decreased blood flow to the kidneys > Na
conservation, increase in urea reaborbtion increases BUN
What vitamin is necessary for NADPH production?

B3 (niacin): used to

make fatty acids & steroids & respiratory burst in WBCs (recycling of
glutathione in RBCs)\n Glucose 6phosphate dehydrogenase requires B3
& it is the ratelimiting step in the O2 portion of the pentose phosphate
pathway
What is FAD used for?

It is an oxidizing agent used to make double

bonds:\ne.g. production of fumarate from succinate in the TCA cycle


Which vitamin is FADH2 derived from? B2 (riboflavin): participates in the
electron transport chain\n FADH2 is generated when succinate is oxidized
to fumarate

What does increased NADH result in? Pyruvate conversion to lactate\n


Stimulates fatty acid synthesis\n Inhibits gluconeogensis\n\n NB: this is
part of the pathophysiology of ALCOHOLISM
How is transferrin saturation calculated?

Serum iron / TIBC\n\nTIBC =

1.4 x serum transferrin


What genetic defect results in hereditary hemochromatosis?

Defect

in the gene that encodes the protein HFE (MHC locus on chromosome
6)\n Result: increased dietary iron absoption & inability to excrete iron >
hepatic storage & liver makes more ferritin, which in turn saturates serum
transferrin\n Dx: increase in transferrin saturation
What antiviral is also prescribed for Parkinson's? What is its MOA?
Amantadine (Rx: influenza & rubella):\n Dopamine agonist:
Promotes the synthesis, release, or reuptake of dopamine\n SE: ataxia,
dizziness, & slurred speech\n Overdose: TOXIC PSYCHOSIS
What antimuscarinics are used to treat Parkinsonian tremor?

Benztropine\n Biperiden\n Orphenadrine\n Procyclidine\n


Trihexyphenidyl
List the SSRIs.

Citalopram\n Fluoxetine\n Fluoxamine\n

Paroxetine\n Sertraline
What monoamine oxidaseB drug is used to increase the availability of
dopamine in Parkinson's treatment?

Selegiline

What is the MOA of bile acid resins like Cholestyramine & Colestipol?

Promote binding & excretion of dietary fats that are bilesoluable\n T/F
the fats don't enter the blood stream effectively\n Decrease serum LDL &
total cholesterol levels
What type of structures result when a fertilized oocyte divides between
days 4 & 8? Monozygotic twins\n Monochorinoic placenta\n
Diamniontic
What type of structures result when a fertilized oocyte divides between
days 8 & 12?1 of everything:\n Monozygotic twins\n Monoamniotic\n
Monochorinoic placenta

What type of structures result when a fertilized oocyte divides within 3


days postfertiziation?

2 of everything:\n Mono or dizygotic twins\n

Diamniotic \n Dichorionic\n 2 placentas


What does an elevated AFP in amniotic fluid possibly indicate?

Neural

tube defects (spina bifida, meningocele, menigomyelocele) & AFP is


leaking out into the amniotic fluid\n Folate Deficiency in 1st 4 weeks of
pregnancy or drugs such as valproate & carbamazepine
What is another name for Potter's syndrome? Bilateral renal agenesis:\n
disruption in the interaction between the ureteric bud & the
metanephrogenic tissue\n fetus does not produce urine & t/f there is a
lower amount of amniotic fluid (oligohydramnios) > \n Hypoplasia\n
Fetal compression with altered facies\n Positioning defects of hands &
feet
What is the ductus arteriosus & what is its function? Connection between
the pulmonary artery & the aorta \n Allows O2 blood from the placenta to
bypass the fetal lungs & enter the systemic circulation\n Open during
gestation & closes with O2 increase @ birth & subsequent decrease in
prostaglandins
What is the MOA of Phenytoin? Blocks Na channels & inhibits the
generation of action potentials \n\nRx: partial & generalized seizures\nSE:
nystagmus, diplopia, lethargy, ataxia, tubulointerstitial nephritis (Up Cr),
Up tox with anything that inhibits microsomal enzymes in the liver (e.g.
cimetidine)
What defective transporter is missing in Hartnup's disease?

for

Neutral amino acids (all except proline) @ the renal & GI tracts\n Result:
aminoacidurias\n can manifest as Pellagra b/c tryptophan is used to make
niacin with B6 as a cofactor
Where is the potential space in the pericardium?

Between the visceral

& parietal layers


How does Kartagener's result in female infertility?

Loss of fimbrae &

tubal cilia "sweeping" leads to immobile eggs that do not advance through
the fallopian tubes @ normal rates\n microtubule dysfunction

What enzyme defect results in albinism?

Melanocyte lack of

tyrosinase: no conversion of tyrosine to melanin\n Increased risk of


squamous, basal cell, & malignant melanoma
What cells/tissues are derived from neural crest cells (part of the
neuroectoderm)?

Melanocytes\n Odontoblasts\n Pia & arachnoid

mater\n Schwann cells\n Cells of the gangila\n Parafollicular C cells of


the thyroid\n Chromaffin cells\n Aorticopulmonary septum\n Pharyngeal
arch skeletal components\n Neurocranium
What conversion do statins inhibit?

HMG CoA to mevalonic acid\n\nSide

effect: muscle pain or injury similar to myositis (MOI unknown)


Where is the horizontal fissure of the right lung on the anterior chest?
fourth rib anteriorly: separates superior from middle lobe
What does the oblique fissure of the right lung separate? Middle from
inferior lobe
What are the symptoms of vitamin E deficiency?

Absent tendon

reflexes\n Ataxia\n Loss of position & vibration sense\n Loss of pain


sensations
What are the symptoms of vitamin B2 (riboflavin) deficiency?

Cheilosis\n Glossitis\n Corneal vascularization\n Seborrheic dermatitis


Where are watersoluable vitamins absorbed?
What are Brenner tumors?

small intestine

Benign ovarian tumors composed of

cells that resemble bladder transitional epithelium\n Adenofibroma with


epithelial component of nests of transitional cells
In what tumor are CallExner bodies seen?

Graunulosa cell tumors (sex

cord stromal tumor): ovarian follicles filled with eosinophilic material\n


secrete estrogen > precocious puberty, endometrial hyperplasai,
endometrial carcinoma
What is Meigs' syndrome?TRIAD:\n overian fibromas\n ascities\n
hydrothorax

What is struma ovarii?

Ectopic thyroid tissue in the ovaries: \n a

benign functional cystic teratoma\n Result: throtoxicosis &/or


hyperthyroidism
How does primary sclerosing cholangitis present?

Progressive

fatigue\n Pruritis\n Icteric Sclera


How is primary sclerosing cholangitis characterized?

A disease

associated with Ulcerative Colitis resulting in:\n Inflammation\n


Obliterative fibrosis\n Segmental constriction of intrahepatic &
extrahepatic bile ducts (visualized as alternating strictures & dilations, or
"beading"\n\n Dx: PANCA (majority)\n Rx: liver transplant\n
Proctocolectomy to eliminate the GI problems with UC & risk of colon
cancer
What is an ERCP "doubleduct" sign?

Pancreatic tumor obstruction of

both the common bile duct & main pancreatic duct \n Seen on a
radiographic visualization of the pancreatic duct & bilary tree
What is the pathology of primary biliary cirrhosis?

Granulomatous

destruction of the mediumsized intrahepatic bile ducts


How does HPV cause carcinoma?

inactivates tummor suppressor

genes such as p53 (viral protein E6) & Rb (viral protein E7)
What 2 types of cancer is EBV associated with?

t(8:14) = Burkett's

(bcell) lymphoma\n Nasopharyngeal carcinoma


What cancer does the Human Tcell lymphotropic virus cause?

Adult T

cell leukemia
What is the treatment for Hep B?

lamivudine (reverse transcriptase

inhibitor to inhibit viral repication)\n interferon alpha (blocks viral


replication)
What is the treatment for Hep C?

Ribavirin + pegylated interferon

alpha
What triptan is a DOC for the acute treatment of migraine?
Sumatriptan (serotonin agonist @ the 5HT1d recetpr): reduces
migraine severity

What type of drugs are used as prophylactic against migraines? TCAs


(e.g. Amitriptyline)\n Ergot Alkalodi derivative (e.g. Methysergide)\n B
blocker (e.g. Nadolol or propanolol)
What is the MOA of diazepam?

Increases the opening frequency of the

Cl channel assocated with the yaminobuteric acid (GABA) receptor: this


inhibits further neuronal firing\n Immediate onset of action when given IV
What is the MOA of Carbamazepine?

Blocks repetitive activation of Na

channels
What is the MOA of ethosuximide?

Blocks lowthreshold Ttype Ca

channels\n Rx: absence seizures


What is the MOA of Gabapentin?It is a GABA analog & blocks Hcurrent
modulators
What is the MOA of Phenobarbital?

Increases the duration of Cl channel

opening
What lab values indicate ALL in a child 37 years old?

Greatly

elevated ABC count (>20% blasts)\n Neutropenia


What is a serum marker of cell lysis?

Elevated uric acid level (from

purine breakdown)
What clotting/bleeding problem can be indicated by petechiae?
insufficient platelets
What is "rose gardener's disease"?

Sporothrix schenckii fungal

infection:\n necrotizing granulomatous lymphocutaneous skin infection\n


S/S: local pustule with nodules in the distribution of draining lymph nodes
What is a common kidney complication of DIC & what is the Tx? Diffuse
cortical necrosis due to generalized infarction of the cortices of both
kidneys\n Dialysis when the patient presents with AEIOU:\nA = Acidosis
refractory to bicarb\nE = severe Electrolyte abnormalities (esp. K)
refractory to meds\nI = intoxication with some drugs\nO = volume
Overload refractory to diuretics\nU = Uremic symptoms (e.g. cardiac
friction rub & altered mental status)

What personality disorder presents with violent swings in affect, recurrent


suicidal ideation, impulsivity, selfmutilation, & strained interpersonal
relationships?

Boarderline

What personality disorder presents with a complete disregard for the


rights of others & criminal behavior?

Antisocial

What personality disorder presents with clingy behavior & constantly need
to be taken care of?

Dependent

What personality disorder presents with the inherent belief that the world
is a dangerous & threatening place, distrust everyone, & believe various
conspiracy theories?

Paranoid

What personality disorder is characterized by interpersonal awkwardness,


a tendency not to relate well to others, & an inability to understand why
others don't find the same things interesting as they do? (also, they tend
to have odd beliefs & eccentric appearance) Schizotypal
What is the difference between Wernicke's encephalopathy & Korsakoff's
pscyhosis

BOTH: B1 deficiency\nWernicke's is reversible:\n ataxia\n

confusion\n nystagmus\nKorsakoff's is permanent (lesion in the mamillary


bodies):\n anterograde amnesia\n confabulation\n personality changes
What results from bilateral lesions of the amygdala?

KluverBucy

syndrome:\n hyperorality\n hypersexuality\n disinhibition


What happens when there is a lesion in the inferior frontal gyrus (Broca's
area)? motor/nonfluent/expressive aphasia:\n Can understand\n Cannot
produce ("no boca")
What happens when there is a lesion in the superior temporal gyrus
(Wernicke's area)? sensory/fluent/receptive aphasia:\n can speak
fluently\n cannot understand others or what they say themselves
What is the most common cause of neonatal sepsis & meningitis Group B
Strep (GBS) \n prophylaxis to mom = Penicillin G prebirth
What types of food can precipitate a gouty attack? Foods rich in purine:
meats (organ meats, e.g. kidneys)\n\nRx: \n NSAIDS (ibuprofen,
naproxen, indomethaicn)\n Colchicine if NSAIDallergic: SE = diarrhea, GI

upset... (inhibits microtubule polyermization by binding to tubulin >


inhibits immune cell motility/activity)
What is the treatment for gonococcal arthritis?

Ceftriaxone

What is a common side effect of an overose of corticosteroids?

Iatrogenic

Cushing's syndrome (moon facies & buffalo hump)


What are the clinical hallmarks of NF1 (aka von Recklinghausen's disease)?
AD defect on long arm of Ch. 17: 1/4000 births\n Cafe au lait
spots\n Neurofibromas\n Lisch nodules (pigmented iris
hamartomas)\n\nNB: NF1 is a risk factor for menigiomas
What syndrome presents with Brushfield's spots?

Down's: white spots in

the periphery of the iris


How can protaglandins be used to induce labor?

In general, they

activate dissolution of collagen bundles & increase the submucosal H2O


content of the cervix\n PGE = potentiates endogenous oxytocin
(increases labor induction)
What causes Kartagener's syndrome? Lack of dynein arms in the
microtubles in cilia = immotile\n immotile sperm\n defective mucociliar
ladder \n situs invertus (reversed/mirrored major organs)
Why are CF males infertile?

Cystic fibrosis causes a bilateral absence

of the vas deferens


What malespecific problem can arise due to familial
hypercholesterolemia?

Atherosclerosis of the vessels of the male

genitalia can cause erectile dysfunction


What are brown granular deposits in the cornea?

KayserFleischer rings:

copper deposits
What is the ABCD's of Wilson's

Asterixis\n Basal ganglia degeneration

(parkinsonian symptoms)\n Decreased Ceruloplasmin level\n Cirrhosis,


Corenal deposits in Descernet's membrane, Copper accumulation,
hepatocellular Carcinoma, Choreaform movements\n Dementia

What is the treatment for Wilson's?

Dpenicillamine: promotes urinary

copper excretion\n\n2nd choice: copper chelators (e.g.


Deferoxamine)\n\nPlasmapheresis if Px has fulminant hepatic failure
What is Dimercaprol used to treat?

Chelator of arsenic, mercury, gold,

or lead
What is Succimer used to treat? Chelator of arsenic, mercury, gold, or lead
What is EDTA used to treat

It is a Ca chelator used to treat LEAD

poisoning
What causes replacement of throid & surrounding tissue with fibrous
tissue?

Riedel's thyroiditis\n presentation: dysphagia, stridor,

dyspnea, hypothroidism or euthyroid\n DDx: thyroid carcinoma


What is the etiology of renal insufficiency in Multiple myeloma patients?
Bence Jones proteins (antiboy light chains) result in renal damage
by forming large, tubular casts that obstruct the renal tubular lumina &
induce inflammation.
What is Rheumatoid factor?

IgM antibody directed against the Fc

portion of IgG
How does renal cell carcinoma or prostate carcinoma damage the kidneys?
obstructive damage by a neoplastic mass
What is Strep pyogen's major virulence factor?

Protein M: inhibits

complement system & protects from phagocytosis


How does a cerebrovascular event of the vertebral artery present?
Vertebrobasilar system: ipsilateral cranial nerve defects &
contralateral motor weakness\n diplopia\n dysarthria\n vertigo\n
ataxia
How does a cerebrovascular event of the anterior cerebral artery present?
contralateral motor weakness, greater in leg than in arm, & mild
sensory deficits

How does a cerebrovascular event of the anterior choroidal artery


present?

contralateral homonymous hemianopsia\n hemiparesis\n

hemisensory loss
How does a lacunar ischemia present? If longterm: dementia\n\nIf acute:
(basal ganglia & pons)\n pure motor deficits (face, arm, & leg
paralysis)\n sensory deficits (sensory loss in face, arm, & leg)
How does a cerebrovascular event of the middle cerebral artery (MCA)
present?

Contralateral motor & sensory deficits that are greater in the

face & arms than the leg


What 2 signals are needed to activate Tlymphocytes?

Provided by

APCs (macs, dendritic cells, B lymphocytes):\n1) Major histocompatibility


complex/peptide\n2) CD28B7 interaction
What cells are the main effectors that mediated acute allograft rejection?
CD8+ T lymphocytes
Name 3 antiVEGF (vascular endothelial growth factor) antibodies that are
used to inhibit tumor angiogenesis.

1) Bevacizumab\n2) Sorafenib\n3)

Sunitinib
What 4 glycoprotein hormones share the same alpha subunit?

beta

hCG (fertilized egg during 1st trimester & then placenta)\n TSH (anterior
pituitary)\n LH (anterior pituitary)\n FSH (anterior pituitary)
What results from taking an MAO with a tyramine food (aged cheese, liver,
red wine)?

the tyramine will not be metabolized and, as a vasoactive

compound, can cause a drastic increase in Bp & hypertensive crisis &


"serotonin syndrome"\n\n MAO example: Phenelzine
What antidepressance can cause the 3 "C"s (Coma, Convulsions,
Cardiotoxicity)?

Amitriptyline (TCA)\n can also cause sedation,

confusion, memory loss, & mania


What symptoms/signs characterize serotonin syndrome?

Anxiety\n

Diaphoresis\n Tachycardia\n Hypertension\n Hyperthermia\n


Vomiting\n Tremor\n Hyperreflexia\n Muscle rigidity

What type of drug is Venlafaxine?

Heterocyclic antidepressant\n\nSE:

anxiety, agitation, headache, weight loss, insomina


What drug causes hallucination, delusions, dilated pupils, and potentially
marked anxiety or depression, nausea, weakness, & parethesias?
LSD: lysergic acid diethylamide
What extraintestinal manifestions are there for Crohn's patients?
Uveitis\n Migratory polyarthritis\n Erythema nodosum
What type of renal calculi are Crohn's patients at risk for? Calcium
oxalate: B12 absorption is compromised & oxalate absorption is enhanced.
How does primary biliary cholangitis present? What is seen histologically?
4050 year old female patient with pruritus with or without
jaundice\n Histo: granulomatous degeneration of the intrahepatic bile
ducts\n LAB: antimitochondiral antibodies are positive in >90% of Px
What liver disease is strongly associated with UC?

primary sclerosing

cholangitis\n inflammatory fibrosis of the intra & extrahepatic bile


ducts\n hyperbilirubinemia, jaundice, ultimately cirrhosis\n Dx:
endoscopic retrograde cholangiopancreatography ("beading" along the
biliary tree)\n Up risk for cholangiocarcinoma
What is a rare but serious side effect of metformin? lactic acidosis due to
over suppression of hepatic gluconeogenesis & promotion of glycolysis
What trinucleotide repeat results in Jacobsen's syndrome? Expansion of
CCG on Chromosome 11:\n Mental retardation\n Facial Dysmophisms\n
Cardiac defects\n Thrombocytopenia\n Trigonocephaly
What trinucleotide repeat results in fragile X syndrome?

CGG on the X

chromosome = FMR1 RNAbinding protein gene\n is the 2nd most


common cause of inherited mental retardation (Down's is 1st)
What trinucleotide repeat results in myotonic dystrophy? CTG on
Chromosome 19\n Autosomal dominate disorder that presents late in
childhood\n Gait abnormalities, weakness of upper extremities (intrinsic
hand & wrist extensors)\n Atrophy of facial muscles resulting in
characteristic facies

What trinucleotide expansion results in Friedreich's ataxia?

GAA on

Chromosome 9:\n AR: pyramidal, dorsal, & spinocerebellar tracts are


affected:\n Progressive ataxia & neurologic delince\n RIP:
cardiomyopathy
What leftshifts the O2 dissociation curve?

Hemoglobin will have a

higher affeinity for O2:\n Fetal hemoglobin \n Increased pH\n decreased


temperature\n decreased 2,3DPG levels\n decreased arterial CO2
pressure
When is tamoxifen prophylaxis indicated?

When the patient has had a

hysterectomy (i.e. no more risk of endometrial carcinoma) & the


endrometrial estrogen receptors will no longer be stimulated.\n It is an
antagonist for DNA in breast tissue & an agonist in endometrial, bone &
hepatic tissue
What drugs are analogues of gonadotropinreleasing hormone?
Leuprolide\n Goserelin\n\nMOA: decrease in FSH & LH > decreased
androgen & estrogen synthesis
What drug is a recombinant cytokine used to augment recovery of bone
marrow?

Filgratim (granulocyte colonystimulating factor)

What is the MOA of Flutamide? Blocks androgen receptors & prevents the
body from usng sex steroids
What is the MOA of Imatinib mesylate? Blocks the ATPbinding site on the
bcrabl tyrosine kinase domain & is used to treat CML.
What is the MOA of Tacrolimus? Suppresses cellular immunity by binding
to the intracellular protein FKBP12 & inhibits the activation of T
lymphocytes
What drug is a monoclonal antibody against HER2 that can kill breast
cancer cells over expressing HER2?

Trastuzumab

What is the MOA of Paclitaxel & other taxanes?

Inhibit

depolymerization of microtubles\n Cell cycle arrest in mitosis\n\nRx:


breast cancer

What type of breast cancer is Tamoxifen useful for? ONLY estrogen


receptorpositive beast cancer: \n blocks estrogen receptors to impeded
the production of estrogenresponsive genes
What drug binds to the glycoprotein receptor IIb/IIIa on activated platelets
& prevents fibrinogen from binding & therefore interfers with platelet
aggegation? Abciximab\n\nRx: acute corinary syndrome & angioplasty
What can leuprolide be used for?

GRH analogue:\n Pulsation:

infertility\n Continuous: prostate cancer & uterine fibroids


What 2 drugs prevent fibrinogen from binding to platelets during
aggregation by irreversibly blocking the ADP receptors?

Clopidogrel\n

Ticlopidine\n\nRx: aucte coronary syndrome or prophylaxis after coronary


stenting
What is the most common cause of communityacquired, lobar
pneumonia? Streptococcus pneumoniae
How does atypical pneumonia present on chest exam?

Diffuse, patchy

inflammation localized to interstitial areas at alveolar walls


What are the physical exam signs of Streptococcus pneumoniae
pneumonia? Lung consolidation resulting in:\n Bronchial breath sounds\n
Dull resonance\n Increased tactile fremitus
What test confirms autoimmune hemolytic anemia? Positive direct Coombs
test after lab values indicate hemolytic anemia:\n Elevated LDH\n
Increased indirect bilirubin\n Low haptoglobin levels
What is the DDx for the etiology of autoimmune hemolytic anemia?

Idiopathic\n Viral induced\n SLEinduced\n Lymphoma


What causes microangiopathic hemolytic anemia? any mechanical
trauma to RBCs:\n Defective prosthetic cardiac valves\n TTP\n
Hemolytic uremic syndrome\n DIC
What are neuroleptics?

Dopamine receptorblocking agents (e.g.

haloperidol: central receptor blocker)\n\n Used to treat positive symptoms


of schizophrenia: illusions/hallucinations, delusions & ideas of reference,
agitation, talkativeness

What are the negative symptoms of schizophrenia? Cognitive


disturbance (memory, orientation, consciousness)\n Flat affect\n Poor
grooming & appetite\n Social Withdrawel\n\nRx: nontraditional/atypical
antipsychotics to modulate central serotonergic activity & dopamine
receptors.
What is the MOA of nondihydroperidines?

Cachannel blocker that

prolongs the PR interval\n Prolongs phase 0 & 2slows the recovery of


Cachannelsespecially in the AV nodal cells\n\nRx: nodal arrhythmias
How can dihydroperidine Cachannel blockers (e.g. nifedipine) help to
reduce hypertension?

Inhibit Ca influx in the vascular smooth muscle

cells \n Decrease vascular tone


How does GH act on tissue (i.e. via what kind of receptor)? Binds a
tyrosine kinase associated receptor that then associates with a tyrosine
kinase to add phosphates & increases intracellular signaling via the
JAK/STAT pathway\n Causes transcriptional modification of the progrowth
pathways
What hormones work via the Gq receptor pathway via phospholipase C &
IP3/DAG pathway? Thyroidreleasing hormone\n Growth hormone
releasing hormone\n Gonadotropinreleasing hormone oxytocin\n
Angiotensin II\n Bradykinin
What hormones work via the Gs protien receptor pathway to increase
adenylate cyclase activity to increase intracellular cAMP & protein kinase A
levels?

TSH\n LH\n FSH\n ACTH\n Glucagon\n ADH\n PTH

What compounds work via a receptor tyrosine kinase (receptor has


intrinsic tyrosine kinase activity that autophosphorylates tyrosine &
progresses down the Ras/MAP kinase pathway)?

Insulin\n Growth

factors: Endothelial growth factor, plateletderived growth factor


What is the action of Bcl2 & bclx?

Antiapoptotic: prevents the release

of cytochrome c & stops it from binding to APAF1 & activating the


caspase cascade
What are the 2 actions of p53

stops the cell cycle & activates DNA

repair proteins\n initiates apoptosis if repair cannot be made

What does cmyc do?

Upregulates the cell cycle (drives cell

proliferation) \n e.g. Burkitt's lymphoma


What type of proteins are bax and bak?

proapoptotic: trigger release

of caspaseactivating proteins to trigger cell death


In what cancers is the constitutive ras G protein activated?
neurofibromatosis
What is the most common presentation of von Willebrand diseases?
Mucocutaneous bleeding (e.g. after a dental extraction)
What do schistocytes indicated? intravascular hemolysis
What is overproduced in Waldenstrom's macroglobulinemia?
Neoplastic plasma cells produce monoclonal imunoglobulin M heavy
chains: \n leads to hyperviscosity syndrome & often to Raynaud's
What genetic disorders increase the risk of a saccular or berry aneurysm?
Polycystic kidney disease\n Connective tissue disorders\n
Coarctation of the aorta
What UTIcausing organisms often result in struvite stones?

Urease

positive (alkalize the pH of the urine):\n Proteus\n Klebsiella


What bacteria causes hospitalacquired or drugresistant UTIs and
produces a blue/green pigment & fruity odor?Psuedomonas aeruginosa
(gram , anaerobic bacillus)
What organism can cause hospitalacquired, drugresistant UTIs &
produce a red pigment & discoloration of the urine? Serratia marcescens
What is the second leading cuase of communityacquired UTIs?
Staphylococcus saprophyticus (gram +)
What causes Microscopic Polyarteritis (MPA)? White individual\n
Activation of neutrophils & monocytes by perinuclear antineutrophil
cytoplasmic antibodies (pANCAs) leads to inflammation & vasculitis in
small arteries (esp glomerulous & pulmonary capillaries):\n S/S: dark
brown stools, palpable purpura along extremities, reddish sputum

What is the DOC (& 2nd choice) for Absence seizures?

1st:

Ethosuximide\n2nd: Valproic acid


What is Carbamazepine used to treat? tonicclonic (grand mal) & partial
(focal) seizures
What is the DOC for status epilepticus?

Diazepam

What analogue of yaminobuteric acid is not approved for use with absent
seizures?

Gabapentin

What antiseizure medication is effective in all seizure types except


absence seizures? Phenytoin\n\nSE: sedation, ataxia, nystagmus,
hirsutism, gingival hyperplasia
What is the cellular failure in Icell disease?

Failure of the addition of

mannose6phosphate by GIcNAc phosphotransferase on the GOLGI


apparatus = no direction of enzymes to lysosomes & are excreted (e.g.
hexoaminidase iduronate sulfatase, arylsufatase A)\n S/S: skeletal
abnormalities, restricted joint movement, coarse facial features, severe
psychomotor impairment, death in 1st decade of life.
Eating what puts immunocompromised patients or pregnant women at risk
for listeriosis?

Poorly pasteurized milk\nSoft

cheeses\nColeslaw\nReadytoeat turkey & pork


What section of the GI absorbs the largest net amount of water? jejunum
(colon only absorbs 400 mL/day out of 9L/day total)
At what pH is it advisable to administer bicarb to correct an acidosis?

pH

< 6.9
What enzyme is lacking in phenylketonuria (PKU)?

phenylalanin

hydroxylase: usually converts phenylalanine to tyrosine using


tetrahydrobiopterin as a cofactor
What causes "fith disease" of childhood with a bilateral, red,
maculopapular cheek rash with a "slapped cheek" appearance (erythema
infectiosum)?

Parvovirus B19 (ssDNA virus)

What are the only pathogenic gram diplococci?

Neisseria

How does a disseminated N. gonorrhoeae infection present?

Acute

onset of fever\n Swelling of 2 joints\n 75% of Px also have skin lesions


on the extremities
What lab value is the hallmark of obstructive lung disease?
Decreased FEV1:FVC ration (to < 80%)\n also increased FRC &
TLC\n\n(FEV1:FVC is normal or increased with restrictive lung disease b/c
FVC is reduced along with TLV)
What genetic translocation results in mucosaassociated lyphoid tissue
lymphoma? t(11:18)
What genetic translocation results in follicular (Blymphocyte) lymphoma?
bcl2 (antiapoptotic) from 18 to 14 (Ig heavy chain locus)
What protein is formed when a patient has CML?

Protein tyrosine kinase

bcrabl: due to the Philadelphia chromosome translocation (9:22)


What is Mesna disulfide used to prevent?

Hemorrhagic cystitis

associated with cyclophosphamide usage\n MOA: reduced to mensa (a


free thiol compound) that reacts with urotoxic products
What drug is used to prevent &/or treat tumor lysis syndrome associated
with chemo for rapidly growing leukemia/lymphoma cells? Allopurionol:\n
inhibits xanthine oxidase to decrease hyperuricemia when DNA synthesis
is inhibited
What drug is used as prophylaxis against doxorubicinassociated
cardiomyopathy?

Dexrazoxane

What drug can be used to reduce the megaloblastic anemia caused by


methotrexate therapy?

Leucovorin (5formyltetrahydrofolic acid):\n

bypasses dihydrofolate reductase (blocked by methotrexate) to replenish


the folate pool
What accumulates in galactosemia?

galactose1phosphate &

galactitol\n\n Galactokinase phosphorylates galactose to galactose1


phosphate & G1PUR converts this to glucose1phosphate.\n G1PUR is
lacking in galactosemia

What causes SjogrenLarsson syndrome & how is this disease


characterized?

1) AR: Mutation in the fatty aldehyde dehydrogenase

gene on chromosome 17p\n2) Congenital ichthyosis (dry & scaly fishlike


skin); mental retardation; spastic paraplegia
What causes the neurological problems associated with B12 deficiency?
B12 is a cofactor for the conversion of methymalonyl coenzyme A
to succinyl coenzyme A for the breakdown of oddchained fatty acid.\n
Without B12, toxic byproducts damage Schwann cells in the PNS\n Dx:
methylmalonic acid (MMA) levels rise (even before B12 levels decrease) +
megaloblastic anemia
What vitamin is needed as a cofactor for the conversion of propionyl
coenzyme A to methylmalonyl coenzyme A? Biotin
What drug is used to revive bone marrow in order to prevent infection &
decrease the duration of neutropenia associated with myelosuppressive
chemo?

Filgrastim (GCSF): \n recombinate cytokine\n stimulates

production, maturation, & activation of neutrophils


What drug is used to treat acute tumor lysis syndrome?
What is Oprelvekin & what is it used for?

Allopurinol

Interleukin11:

throbopoietic growth factor\n Combats thrombocytopenia


What results from prolonged oligohydramnios (<0.5 L of amniotic fluid)?
Potter's (lethal): hypoplastic lungs, flattened facies, & clubbed feet
What is the MOA of Clomiphene when used as a drug to combat infertility?
Selective estrogen receptor antagonist in the hypothalamus &
anterior pituitary\n Shields negative feedback of estrogen & increases
GnRH release\n GnRH stimulates release of FSH & LH to stimulate the
ovary
What is a common side effect of bisphosphonates? Esophagitis
What is a common side effect of Cabergoline?

psychosis, nasal

congestion, orthostasis, nausea/vomiting, headache\n MOA: Inhibits


prolactin secretion by dopamine receptor agonist action

What is the MOA of gonadotropins when used to treat female infertility?


Increase overian follicular maturation\n\nSE: weight gain & fluid
retention
What is the DOC for CNS stimulate street drugs?

Labetalol:

nonselective alpha & beta antagonist that blocks hypertension & cardiac
stimulation\n\n&/or \n\nNeurolepics: to control agitation & psychotic
symptoms\n\n&/or\n\nDiazepam: to control possible seizures
What is the antidote for acetylcholinesterase inhibitor overdose? Atropine
(muscarinic antagonist)
What is the antidote for benzo overdose?

Flumazenil (benzo receptor

antagonist)
What is the antidote for a CNS stimulate overdose? Fluoxetine (SSRI)
What is the antidote for opiate (heroine/morphine) overdose (sleepy,
lethargic, comatose with miotic pupils, low Bp & depressed respiration)?
Naloxone (opiodreceptor antagonist)
What is the antidote for antimuscarinic drug (atropine, scopolamine,
Jimson weed) overdose (hot & dry skin + CNS stimulation)?
Physostigmine (acetylcholinesterase inhibitor)
What organelle is the sperm's acrosome a development from?

Golgi

apparatus\n Acrosome covers the anterior 2/3 of the nucleus & contains
enzymes (e.g. hyaluronidase) that permit the sperm to penetrate the zona
pellucida of the egg @ fertilization)
What are the chemical markers of bone formation & bone resorption?
Formation: Alkaline phosphatase\nResorption: hydroxyproline
What are the 3 stages of Paget's disease of the bone

Primary

abnormality: overproduction & overactivity of osteoclast\n1) Osteolytic


lesion of marked bone resorption\n2) Disorganized bone formation \n
Bonespecific alkaline phosphatase = up to 10x normal\n Phosphate =
normal \n Ca = slightly elevated or normal\n3) Sclerotic or Burnedout
phase

How does hemophilia A present?

Bleeding into a joint or

retroperitoneal space\n Lab: Elevated PTT (b/c of factor VIII deficiency)


Which is more common, hemophilia A or B?

A = factor VIII deficiency

(more common)\n\nB = factor IX deficiency


What vessel supplies Wernicke's area? Inferior division of the left MCA: to
the posterosuperior portion of the superior temporal lobe
What vessel supplies Broca's area?

Superior division of the left MCA: to

the posteriorinferior frontal lobe


What is transcortical motor aphasia?

Caused by damage to the

watershed region between the anterior & middle cerebral arteries\n Like
Broca's aphasia, but with better repetition
What is transcoritcal sensory aphasia? Caused by damage to the
watershed region between the middle & posterior cerebral arteries\n
Sounds like Wernicke's, but the Patient's repetition is better than
spontaneous speech
How do patients with agromegaly present?

1) Glucose intolerance\n2)

Coarsening of facial features\n3) Thickened fingers\n4) Macroglossia >


obstructive sleep apnea\n\nCause: GHsecreting adenoma
What is the DOC for agromegaly?

Octreotide (somatostatin analog

that suppresses GH secretion @ the anterior pituitary)


What is the MOA of Somatrem? Somatotropin (GH analog) that stimulates
the release of insulinlike growth factor1 (aka somatomedin) from the
liver\n\nRx: GH deficiency
What are the derivatives of the 6th aortic arch?

Proximal pulmonary

arteries\n Ductus arteriosus: connects the pulmonary trunk to the aorta


(lung bypass)
What are the derivatives of the 2nd aortic arch?

stapedial & hyoid

artery
What are the derivatives of the 3rd aortic arch?
artery\nProximal part of the internal carotid

Common carotid

What are the derivatives of the 4th aortic arch?

Aortic arch on the

left\nProximal right subclavian


What is a SE of magnesium treatment for preeclampsia?

Depressed

tendon reflex
What neuroexam finding can indicate hypomagnesemia? Babinski's sign
What does Chvostek's sign (ipsilateral contraction of the facial muscles
when the facila nerve @ the angle of the jaw is tapped) indicate?
hypocalcemia
What is an ELISA test?

Antigenantibody hybridization: used to detect

antigenic match in a patient's blood sample \n Has a Patient's blood


encountered a particular antigen? If positive: Px has a vaccination or
exposure
What test is used to detect singlepoint mutations? Ligase chain reaction
(LCR): used to diagnose carriers of disease genes
What muscles abduct & medially rotate the thig to keep the pelvis level?
(hint: damage results in Trendelenburg gait) Gluteus medius & minimus
(supplied by the superior gluteal nerve)
Describe the Trendelenburg gait.

Standing still: pelvis sinks away

from the affected muscles\n Walking: trunk lurches to the lesioned side
What muscle(s) extends and laterally rotates the thigh & assists in
standing from a sitting position? gluteus maximus (inferior gluteal nerve)
What do the branches of S1 & L5 innervate? Quadratus femoris:
laterally rotates the thigh\n Oburator internus: laterally rotates when the
leg is extended & abducts when the thigh is flexed
What innervates the obturator externus>

Obturator nerve\n

Laterally rotates the thigh


What is the action of the piriformis & what innervates this muscle?

When leg is extended: laterally rotates the leg\n When the leg is flexed:
abducts the leg

What is the vector for Borrelia recurrentis?

Human louse: causes

relapsing fever\n Changes cell surface proteins (antigenic variation) to


evade humoral immunity
What organisms are capable of antigenic variation? 1) Salmonella
species\n2) Trypanosomes\n3) Neisseria gonorrhoeae\n4) Influenza virus
What filarial nematode causes river blindness?

Onchocera volvulus

What drug can reverse an isoniazid overdose induced seizure?


Pyridoxine b/c isoniazid is a competitive antagonist of pyridoxal
kinase that helps to synthesize GABA from vit B6 (pyridoxine).
What 3 channels can be defective in Brtter's syndrome?

Channel

dysfunction causes hypokalemia & metabolic acidosis:\n1) Na+/K+/2Cl


(NKCC) \n2) K+ channel\n3) Cl channel\n\nAll in the thick ascending limb
(where loop diuretics act)
What causes NRDS (neonatal respiratory distress syndrome)?
Inflammatory cellmediated endothelial injury & t/f disruption of
alveolar or capillary integrity with leakage of proteinrich fluid exudate
into alveoli
What is seen in pulmonary alveolar proteinosis?

Alveolar filling with

proteinaceous material
How is ARDs (Acute respiratory distress syndrome) characterized?
Severe lung injury with acute hypoxemia & pulmonary edema
resulting from increased pulmonary capillary permeability
What immunological mediated disease results in deposits at the
alveolocapillary membrane?

Goodpasteur's

What cause an intense inflammatory response in the bronchi & alveoli


(pneumonitis) within hours in a neonate?

Meconium aspiration

syndrome
What is the MOA of metoclopramide & what are it's main SE?

MOA:

antiemetic that also relaxes the pylorus (Rx: gastroparesis)\n\nSE:


parkinsonismtype extrapyramidal effects

What is sodium thosulfate used for?

Convert cyanide into into

thiocyanate via Rhodenase\n Cyanide is a a metabolite of nitroprusside &


can inhibit cytochrome oxidase
What is the MOA of DNP (2,4dinitrophenol)? Uncoupler: destroys the
mitochondrial membrane's proton gradient
What substance is formed in CO poisoning?

Carbon monoxyhemoglobin

What is a common complication of CLL (chronic lymphocytic leukemia)?


Autoimmune hemolytic anemia with warm antibodes
How many eggs must one eat to become biotin deficient? > 20
What vitamin found in green leafy veggies is heat sensitive & can be
destroyed by cooking?

Folic acid: methyl group transferer to form

amino acids, purines, & thymidine\n\nDeficiency: megaloblastic anemia or


neural tube defects
What is the symptomatic difference between folic acid & B12 deficiency?
B12 has neuro in addition to megaloblastic anemia
What are the symptoms of vitamin E deficiency?

Defective lipid

absorption\n\nPremature infants: RBC membrane deposits


What artery supplies the LV, the RV?

LV: left (or obtuse) marginal (follows

the left border of the heart)\n\nRV: Right marginal (follows the right border
of the heart)
What is the most treatable form of dementia?

Hypothyroidism

What causes a wellcircumscribed granuloma in the liver? Echinococcus


infection
What typically causes an irregular mass lesion in the liver?
Hepatocellular carcinoma
What viral illness predisposes to subsequent bacterial pneumonia in an
otherwise helathy individual?
the upper respiratory tract

influenza via damage of the epithelium of

What infection causes profuse, frothy discharge that is associated with


vulvovaginal pruritis, tenderness, & burning? Trichomonas vaginalis:\n
Histo: small pearshaped flagellate orgs on web mount\n Can cause
small, red, punctuate leasions on the cervix & vagina
What does bloody, foulsmelling vaginal discharge suggest?

Vaginal

foreign body
What is indicated with pelvic or lower abdominal pain with an abnormal,
foulsmelling vaginal discharge? PID: \n Can involve endometrium,
fallopian tubes, ovaries, peritoneum\n Orgs: Neisseria & Chylamydia
What causes intense pruritis with a thick, odorless, white, cottage cheese
like vaginal discharge?

Candida: buddy yeasts & hyphae on wet mount

What presents with an unpleasant, fishysmelling vaginal discharge that is


thin, graywhite & homogenous?

Bacterial vaginOSIS\n \nDx: clue

cells (bacteria coating epithelial cells)


What tricyclic antidepressant can reduce night terrors & enuresis?
Imipramine: decreases stage 4 sleep
What infection causes HUS (hemolyticuremic syndrome)? E.
coli\n\nResult: Thrombocytopenia, Acute Renal Failure & DIC
What can be used to treat moderate cases of von Willebrand disease or
hemophilia A?

Desmopressin (ADH analog): improves platelet function

and promotes the release of vonWillebrand factor & factor VIII


What is a jawrelated complication of chronic bisphosphonate therapy?
Osteonecrosis of the jaw (avascular necrosis of the jaw)
What differentiates Osteomalacia from osteoporosis?

Osteomalacia:

new matrix can be synthesized, but it cannot be mineralized (usually due


to vitamin D deficiency)\n\nOsteoporosis: reduced synthesis of new bone
matrix
How does osteomyelitis show up histologically?

Neutrophils & bone

necrosis on micrscopic exam\n\nCause: S. aureus or M. tuberculosos


(macrocytic predominance if M. tub...)

Where is osteosarcoma usually found? Metaphysis of long bones: \n


Characterized by formation of new bone matrix\n Most common primary
malignancy of bone
When can neuroendocrine tumors cause carcinoid syndrome?

When

they met to the liver and serotonin is no longer metabolized by 1stpass


hepatic metabolism:\n diarrhea, cutaneous flushing, asthmatic wheezing,
Rtsided heart disease
Besides the skin, in what organs is Kaposi's sarcoma also found?
Lungs\n GI\n Biliary tree
How does Kaposi's infiltration of the GI present?

Mucosal bleeding:\n

Hematochezia\n Hematemesis\n Melena (blackcolored stools)


How is CriglerNajjar syndrome type 1 characterized?

AR: UGT1

deficiency (Uridine diphosphateglucuronoxyltransferase)\n unconjugated


hyperbilirubinemia b/c no conjugation of bilirubin with glucuronic acid \n
fatal within 18 months due to kernicterus
What is the most commonly diagnosed porphyria? Porphyria cutanea
tarda\n Deficiency of uroporphyrinogen decarboxylase\n Can result in
2ndary hemochromatosis
What is primary gain?

When an individual subconsciously expresses an

internal conflict as a symptom of a physical or mental illness, leading to a


relief of their anxiety
What is La belle indifference?

When a person has a strikingly cavalier

attitude about something very serious


What is secondary gain? Willful invention of symptoms in order to obtain
something desired or avoid something unpleasant
What is tertiary gain?

Benefit that the care provider derives from

treating the sick individual


What pathway is monitored when treating a patient with Heparin?
PTT (intrinsic coagulation pathway)\n Heparin activates
antithrombin III > decreases thrombin & inhibits factor Xa

When do you monitor INR (international normalized ratio)? Standarizes


prothrombin (extrinsic pathway) time: warfarin\n INR should be between
2&3
What is an ArnoldChiari malformation?

Displacement of the

hindbrain through the foramen magnum\n causes syringomyelia: damage


to the crossing fibers of the spinothalamic tract & bilateral loss of
pain/temp sensation in the upper extremities
How does Cushing's syndrome cause poor wound healing? Inhibition of
collagen synthesis by glucocoritcoids
Does smoking increase or decrease the risk of endometrial cancer?
Decreases
What is the classic triad of acute graftversushost disease (GVHD)?

1)

Dermatitis\n2) Hepatitis\n3) Gastroenteritis


Are isolation, acting out, sissocation, denial, fixation, identification,
isolation of affect, projection, rationalization, reaction formation, splitting,
repression, or displacement mature or immature defense mechanisms?
Immature
What is the mechanism of primary graft rejection? When neutrophil &
platelet recover does not occur after transplation\n Due to recipient
immune system attack on alloantigens expressed on donor stem cells
Is humor (appreciating the amusing nature of an anxietyprovoking or
adverse situation) a mature or immature defense mechanism?
How do (some) bacteria use IgA protease?

Mature

To cleave secretory IgA &

colonize mucosal areas


What are the mature defense mechanisms?
SASH:\n\nAltruism\nHumor\nsublimation\nsupression
What does Clostridium perfringens produce which causes gas gangrene,
cellulitis, & diarrhea?

Lecithinase

What is the pathogenesis of communicating hydrocephalus secondary to


meningitis? Arachnoid granulations may become scarred & fail to resorb
adequate CSF\n CSF builds up in subarachnoid space & ventricles
What are 3 examples of acutephase proteins?

Creactive

protein\nMannosebinding Lectin\nPulmonary surfactant proteins\n\n All


produced by the liver\n All are opsonins & activate complement
What is hydrocephalus ex vacuo?

When brain mass is progressively

lost, cerebrospinal fluid accumulates to fill the void.


What do the following symptoms indicate?\n Anxiety & irritability\n
Neuromuscular excitability\n Tetany\n Intracranial calcifications\n
Dental abnormalities\n Cardiac conduction abnormalities
Hypoparathyroidism\n\nCauses: thyroidectomy, metastatic cancer,
DiGeorge's syndrome
What can cause hydrocephalus via CSF overproduction?

Choroid plexus

papilloma
What gramnegative, nonmotile facilitative anaerobe causes abdominal
pain, high fever & bloody diarrhea in the developing world?

Shigella

flexneri
What infection is associated with 30% of polyarteritis nodosa patients?
Hepatitis B
What urine values are characteristic of Nephrogenic or Central DI?

Low urine specific gravity\n High serum osmolality


What are the xray signs of a transposition of the great vessels? 1)
Enlarged heart\n2) Domeshaped heart (fried egg yoke seen from the
side)\n3) Pulmonary vascular markings are increased
What is the most common invastive tumor of the breast? Ductal
carcinoma\n Histo: anastomosing sheets of pleomorphic cells\n Most
aggressive tumors overexpress HER2/neu
What reflex action happens with norepinephrine?
to an increased in SV.

Reflex bradycardia due

What nerve is damaged when the eye appears "down & out"?

CNIII

(occular motor):\n Loss of superior & inferior rectus, inferior oblique


muscles\n Dominance of superior oblique & lateral rectus\n Ptosis: loss
of levator palpebrae superioris\n Dilated pupils due to loss of
parasympathetic fibers (unless the damage is an infarction)
What are "pressor" drugs?

drugs used to increase Bp

What are the endocrine hormones of the pancreas? Alpha Cells:


glucagon\nBeta islet Cells: insulin\nDelta islet cells: somatostain
What is the formula for MAP?

2/3 diastolic + 1/3 systolic Bp

What are the exocrine hormones of the pancreas?

Digestive enzyme:

amylase, lipase, trypsin, chymotripsin\n\n Acinar cells (large, eosinophilic


cytoplasm, pyramidalshaped, polar)
How does a patient with an L5S1 disc herniation present? S1 nerve root
damage:\n Pain & paresthesia & sensory loss over: back of thigh, lateral
posterior calf, & lateral foot; loss of motor of gastrocnemius w/ or w/o
deficit in foot eversion\n LOSS of Achilles relfex
What cells of the Gi secrete CCK (cholecystokinin)? mucosal
neuroendocrine cells \n primarily in the dudenum\n pyramidal, extend
from the BM to the lumen with microvilli & secretory granules
When is the patellar reflex lost? Herniation affecting the L4 nerve root:\n
pain & sensory loss over anterior thigh & interior shin, + motor deficits of
the quadriceps
What does a deficiency of the CD40 ligand on activated CD4+
lymphocytes cause?

Xlinked, hyperIgM syndrome:\n Deficiency of

IgG\n Normal/Elevated IgM\n Recurrent pyogenic infections (no IgG


opsinization) & P. jiroveci pneumonia b/c of the defect in cellmediated
immunity
What results in an abnormal plantar reflex or Babinski's sign?

UMN

injury with the lesion to the corticospinal tract above S1


What does a patient with chronic, intermittent abdominal pain, diarrhea,
fever, & megoblastic anemia likely have?

Crohn's

What is the Conus medullaris syndrome?

Damage to the distal part of

the spinal cord that presents with:\n lax anal tone\n Urinary retension\n
incontinence\n impotence\n Bilateral saddle anesthesia (S35) may be
present
What causes neurogenic claudation?

Lumbar spiral stenosis \n back or

buttocks pain induced by walking or prolonged standing.\n Pain relieved


by rest or forward flexing of the vertebral column.
What does an increase in hemoglobin A2 indicate? Bthalassemia minor
= hypochromatic, microcytic anemia
What is the triad for parathyroid adenomas? Stones, bones, abdominal
groans, & psychic moans:\n1) Nephrolithiasis\n2) Osteoporosis or osteitis
fibrosa cystica\n3) Constipation, nausea, vomiting, ulcers, pancreatitis,
gallstones\n4) Depression, lethargy, & eventual seizures
What neurological symptoms are seen with vitamin B12 deficiency?
Increased methylmalonic acid levels impairs myelin synthesis:\n
Primary impact: posterior & lateral spinal columns\n Paresthesias &
impaired proprioception\n\n+ megaloblastic anemia
What are signs of vitamin D deficiency in the elderly due to poor
nutrition/feeding? Poor Dentition\nLoss of appetite\nCreeping dementia
What type of sputum is indicative of hosptialacquired or alcoholic
pneumonia? Bloody sputum resembling dark jelly; caused by gram
rods:\n Klebsiella pneumoniae\n Psuedomonas\n Enterobacter
What lab values indicate Paget's disease of the bone?

Normal Ca &

PO4\n Elevated Alk Phos (significantly elevated)


What are the only endogenous agents that allow dilation of the afferent
arteriole of the kidney?

Prostaglandins\n\n Their production blocked by

NSAIDs and COX2 inhibitors = increased vascular tone & constriction


What 3 endogenous systems/agents constrict the efferent arteriole of the
kidney?
Vasopressin

1) Sympathetic nervous sytem\n2) Angiotensin II\n3)

What lab values indicate parathyroid carcinoma?

1) Hypercalcemia\n2)

Hypophosphatemia\n3) Elevated PTH


What is the action of COX2 inhibitors? Modulate vascular & bronchial
tone\n\n DO NOT modify vascular permeability or platelet aggregation
(NSAIDs change platelet aggregation)
What is the physical sign of LMN lesions?

Fasciculations: twitching of

the muscle as it loses innervation\n Cause (in ALS: amylotrophic lateral


sclerosis): release of acetylcholinesterase from the degenerating nerve
terminal at the neuromuscular junction
What is the cutaneous innervation of the median nerve?

1) Palmar hand:

lateral portion up to the lateral 1/2 of the 4th digit\n2) Dorsal hand: \n
distal to the proximal interphalangeal joints of the 13 digints\n the dorsal
aspect of the radial 1/2 of the 4th digit distal to the proximal
interphalangeal joint\n THENAL muscles ("Oaf"): Opponens pollicis,
abductor pollicis brevis, flexor pollicis brevis
WHat causes hypogonadism due to hypogonadotropic cuases?

Cause:

Impaired secretion of LH & FSH do to congenital GnRH deficiency \n


Characterized by low testosterone in the setting of low LH & FSH
Where is warfarin metabolized? p450 system
What connection caus increase the rist of testicular torsion?

When the

tunica vaginalis is attached high on the spermatic cord


What drugs for GERD inhibit the CYP450 system?

PPIs (e.g.

omeprazole)\n Cimentidine (antihistamine)


What reverses heparinization?

Protamine sulfate: it is + charged and

binds charged heparin decrease antithrombin III activation (& thereby


reinstating levels of thrombin & factor Xa)
What type of seizures is Phenytoin used for? Gran mal (generalized tonic
clonic)\nComplex partial
What is Aminocaproic acid used for?

Reverse the thrombolytic effects

of:\n Streptokinase\n Urokinase\n Tissue plasminogen activator


\n\nMOA: inhibits fibrinolysis

What drug is used prophylactically to prevent blood loss in cardiothoracic


surgery? What is its MOA?

Aprotinin: slows bleeding by blocking

plasmin \n\n Also inhibits streptokinase


What type of seizures is Carbamazepine used to treat?

Partial complex

What results from a loss of function of the NF2 gene on chromosome 22q?
NF2 gene no longer produces a protein that interacts with the cell
membrane & cytoskeleton:\n AD Neurofibromatosis type 2 with bilater
vestibular (CNVIII) schwannomas & meningiomas\n\nMneumonic: NF type
2 = Ch. 22
What 3 seizure meds induce the CYP450 system?
Penytoin\nCarbamazepine\nPhenobarbital
What ECG finding indicate hypOkalemia? hyPERkalemia? Hypo:
Flattened T wave & U wave\n Hyper: Peak T waves & prolonged PR
interval
What are the side effects of Quinidine? Diarrhea\n Cinchonism: tinnitus,
vertigo, vision problems, & delirium
What is DOC in a penicillinallergic patient with a pseudomonal infection?
Aztreonam: inhibits cell wall synthesis
What antimalarial med can also be used to covert atrial fibrillation or
suppress ventricular arrhythmias?
What is Tinea unguium?

Quinidine

Fungal infection of the nail with

Trichophyton\n\nRx: Griseofulvin (CYP450 inducer)


What is Beck's triad?

Indicates hemopericardiua:\n1) Hypotension\n2)

Increased JVP\n3) Distant heart sounds


What are the drugs used to treat trigeminal neuralgia?

1st:

carbamazepine\n2nd: Phenytoin\n\nBoth induce CYP450


What type of degenerative dementia presents with visual hallucinations,
problems with executive & visuospatial functions, & trouble driving?
Lewy body Dementia:\nDementia + 2 of these\n1) Cogntivie
fluctuations\n2) Visual hallucinations\n3) Parkinsonism\n\nLewy bodies are

eosinophilic & involve the entire cerebral cortex (stained by alpha


synuclein)
What is the clinical triad for renal cell carcinoma?

1) Flank pain\n2)

Palpable abdominal mass\n3) Hematuria\n\n+ polycythemia (ectopic epo),


fever, & weight loss\n\nIf genetic, associated with von HippelLindau
syndrome (Ch. 3 mutation)
What histopathology is seen in Alzheimer's disease?

1) Amyloid

proteins\n2) ApoE\n3) Taupathy \n4) Synaptic loss\n5) Senile plaques\n6)


Neurofibrillary tangles\n7) Granulovascular degeneration
What symtom is present with CreutzfeildJakob disease & missing in
dementia with Lewy bodies?

CJD is must rarer & presents with

myoclonus\n Electroencephalography shows burst suppression & periodic


complexes
What conditions go along with APKD? Polycycstic liver disease\n Berry
Aneurysms\n Mitral valve prolapse\n\nCause: APKD1 mutated gene on
Ch. 16
What protease inhibitor is associated with crystalinduced nephropathy?
Indinavir: precipitates in the urine & obstructs its flow\n\nRx:
HIV/AIDS
What organism causes a chest Xray picture of diffuse infiltrates bilaterally
along with a cough & shortness of breath in an HIV patient?
Pneumocystis jiroveci
What the does the allantoic duct eventually become?

Median

umbilical ligament\n\n"allaNtosis" has an "N", as does "mediaN"


What organism presents on Xray with consolidations that are typically
noted in the apical or posterior portions of the upper lobe or superior
portions of the lower lobe?

Mycobacterium tuberculosis

What is the ligamentum teres hepatis a remnant of?

Umbilical vein

(carries O2 blood from mother to baby)


What type of colonic polyp is a risk factor for colorectal cancer? Villus
adenoma

What is the ligamentum arteriosum a remnant of?

Ductus arteriosum

(shunts blood from the pulmonary artery to the aorta in the normal fetal
circulation)
What are the medial umbilical ligaments a remnant of?

Fetal umbilical

arteries (carry blood from the baby to the mother)


How is prolactin endogenously regulated?

Dopamine secreted by the

neuroendocrine cell nuclei of the hypothalamus suppress prolactin


secretion from trophic hormonesecreting cells of the anterior pituitary.\n
Bromocriptine (drug) can stimulate this loop to inhibit prolactin
What drug, used to treat gastric ulcers induced by NSAID use, is an
abortofactant?

Misoprostol (Prostaglandin E1 analog)

What enables working muscle to have a greater blood supply during


exercise?

Selective vasodilation via metabolic factors: lactate,

potassium, & adenosine\n Overcomes sympathetic effect that exercise


also induces to vasocontrict via alpha receptors
What drugs are H2receptor antagonists?
Ranitidine\nCimetidine\nFamotidine\n\nRx: inhibit acid secretion at
the histamine receptor
What happens in the lungs during exercise? Pulmonary resistance
decreases to accommodate greater pulmonary blood flow\n Decreased
resistance improves gas exchange & allows an even distribution of blood
throughout the lungs.
What diuretic is the DOC for a patient with a sulfa allergy? Ethacrynic
acid\nMannitol (but not for CHF or anuric patient)
What is reactive arthritis? a seronegative spondyloarthopathy with HLA
B27 \n an arthritis that follows a GI or GU infection by 24 weeks\n "can't
SEE, can't PEE, can't CLIMB A TREE"\n Extraarticular manifestations
include:\n1) conjunctivitis\n2) keratoderma blennorrhagia
(papulosquamous skin rash on palms/soles)\n3) Urethritis\n4) Circinate
balanitis (lesion on the penile shaft)\n5) Aortitis (occasionally seen)

What drug causes gray baby syndrome?

Chloramphenicol (also

causes aplastic anemia)


What are Heberden's nodes on distal interphalangeal joins of fingers
manifestations of? Osteoarthritis: destruction of articular cartilage
What drug group has a disulfiramlike reaction with alcohol use?
Cephalosporins (cefamandole, cefotetan, cefoperazone,
cefmetazole)
What is Ulnar deviation of the fingers a late manifestation of?
Rheumatoid arthritis
What chronic autoimmune liver disorder can present with sever itching,
hepatomegaly, & xanthomas?

Primary biliary cirrhosis\n\nLAB:

antimitochondrial antibodies\nHISTO: Granulomatous destuction of


mediumsized intrahepatic bile ducts
What drug group can cause Kernicterus?

Sulfonamides: they displace

bilirubin from protein, resulting in excess free bilirubin


What do elevated levels of anticentromere antibodies indicate?

CREST

variant of scleroderma
What antibiotic drug groups are known to cause ototoxicity?
Aminoglycosides\nVancomycin
What does antihistone antibodies indicate?

Druginduced lupus

syndromes
What do antinuclear antibodies often indicated?

Connective tissue

diseases like SLE


What antibiotic is associated with teeth discoloration?
What does uvula deviation indicate?

Tetracyclines

LMN lesion contralateral to the side

to which the uvula is deviating \n\nOR\n\nUMN vagus nerve (CNX) lesion


will cause the uvula to deviate toward the side of the lesion
What heart sound is present in sickle cell patients? Systolic ejection
murmur due to increased SV & velocity to compensate for anemia\n

Heard when Hb falls below 6 mg/dL & blood viscosity decreases (increased
turbulence)\n Over mitral or aortic valve
What nerve innervates the palatal archers & uvula? Vagus (CNX)
What class of diuretics can worsen osteoporosis?

Furosemide & other

loops (decrease Ca2+ absorption)


What does tongue deviation indicate? Injury to the LMN of the hypoglossal
nerve (CNXII) on the opposite side due to unopposed action of the
opposite genioglossus muscle
What are the signs of pulmonary edema?

1) Orthopnea\n2)

Dyspnea\n3) Pulmonary rales\n4) Systemic edema of extremities


("backward failure")
What components of the CNS are derived from mesoderm?

Microglia (like all macrophages)\n Dura mater\n Connective tissues:


endo, peri, & epineuria
What are the signs of "forward failure"?

Fatigue due to insufficient CO

to meet matabolic demands


What tissues of the nervous system are derived from neural crest cells?
Peripheral ganglia\n Afferent sensory nerves (dorsal root)\n
Schwann cells
What heart sound indicates volume overload?

3rd heart sound on

auscultation
What are the derivatives of the neuroectoderm?

Posterior pituitary\n

CNS neurons\n Oligodendrocytes\n Astrocytes\n Ependymal cells\n


Pineal gland
What are the 2 main indications for carbonic anhydrase inhibitors like
Acetazolamide?

1) Metabolic alkalosis\n2) Alkalize the urine to remove

a toxic drug
What are the lends of the eye, epidermis, & anterior pituitary derived
from? Surface ectoderm

What are the major actions of Captopril?

ACE inhibitor:\n1) Arteriolar

vasodiation (decreases afterload)\n2) Venous vasodilation (decreases


preload)\n3) Decreased aldosterone (decreases preload)
What hormones act through nuclear steroid hormone receptors? 1)
Thyroid hormones\n2) Cortisol\n3) Aldosterone\n4) Vitamin D\n5)
Testosterone\n6) Estrogen\n7) Progesterone
What is the order of items within the common carotid sheath?

1) Artery

is medial\n2) Internal jugular is lateral\n3) Vagus is posterior


How does Glucagon act on target cells?

Gprotein receptors in the

plasma membrane, an alpha subunit activates adenylate cyclase


How do Histamine & vasopressin act on target cells?

Activate

phospholipase C, resulting in cleavage of phosphatidylinositol diphosphate


to inositol triphosphate & diacyglycerol
Describe the embryo origins of the ear structures.

1st branchial arch:

incus & malleus\n 1st branchial cleft: external auditory meatus\n 1st
branchial pouch: iddle ear cavity, Eustachian tubes, mastoid air cells\n
2nd branchial arch: stapes\n 2nd branchial cleft: temporary cervical sinus
How does intracellular insulin act?

Via a tyrosine kinase cascade

What is the pathogenesis of Alport's syndrome?

Xlinked genetic

disorder with absent or mutated type IV collagen:\n Renal disease\n


Nerve disorders (deafness)\n Ocular disorders\n\nEM: split basement
membrane due to the collagen IV mutation (kidneys & ears)
How does norepinephrine act on target cells? Binding to & activating
adrenergic receptors
What kidney problem shows up as wireloop appearance under EM?
SLE (systemic lupus erythematosus) due to subendothelial
basement membrane deposits
What is acute coronary syndrome?

Either unstable angina or acute

MI\n T wave inversion of ECG indicates myocardial ischemia

What causes dysentery? Mucosal destruction leading to purulent


(epithelial cells) & bloody stools:\n Persists with fasting\n Cause: Shigella
(verotoxin inactivates 60S subunit & kills epithelial cells) & some E. coli
species
Why are ACE inhibitors prescribed after a patient recovers from an MI?
Maintenance therapy to reduce ventricular remodeling
Is osmotic diarrhea relieved by fasting?

Yes: \n Osmotic gap (290

2(Na(stool) + K(stool)) > 50 mOsm\n No increase in fecal fat\n Classic


Causes: celia disease, lactose or fructose intolerance
What disease presents with high fever with uniform renal impairment &
mild hepatitis after a recreational freshwater exposure?
What is secretory diarrhea?

Leptospirosis

Isotonic with plasma\n Net intestinal

fluid secretion\n No abatement with fasting\n Causes: viral or bacteria


(Cholera)
What does a radiologic study showing soft tissue swelling, bone
destruction, & periosteal reaction indicate?

Hematogenous

osteomyelitis\n\n Usually due to Staphylococcus aureus


What drug is most effective med to increase HDL?
What is the MOA of tentus toxin?

Niacin

Blocks release of glycine from

Renshaw cells in the spinal cord & results in "lockjaw"\n Early symptoms:
risus sardonicus (muscle spasminduced grimace), poor prognosis, and
opisthotones (severe hyperextension of the head, neck, & back)
What organism is part of the normal oral flora & can be responsible for
abscesses in the mouth, pharynx, brain, & lung?

Prevotella

melaninogenica
What causes rosegardener's disease? Sporothrix schenckii (fungus) \n\n
Pustule or ulcer at entry point with ascending lymphangitis (rarely causes
a systemic effect)
What 4 components develop into the diaphragm?

"Several Parts Build

the Diaphragm":\n Septum transversum\n Pleuroperitoneal folds\n Body


wall\n Dorsal mesentery of the esophagus

What is the MOA of unmetabolized tyramine? Loaded with beer, red wine,
cheese, sausage & pickles, the TYRE of AMINE is squeezing out biogenic
amines from a presynaptic vesicle of the adrenergic terminal\n
Sympathomimetic: NE release stimulant
What is fetal hydantoin syndrome?

Due to maternal use of anti

epileptics like phenytoin during pregnancy:\n Abnormal skull & facies\n


Growth deficiency\n Underdeveloped nails of fingers & toes\n Mild
developmental delay
What patients should avoid Tyramine? In patients taking MAOs b/c this will
block liver MAO A & cuase the highpressure TYRE of AMINE crisis
What is the MOA of polymyxins? disrupts the cell membrane's osmotic
properties
What is the MOA of phyenlephrine?

Selective alpha1 agonist\n This

PHENomEnaL ElePHant tightens the alpha1 RINg on a segment of


arteriole\n Mydriasis dues to alpha1 activation of the radial muscle of
the iris
What is the MOA of rifampin? What is it used for?

Inhibits DNA

dependent RNA polymerase encoded by the rpo gene\n\nRx: \n


Antimycobacteria\n Leprosy (to delay dapsone resistance)\n
Chemoprophylaxis for N. meningitidis exposure & also for kids exposed to
H. influenzae
What is the clinical use of PHENylephrine?

Priaprism (alpha1 on the

corpora cavernosa to decrease blood flow)\n Hypotension or shock (with


low PVR)\n Eye disorders or exams (mydriatic)\n Nasal & sinus
congestant (decongestant or vasoconstriction in sinus surgery)
What is the MOA of ethambutol? Inhibits the arabinosyl transferase
mediated synthesis of arabinogalactin for mycobacterial cell walls
What is the MOA of dapsone?

Inhibition of folic acid synthesis

What are the major side effects of PHEnyleprine?

P = prostatic

hyperplasia symptoms\nH = hyperemia (rebound) due to chronic use with

nasal/sinus congestion\nE = elevated Bp in Px with


hypertension\n\nHYPERtensive crisis with MOAIs
What is prescribed prophylactically in asymptomatic patients with a
positive PPD?

Isoniazid & B6 (the latter to prevent neurotox)

What is the MOA of Epinephrine?

Equally gives everybody a shot of

adrenaline (alpha1, apha2, beta1, beta2 agonist):\n vasoconstriction


(a1)\n vasodilation (b2)\n cardiac stimulation (b1)\n bronchoconstriction
(b2)\n decreased IOP (increased aqueous humor outflow: E for Efflux)
What is the most poten antituberculous med?

Rifampin (blocks DNA

dependent RNA polymerase)\n\nSE: liver toxicity


What are the 6 clinical uses of Epinephrine? 1) Anaphylaxis\n2) Bleeding
(superficial)\n3) Cardiac arrest\n4) (local) Anesthesia\n5) Bronchospasm
(rarely used)\n6) Coup induced by intubation or infation\n\n7) Openangle
glaucoma
What is the clinical presentation of StevenJohnson's

Prodrome: fever

& fatigue, then skin & mucocutaneous lesions that begin as erythematous
macules, progress to form bullae, & then slough
What drug interacts with Epinephrine to cause severe hypertension or
even cerebral hemorrhage?

nonselective Betablocker

What 3 drugs often cause cutaneous flushing?


Niacin\nAdenosine\nVancomycin
What are the major side effects of Epinephrine?

HAT on the head &

heart:\n Headache, restlessness, cerebral Hemorrhage\n Angina and


cardiac Arrhythmias\n Tachycardia & palpitations
What characterizes erythema nodosum?

Painful erythematous

nodules on the lower legs (+ fever & malaise)\n\nDiverse etiology: drugs,


infections, inflammatory diseases
What is the MOA of Norepinephrine

I'm NO Epinephrine because I can't

activate B2 receptors:\n A1 = A2; B1 > B2

With what underlying disease is pyoderma gangrenosum associated?


Pyoderma gangrenosum: boggy, red ulcers with purulent, necrotic
bases (painful to the touch)
What are the TPR, CO, HR, & MAP values for phenylephrine?

alpha1

>> apha2:\n\nTPR = increased\nCO = reduced (reflex)\nHR = reduced


(reflex)\nMAP = increased
What are the lab results for stage I & II of Lyme disease?

I: elevated IgM

for B. burgdorferi (for 2 weeks)\n\nII: elevated IgG (6 wks later)


What are the TPR, CO, HR, & MAP values for NOrEpinephrine?

alpha1 =

alpha 2 = Beta1 >>> Beta2:\nTPR = Increased\nCO = NA or decreased


(reflex)\nHR = decreased (reflex)\nMAP = Increased
What are the Dx criteria for rheumatic fever?
Pancarditis\nPolyarthritis\nSydenham's chorea\nSubcutaneous
nodules\nErythema marginatum\n\nLab: Leukocytosis with neutrophilia
What are the TPR, CO, HR, & MAP values for Epinephrine? TPR =
decreased\nCO = increased\nHR = increased\nMAP = NA or increased
How does Q fever present?

Sudden onset of 1(+):\n High fevers\n

Chills\n Severe headache\n Malaise\n Myalgia\n Confusion\n Soar


throat\n Nonproductive cough\n Nausea/vomiting\n Abdominal pain\n
Chest pain\n\nDx: serologic testing for antibodies to Coxiella burnetii
antigens
What are the TPR, CO, HR, & MAP values for Isoproterenol?

beta1 =

beta2 >>> alpha 1\nTPR = decreased\nCO = increased\nHR =


increased\nMAP = decreased
Why are benzos prefered over barbiturates in the treatment of anxiety?
Less likely to lead to respiratory depression and central cardiac
depression
What is the MOA of Isoproterenol?

I am SO PRO for the beta

TERRItory of PROpraNOLol a nonselective betablocker, although we are


opposite in actions\n Cardiac stimulation (B1 via Gs)\n Vasodialtion (B2

via Gs in skeletal muscle)\n Bronchodilation (B2 via Gs in broncial smooth


muscle)
What are the signs of a long thoracic nerve injury? 1) Difficulty raising the
arm above horizontal\n2) Winged scapula
What are the 3 clinical uses of Isoproterenol? 1) Bradycardia\n2) heart
Block\n3) Asthma (rarely)
What are the major side effects of isoproterenol?

HAT on the head &

heart:\nH = headache\nA = Angina & cardiac Arrhythmias\nT =


Tachycardia & palpitations
What is the receptor MOA of PTH on the kidney?

Binds to basolateral

hormone receptor in the proximal tubule cells that is coupled to adenylate


cyclase via a Gs protein\n Increased renal Ca reabsorption\n Increased
urinary cAMP\n Phosphaturia
What G proteins are alpha1 & alpha2 receptors linked with?

Gq,

Gi\n\n"AA flight 12 flies on the qi" (chi = air in chinese)


What is the MOA of PTH on the kidney to make acitve vitamin D?
Stimulates 1alpha hydroxylase to produce 1,25OH vitamin D from
25OH vitamin D
What G proteins are beta1 & beta2 receptors linked with? Gs, Gs\n\n"BaBy
goes to SS (sunday school)"
Why do emphysema patients often breathe through pursed lips? Upon
expiration, larger airways collapse more (due to loss of elastic fibers &
recoil) and lead to air trapping. \n Pursed lips causes increased
resistance & increases pressure to keep the airways open.
What G proteins are D1 & D2 receptors linked with? Gs, Gi\n\n"2D grades
makes everybody SIgh"
What is the function of G6PD?

To generate reduced nicotinamide

adenine dinucleotid phosphate from oxidized NADH, which is then used to


reduce glutathione\n Glutathione detoxifies oxidizing agents

What G proteins are M1, M2, & M3 receptors linked with? Gq, Gi,
Gq\n\n"3M is quick (qiq) for posting"
What enzyme generates Glucose6phosphate?

From glucose via

hexokinase
What channels are Nn & Nm receptors linked with? Na channel, Na
channel\n\n"Both Nn & Nm contain the N of Na+"
What is NADPH used for? Maintain a reduced pool of glutathione\n Fatty
acid & steroid synthesis\n Cytochrome P450 system\n Phagocytosis
What G proteins are H1, H2 receptors linked with?

Gq, Gs\n\n"DoubleHit

is q.s. (L quantum sufficit [enough]"


What is the classical pattern of sensorineural hearing loss in older
patients?

Loss of hair cells at the organ of Corit in the proximal portion

of the basilar membrane (loss of hearing highpitched sounds)


What G proteins or channels are 5HT1, 5HT2, 5HT3, 5HT4 receptors
linked with? Gi, Gq, Na+ channel, Gs\n\n"Albeit with a 5feet HeighT, his
IQ is Not Significant (IQNS)"
What are the utricle and saccule for?

Part of the vestibular apparatus in

the ear & responsible for seansing changes in head position with respect
to linear acceleration
What G proteins or channels are GABAa & GABAb receptors linked with?
Cl channel, Gi\n\n"ABCi"
What is rheumatoid factor?

an IgM autoantibody that is directed

against the Fc region of the patient's IgG antibody: immune complexes


form
What G proteins are NMDA receptors linked with?

Na+, Ca2+, K+

channel\n\n"Nmda also contains the N of Na+"


What is the sensitive & specific serological diagnostic test for Rheumatoid
arthritis?

AntiCCP (citrullinecontaining protein) antibodies

What is the MOA of Dobutamine?

DOes the BUTA of MINE excite the

BeTA1 receptor of the heart? definately yes!\n Selective B1agonist\n

Cardiac stimulation (Gscoupled = rise in cAMP)\n Weak alpha1 & beta2


receptor agonism = vasoconstriction & vasodilation
Where is ACh synthesized in the brain?basal nucleus of Meynert
What is the clinical use of Dobutamine?

Refractory or severe CHF

Where is NE synthesized? locus ceruleus in the brain


What are the major SE of Dobutamine?HAT on the heart:\n
Hypertension\n Angina & cardiac Arrhythmias\n Tachycardia
Where is serotonin synthesized? Raphe nucleus
What is the MOA of Terbutaline & Ritodrine? Tow 2 BeTA LINES, the RIghT
sTRINgs used to Relax uTeRINE muscle\n Selective B2 agonists\n Uterine
relaxation (Gs coupled B2 receptors)\n Bronchodilation (Gs coupled B2
receptors)
Where is Dopamine synthesized?

Ventral tegmentum & substantia

nigra
What are the 2 clinical uses of Terbutaline & Ritodrine?

1) Premature

labor (delays labor)\n2) Asthma (Terbutaline, not ritodrine)


What are the 1st symptoms of botulism in adults?

Diplopia &

dysphatiga\n\nCause: contaminated canned goods & smoked fish


What are the major SE of Terbutaline & Ritodrine?

RAT in the brain, HAT

on the heart, & PET in the Lungs:\n\nR = restless\nA = anxiety\nT =


tremor\n\nH = hyerglycemia & hypokalemia\nA = angina &
Arrhythmias\nT = Tachycardia\n\nP = low PO2 (early in Rx for severe
asthma)\nE = edema (pulmonary, rare)\nT = tolerance
How is Follicular lymphoma recognized histologically?

Cellular

attempt to recapitulate lymph node architecture by forming follicles\n


t(14:18) & over expression of antiapoptotic bcl2 gene
What is the MOA of Ephedrine? Activates alpha and beta receptors and
stimulates CA release\n A noncatecholamine (CA) agent

What is overexpressed in Mantle cell lymphoma?

Cyclin D1 (proto

oncogene)
What selective B2agonists end in buterol? Al, Leva, Pir\n\nRx: Asthma
What gives Pseudomonas aeruginosa its bluegreen color? It produces
procyanin \n Gram , oxidase positive, lactosefermenting\n Virulence
factor: exotoxin A (ADP riboxylates & inhibits eF2 in the host cell)
What selective B2agonists end in terenol? Metaproterenol
What is the leading cause of death in cystic fibrosis patients?
Pulmonary infection by alginateproducing P. aeruginosa
What selective B2agonists end in terol?

Bitolterol\nFormoterol (long

acting)\nSalmeterol (longacting)\n\nRx: asthma


What is the MOA of superantigens (like TSST1 by S. aureus)?

Directly

bind to & activates MHCII & Tlymphocyte receptors


What selective B2agonists are used to stop premature labor?
terbutaline & ritodrine
What vessel supplies the region of external hemorrhoids? External
hemorrhoids are below the pectinate line\n Supplied by the inferior rectal
artery (branch from the internal pudendal) & drained by the inferior rectal
vein
What type of drug is Dopamine? Direct sympathomimetic, nonselective
Dagonist
What vessel supplies the external genitalia & a portion of the thigh?
External pudendal
What are the dosedependent effects of dopamine? "DOPeDOse
dePendent Awards of MINE:\n bronze (D1) at low doses = increased renal
Perfusion\n silver (B1) at intermediate = Positive inotropy\n gold
(Alpha1) at high doses = increased Peripheral resistance
What does the inferior mesenteric artery become? In the rectum:
superior rectal artery\n Site of internal (painless) hemorrhoids)

What are the 3 MOAs of dopamine?

1) Low dose: Gscoupled vascular

D1 activation = up in blood flow to renal, mesenteric, & coronary vascular


beds\n2) Medium dose: Gscoupled cardiac B1receptors = positive
inotropic\n3) High dose: Gqcoupled vascular alpha1 receptors to
increase intracellular IP3/Ca2+ to vasoconstrict, increase PVR & Bp
What is the main step in melanin production? Oxidation of tyrosine via
tyrosinase in melanocytes
What are the 2 clinical uses of dopamine?

1) Refractory or severe CHF,

especially for patients with decreased renal blood flow & low Bp\n2)
Cardiogenic or septic SHOCK
What is the most effective medical therapy for a uterine leiomyoma?
GnRH agonists like leuprolide\n leads to a desensitization and
downregulation of these receptors resulting in a hypogonadal state
resembling menopause)\n decreases both tumor & uterus size
What are the major side effects of Dopamine?

HAT on the heart:\n

Hypertension\n Angina, Arrhythmias\n Tachycardia


What is the main feature of myasthenia gravis (MG)?

Fluctuating

weakness of certain voluntary muscles, particularly those innervated by


the motor nuclei of the brain stem\n Manifest during continued activity,
quick restoration of power with rest & dramatic improvement in stregnth
following admin of neostigmine
What does the activation of an adrenergic alpha1 receptor elicit?

Gq > increased DAG/IP3/Ca2+\n Increased TPR & BP (vasoconstriction


in A & V of superficial tissues)\n Decreased Urination (Urinary SM
contraction)\n Mydriasis without cycloplegia (contraction of the Radial m.
of the iris)\n Ejaculation (Ductus deferens contraction)
What defect often results in myelin sheath malformation? Defective long
chain fatty acid metabolism:\n Peroxisome disfunction (longchain fatty
acid catabolism, detox, & microbial killing)
What does the activation of an adrenergic alpha2 receptor elicit?
= decreased cAMP\n Reduced central sympathetic outflow (decrease

Gi

presynaptic NE release)\n Decreased intraocular pressure (decreased


aqueous humor secretion)
What is the dominant component of pulmonary surfactant?
Phosphatidylcholine (aka lecithin)\n Produced in adequate amounts
by the 36th week of gestation
What does the activation of an adrenergic beta1 receptor elicit? Gs =
increased cAMP\n increase HR, CO, & systolic BP (heart stimulation)\n
Increased ATII (increased renin release)
What measurement is used to estimate lung maturity?

Lecithin:Sphingomyelin ratio\n <2 = high rist for neonatal respiratory


distress
What does the activation of an adrenergic beta2 receptor elicit? Gs =
increased cAMP\n Reduced TPR & BP (skeletal muscle vasodilation)\n
Increased Airflow (Bronchodilation)\n Delay of labor (Uterine
relaxation)\n Increased plasma glucose & decreased K+
(Glycogenolysis/Gluconeogensis)
What is the genetic basis of Duchenne's muscular dystrophy?

Xlinked

disorder due to frameshift mutation\n Progressive muscle weakness &


early death due to accelerated rate of muscle breakdown
What does the activation of an dopaminergic D1 receptor elicit? Gs =
increase in cAMP\n Increased renal perfusion (Arterial dilation in the
renal, mesenteric & coronary beds)\n Decreased TPR & Bp
What is the genetic root of Becker's muscular dystrophy? Point mutation
in the dystrophin gene
What does the activation of an dopaminergic D2 receptor elicit? Gi =
decrease in cAMP\n Extrapyramidal motor activity
What drug can be used to treat IBS?

loperamide (an opioid antidiarrheal)

What adreneric receptor is the only one to result in decreased cAMP?


Alpha2: Gi = decreased cAMP
What is Bactrim?

Trimethoprimsulfamethoxazole

What causes a greenish vaginal discharge, friable cervix, & motile cells on
wet prep?

Trichomonas vaginalis\n\nRx: metronidazole

How does Haemophilus ducreyi present?

Gram \n Chancroid:

painful genital ulcer & inguinal lymphadenopathy


What does Staphylococcus saprophyticus cause in young adults? UTIs
What is the histological presentation of papillary thyroid carcinoma?

Fingerlike projections of epithelium surrounded by a centeral fibrovascular


core\n Psammoma bodies (calcified spheres)\n "Orphan Annie" nuclei &
nuclei molding\n\n Risk factor: prior history of radiation to the head &
neck
What HLA is found in patients with Hashimoto's?

HLADR5

What is the histological presentation of Hashimoto's?

Substantial

lymphocyte infiltration & germinal central formation\n Characteristic =


Hurthle cells (epithelial cells with eosinophilic granular cytoplasm)
How is interstitial (or idiopathic) pulmonary fibrosis (IPF) characterized?
Abnormal proliferation of mesenchymal cells, disruption of collagen
structures, & impaired gas exchange\n RIP within 5 years without lung
transplant\n Xray: white lines in a netlike pattern at the lower part of the
lungs
What is Lethal Factor?

Released by Bacillus anthracic \n Zinc

metalloprotease\n Inactivates protein kinase that stimulates


macrophages to release tumor necrosis factor alpha
What organism causes Q fever? Coxiella burnetti (a rickettsial org)\n
transmitted by aerosolized endospores\n Q fever: high fever, headache,
shaking malaise, myalgia\n Can cause a lifethreatening endocarditis
(esp, if Px has abnormal heart valves)
What causes hand, foot, & mouth disease?

Coxsackie A (RNA virus)

How is endemic typhus (caused by Rickettsia typhi) spread?


(oftn on rodents)\n Sign: Centrifugal rash

Fleas

What is the MOA of Albendazole?

Interrupts microtubule function\n

Rx: parasitic worms (e.g. hook & pin)


What is the pathophysiology of BPH (Benign Prostatic hyperplasia)

Estradiol level increases with age\n Sensitization of the prostate to the


effects of dihydrotestosterone (DHT)\n Porstatic cells grow
What drug for BPH also promotes hair growth?

Finasteride: 5alpha

reductase inhibitor that prevents further cellular growth


What commonlyused antifungal also has antiandrogen effects?
Ketoconazole used in polycycstic ovarian syndrome to prevent
hirsutism
What K+ sparing diuretic also has antiandrogen effects?

Spironolactone

used to prevent hirsutism in polycystic ovarian syndrome


What are the clinical signs of a splenic infarct due to an embolism?

Left upper quadrant abdominal pain\n Left shoulder pain (Kehr's sign)\n
Nausea\n Vomiting\n History of embolic disorder
Describe the source of blood to the stomach. Left Gastric (branch of the
celiac)\n Left gastroepiploic artery (branch of the splenic)\n Right Gastric
(branch of the common hepatic)\n Right gastroepiploic (branch of
gastroduodenal, a branch of celiac)

What anastomosis in cirrhosis results in hemorrhoids?

Superior rectal

& Middle Rectal\n Sign: bright red blood after a bowel moment
What anastomosis results in esophageal varices?

Left gastric &

Esophageal\n Painless hematemesis


What type of surgery predisposes patients to vitamin A deficiency?
Intestinal (esp. if ileum is involved) absorbtion requires micelles
formed with bile salts
What is required for normal iron absorption? Ferrous iron requires
apoferrin

Where in the GI tract is vitamin B12 absorbed?

Terminal ileum via

binding to intrinsic factor (secreted by gastric parietal cells)


What enzyme is most essential in activation of pancreatic enzymes in the
duodenum? Enterokinase: activates trypsinogen to trypsin\n This
enzyme is normally embedded in the intestinal mucosa
What are premalignant, hyperkeratoic areas with surrounding erythema
that often preceded the development of squamous cell carcinoma?
Actinic keratoses
What is acral lentiginous? Melanoma that often appears on the palms &
soles of darkskinned individuals
What skin cancer is described as "pearly papules with overlying
telangiectasias"?

Basal cell carcinoma

What lifestyle factor doubles the risk of peptic ulcers?

Tobacco use

What is a left shift in the hemoglobin curve? An increase in


hemoglobin's affinity for oxygen = less O2 unloading to tissue
What is a good way to remember what activity decreases O2 affinity to
hemoglobin? Exercise\n Increase in temperature\n Decrease in pH\n
Increase in CO2
What does 2,3DPG do to the hemoglobin curve?

Decreases its affinity

for O2 = right shift


What does methemoglobin do to the hemoglobin curve?

Has a

decreased affinity for O2 = right shift


Why are nitrates used to treat cyanide poisoning?

Nitrates can increase

the amount of Methemoglobin\n Methemoglobin: the iron in hemoglobin


is oxidized to Fe3+ & has an increased affinity for cyanide. \n Then this
can be treated with methylene blue
What is the normal vaginal pH? 3.8 4.5\n This is maintained by
Lactobacilli species that secrete lactic acid & hydrogen peroxide
At what pH is vulvovaginal candidiasis found?

Normal vaginal pH

range (<4.5)\n On Gram Stain: Hyphe & bastospores

At what pH is bacterial vaginosis found?

5.06.0 (moderately

elevated)\n Dx: Add KOH to vaginal discharge = strong fishy or amine


odor = WHIFF TEST
At what pH is a Trichomanos vaginalis infection found?
What is the pH of cervical mucus or sperm?

5.07.0

8.0

What is WaterhouseFriderichesen syndrome?

Sepsis with adrenal

insufficiency
What tumor can cause paroxysmal hypertension, palpitations, anxiety, &
drenching sweats? pheochromocytoma: tumor of the adrenal medulla that
leads to the overproduction of epinephrine
What presents with Thrombocytopenia, hemolytic anemia with
schistocytes, & uremia... & is a sequelae of a bacterial infection?

Hemolyticuremic syndrome\n Shigella or Escherichia coli O157:H7


infection
What type of drug is omeprazole?

PPI

What type of drug is Loratadine?

2nd generation, H1receptor

antagonist\n Rx: relieves allergy symptoms


What type of drug is Diphenhydramine?

1st generation H1receptor

antagonist \n Rx: symptomatic relief of allergy symptoms or as a sleep


aid
How do NSAIDS cause GI problems?

They are nonspecific inhibitors of

cycloxygenase enzymes\n T/F they reduces the production of


prostaglandin & thromboxane from arachidonic acid \n Prostaglandins
play an important role in preserving mucosal integrity
What V/Q scan value indicates a physiologic dead space? Approaches
infinity\n There is a obstruction to perfusion (like a spontaneous
pulmonary embolism) and perfusion is decreased
What value is found in a V/Q scan when there is a ventilatory obstuction to
the lung?

Approaches 0\n Airway obstruction leads to some

perfusion, but no ventilation

What is teh V/Q ratio at the apex of the lung? High\n Low blood flow &
only a slight decrease in ventilation
What is the classical presentation for CML?

Middleaged person\n

Weakness\n Weight Loss\n Left Upper Quadrant Pain\n\nLAB: increased


# of cells of myeloid lineage + t(9:22)
What is CML?

Stem cell disorder that results in the overproduction of

myeloid cells (mature forms)


What is the DOC for CML? Imatinib mesylate: inhibits the abnormal bcr
abl tyrosine kinase found in >90% of CML\n Not a cure but restores
normal blood cell counts & prevents progression to bast phase for several
years
What is infliximab? Chimeric monoclonal antibody to tumor necrosis
factor alpha\n Rx: refractory Crohn's disease
What is the MOA of Propylthiouracil?

Blocks thyroid hormone synthesis

by partially inhibiting peripheral deiodination of thyroxine to


triiodothyronine\n Rx: hyperthyroidism or thyroid storm
What is subacute sclerosing panencephalitis (SSPE)?

A sequela of

rubeola (measles virus)\n Presents 710 years after initial infection \n


Demyelination of the CNS\n 1st stage: dementia & personality changes (1
yr. duration)\n 2nd stage: Severe myoclonus (312 months)\n 3rd & 4th
stage: Worsening dementia & deterioration of the ANS + decorticate
rigidity or flaccidity\n FATAL
What causes mutifocal leukoencephalopathy & typically affects only
immunosuppressed people though reactivation of latent infection?

JC

virus:\n altered mental status\n motor deficits (hemiparesis or limb


ataxia)\n hemianopia
What is the symtomatic course of Rabies virus infection?

Months after

infection: convulsions, parasthesias, & difficulty swallowing solids/liquids


due to spasticity of pharynx muscles\n Cognitive effects: manic episodes
& altered mental status\n Death after flaccid paralysis sets in

What is the major problem with Rubella (or German measles) infection?
antenatal infection = CRS (congenital rubella syndrome)\n High
degree of fetal mortality\n Cardia & ophthalmologic defects\n RARE:
SSPElike syndrome
What symptoms indicate a manic phase?

3 of the following:\n1)

inflated selfesteem/grandiosity\n2) Decreased need for sleep\n3)


Increased rate of speech/talkativeness (pressured speech)\n4) Flight of
ideas/racing thoughts\n5) Distractibility\n6) Increased goaldirected
activity/psychomotor agitation\n7) Increase in pleasurable activities
without regard for consequences
What is unique about Bacteroides fragilis?

Gram but without lipid A

= NO ENDOTOXIN\n anaerobic, Rx: metronidazole or clindamycin or


surgical drainage for abscesses
What causes wollsorters' disease?

Bacillus anthracis (Gram +, aerobic,

sporeforming rods) inhalation


How does Candida albicans infections of the vagina present?

Vaginal

itching\n Cheeselike discharge\n Dx: 10% KOH prep = pseudohyphae


What anaerobe is a major colonizer of the colon?

Bacteroides

How does a Trichomonas (protozoa) infection of the vagina present?

Frothy, foulsmelling discharge\n Itching & burning on urination\n\n Dx:


Motile parasites on wet mount
What is Protamine sulfate used for?

Heparin OD\n heparin is negatively

charged and this positively charged molecule binds to it to reverse its


effects
Where is Alanine aminotransferases (ALT) found?

Liver = marks

hepatocyte damage
Where is Aspartate aminotransferase (AST) found? Heart, liver, skeletal
muscle
Where is creatinine kinase (CK) found? Inner mitochondrial membrane, on
myofibrils, & in the muscle cytoplasm \n Catalyzes the production of
ATP\n Types: MM (skeletal muscle), MB (cardiac muscle), BB (brain)

Where is Lactate dehydrogenase (LDH) found?

Many tissues\n

Used as a marker for liver disease, MI, & hemolysis, but is not specific
Laceration of what region can lead to postvaginal delivery incontinence in
the mother? Perineal body: convergence of several muscles of the
urogenital diaphragm (anterior to the anus)\n Bublospongiosus\n
External anal sphincter\n Perineal muscles
Damage to what muscle can lead to postpartum urinary incontinence?
Coccygeus or levator ani (together = pelvic diaphragm)
What CSF finding points to JC virus?

1) Myelin pasic protein = due to

demyelination\n2) Dx: CSF PCR for JC virus or Stereotactic biopsy


How does PML (Ptrogressive mutifocal leukoencephalopathy) present?
Multiple nonenhancing T2hyperintense lesions\n Histology: nuclear
inclusions in oligodendrocytes
How does CMV encephalitis show on a scan? Enhancing periventricular
white matter lesions in cortical & subependymal regions\n Histo: Giant
cells with eosinophilic inclusions in cytoplasm & nucleus
How does CNS lymphoma present on a scan 1(+) enhancing lesions
surounded by enema = mass effect\n PCR CSF for EBV
What is the most common cause of multiple cerebral mass lesions in HIV
infected patients? Space occupying lesions due to toxoplasmosis
infection\n Usually at corticomedullary junction\n Surrounded by edeam
= mass effect
What is the treatment for Toxoplasma infection?

Sulfadiazine &

Pyrimethamine\n Trimethoprim/Sulfamethoxazole
What is Hydrochlorothiazide used to treat?

Diuretic: inhibits NaCL

reabsorption @ distal tubule (blocks the NaCl symporter on lumina) &


reduces Ca excretion (Increases Na gradient = upregulates Na/Ca
antiporter & sends more Ca to interstitium & more Na into the cell)\n1)
Hypertension\n2) Chronic kidney stones (due to hypercalciuria)
How does Acetazolamide work to treat altitude sickness? 1) Can cause a
hyperchloremic metabolic acidosis \n2) The drop in pH results in increased

breathing drive and higher O2 in the body\n3) Result: Reverses the effects
of hypoxemia
What is the MOA of Amiloride?

PotassiumSparing Diuretic:\n Inhibits Na

reabsorption transportion @ the cortical collecting duct \n T/F reduces K


secretion at the same time
What is the MOA of Furosemide? Loop diuretic:\n Inhibits cotransport of
Na, K, & Cl in the thick ascending loop of Henle\n This reduces the
medullary concentration gradient & the lumen gains a positive potential\n
T/F divalent cations are not absorbed as much (including Ca)
What is the MOA of both cyclosporine & tacrolimus?
Immunosuppressants:\n Bind to intracellular proteins called
immunophilins (Cyclosporine = cyclophilins; Tacrolimus = FKbinding
proteins (FKBP12))\n These proteindrug complexes then interfere with
intracellular signaling events that occur after Tlymphocyte receptor
ligation\n T/F prevent T lymphocyte activation\n\nRx: Transplant
recepients to prevent rejections\nSE: nephrotoxicity
What is the MOA of Azathioprine?

Antimetabolite derivative of 6

mercapopurine\n Interferes with the metabolism & synthesis of nucleic


acids\n Toxic to proliferating lymphocytes\n\nRx: immunosuppression for
autoimmune disorders (e.g. glomerulonepritis, hemolytic anemia) or with
kidney transplants\nSE: Hyperglycemia
What is Abruptio placentae?

Premature separation of the placenta from

the uterus (despite normal location of implantation) due to rupture of


defective maternal vessels in the decidua basalis\n Risk factors:
hypertension, cocaine use, short umbilical cord, trama, cigarette smoking,
uterine fiboids, advanced age, sudden uterine decompression, premature
membrane rupture, bleeding diathesis
How does Abruptio placentae present? Bleeding with uterine contractions
What is a concealed abruption (of the placenta)?

An abruptio

placentae that occurs near the center of the placenta\n Hematoma is


hidden behind the placenta & there is no external bleeding

What is Placenta accretia?

An abnormally strong adherence of the

placenta to uterine weall\n Placental villi attach directly to the


myometrium due to a defect in the decidua basalis layer
How does Placenta accretia present & what are the risk factors??
Incomplete separation of the placenta after delivery > severe postpartum
hemorrhage\n RF: placenta previa, prior cesarean, prior intrauterine
manipulation or surgery
What is Placenta previa? When the placenta overlies the internal
cervical os \n This condition is painless
What is the action of NA+K+ATPase pump inhibitors (like Digoxin) on
vascular smooth muscle & the kidney? 1) SM: increase of intracellular [Na]
= blocked Na+Ca2+ exchange & an increase in vascular resistance &
Bp\n2) Kidney: impairment of tubular Na resorption
What happens if a fetus in homozygous for an SOX9 mutation and the
male gonade does not form testes?

The individual will have no Sertoli

cells (no MIS & no 17bestradiol) & no Leydig cells (no testosterone):\n
female internal genitalia: persistence of the paramesonephric ducts b/c
no mesonephric duct stimulation via testosterone & no MIS\n female
external genitalia: no DHT production
What do plasma cells look like? Offcenter nuclei\n Clockface
chromatin distribution
What are the clinical signs of epiglottis?

High

fever\ndysphagia\ndrooling\ninspiratory stridor\nrespiratory distress\n\nX


ray: thumbprint sign (epiglottis thickening & aryepiglotic folds\n\nCause:
Hemophilus influenzae
What virus causes croup? Parainfluenze\n\nXray: steeple sign: narrowing
of the upper trachea
What presents with an inspiratory stridor, seallike barking cough,
retractions, & coryza?

Parainfluenza virus = CROUP

What does RSV cause?

Bronchiolitis: gradually developing respiratory

distress & paroxysmal wheezing

What is the formula for Clearance of Inulin (aka: GFR)?

Cl = (Urine

inulin)(Urine volume) / (Plasma inulin)


What is the DOC for Entaerobius vermicularis infection

Pinworm:

transmitted in food contaminated with eggs > intetinal infection & anal
pruritus\n Dx: tape on anus to catch noctural worms\n DOC:
mebendazole
What are Benznidazole & nifurtimox used to treat? Trypanosoma cruzi
(Chaga's disease)\n South/Central America
What is ivermectin used to treat?

Onchocerca volvulus infection

(causes River blindness)


What is Niridazole used to treat?Dracunculus medinensis (causes skin
ulceration & inflammation)
What is measure in the urease breath test?

13Clabeled CO2\n Px

ingests 13Clabeled urea, which is hydrolyzed into 2 ammonia molecules


& 1 CO2 by H. pylori
What is the DOC class for costochondritis or pericarditis? NSAIDs
What are condyloma acuminata?

Multiple, white, hyperkeratoic

peduculated verrucase clusterd in the perianal region\n Caused by HPV


type 6&11
What causes chancroid? Haemophilus ducreyi\n Painful genital ulcers
What 4 problems can HSV1 cause?

gingivostomatitis\n

keratoconjunctivitis\n temporal lobe encephalitis\n herpes labialis


What does HSV2 cause? Herpes genitalis & neonatal herpes\n 1st lesion
appears as ULCERS
What HPV causes genital flat warts & is associated with cervical dysplasia
& carcinoma?

HPV types 16 & 18

In what age group are fibroids found? <40 years old & can regress after
menopaus\n ~75% of these are asymptomatic

What is the function of vonWillebrand factor? It is a protein that binds


platelets to the endothelium during clot formation\n Carries factor VIII
How do symptomatic vWD patients present? Easy bruising or
bleeding\n Prolonged bleeding\n Menorrhagia (heavy menses)
What are the most common fungal causes of pneumonia?
Blastomyces\nCoccidioides\nHistoplasma
How is Crytococcus neoformans visualized?

India ink stain\n\nReservoir:

pigeons
What conditions show Mallory bodies on histological exam of liver tissue?
Intracytoplasmic hyaline inclusions derived from cytokeratin
intermediate filaments:\n Primary biliary cirrhosis\n Wilson's \n Chronic
cholestatic syndromes\n Hepatocellular tumors\n Alcoholic Cirrhosis
How does Cocaineinduced hepatitis present?

Clinical: Acute

subfulminant hepatitis\n\nHisto: necrosis & microvesicular steatosis


What enzyme is deficient in TaySachs & how does the disease present?
Hexosaminidase A\n Loss of motor skills, increased startle
reaction, macular pallor, CHERRYRED spot on Macula\n Dx: quantify
hexosamidase level in isolated WBCs from flood\n 1/25 carrier rate of this
AR disease in Ashkenazi Jewish population
What does a deficiency in Arylsulfatase A result in? Metachromatic
leukodystrophy (AR):\n Central & peripheral demyelination\n Ataxia\n
Dementia
What is the pathogenesis of EhlersDanlos syndrome?

Reduced

hydroxylysine in collagen\n T/F the content of collagen crosslinks\n


Results in decreased tensile strength of collagen
In what 2 syndromes is Wilms' Tumor a member?

WAGR\n Beckwith

Wiedemann
List the components of the WAGR syndrome? Wilms' Tumor\n Aniridia
(partial or complete lack of the iris)\n Genital anomalies (gonadal
dysgenesis, hypospadias, crytochidism...)\n Mental Retardation

What are the typical findings in VHL (Von HippelLindau) syndrome?

hemangioblastomas (when in brain: primary brain tumor with foamy cells


& high vascularity) in cerebellum/retina/medulla\n Increase in epo >
2ndary polycythemia\n 59% of Px develop bilateral renal cell
carcinoma\n Mean age of diagnosis = 29 years old (Very rare genetic
disorder)
What 2 neurotransmitters does Monoamine oxidase (MAO) decrage?
Norepinephrine\nSerotonin
What 4 diseases are associated with an imbalance of ACh & Dopamine?
Decreased Ach:\n Alzheimers \n Huntington's\nIncreased
Dopamine:\n Schizophrenia\nDecreased Dopamine:\n Parkinson's
Which T cells (type 1 or 2) are involved in a type IV hypersensitivity
reaction?

Type 1 (Th1)

What type of Thelper cells (type 1 or 2) are involved in mediating an


antibody response?Type 2 (Th2)
What diagnostic test results indicated multiple sclerosis? 1) MRI: multiple
white matter plaques\n2) CSF: IgG oligoclonal bands\n\nRx: beta
interferon (from connective tissue fibroblasts)
What diseases is alphainterferon used to treat?

Hep B&C\n Kaposi's

sarcoma\n Malignant melanoma\n Certain leukemias


What is yinterferon used to treat?

Chronic graulomatous disease\n

Produced by T lymphocytes & activated tumoricidal macrophages


What is interleukin11 (aka oprelvekin) used to treat?
Thrombocytopenia: stimulates megakaryocyte & platelet growth
Why is Sildenafil contraindicated in patients taking nitrates?

Increased

risk of priapism & hypotensive shock\n Phosphodiesterase inhibitiors


promote higher levels of cGMP & smooth muscle relaxation\n Nitrates
vasodilated by releasing nitric oxide in smooth muscle, which also raises
cGMP levels
Why are ACE inhibitors and NSAIDs a bad idea for the kidney?

ACE =

decreased efferent arteriolar tone & decreased glomerular capillary

perfusion pressuer\nNSAIDS = similar mechanism\n\nPotential for ARF


(due to ATN) if there is a drop in blood volume for any reason
What loops are used to treat congestive heart failure?

Loops like

furosemide to treat the edema


What neuro exam finding indicates sensorineural hearing loss?

Equal

bone and air conduction \n\nCause: acoustic neuroma (schwannoma)


compressing CNVIII where it enters the internal auditory meatus
What neuro exam finding indicates a conductive hearing loss?

Bone is

more effective than air conduction through the blocked ear canal (e.g.
cerumen) or sclerosis of the ear ossicles or perforated membrane
What causes TTP (thrombocytopenic purpura)?

Defect in ADAMTS

13, metalloprotease respoonsible for cleaving highmolecularweight,


large vonWillebrand factor multimers\n T/F platelets aggregate in the
microcirculation & schistocytes are produced \n Rx: plasma exchange
What causes PNH (paroxysmal nocturnal hemoglobinuria)?

Mutated

PIGA gene\n Abnormal synthesis of surface phospholipids that anchor


inhibitors of complement activation on all cell lines (WBCs, RBCs,
platelets)\n Deficiency of DAF (decayaccelerating factor) in the
compliment cascade
What enzyme in heme synthesis can be inhibited by heavy metal
poisoning?

Aminolevulinic acid dehydratase\n Elevated aminolevulinic

acid & anemia


What is the action on heme synthesis in lead poisoning?

1) inhibition of

aminolevulinic acid dehydratase\n2) Denatures ferrochelatase\n3)


Denatures ribonuclease in RBCs (basophilic stippline due to ribosomes
NOT breaking down)
What is the initial DOC for generalized anxiety disorder?
Benxodiazepines
What causes the bradykinesia in Parkinson's There is excessive inhibition
of the ventral lateral nucleus of the thalamus\n Loss of dopaminergic
neurons in the substantia nigra leads to a lack of inhibition of the globus

pallidus, which would lift the inhibition of the thalamus.\n\nRx: levadopa or


ablation of the internal segment of the globus pallidus (pallidotomy) in
refractory cases
What does a positive leukocyte esterase & nitrite on urine dipstick, +
fever, indicate?

UTI

What structures are the remnants of the umbilical arteries?

Medical

umbilical ligaments
What causes a urachal fistula?

When the allantois fails to regress;

predisposes to a UTI
What embryological structure is the nucleus puposus a remnant of?
notochord
In what phase of the cell cycle does Bleomycin act? G2: causes stand
breaks in DNA via intercalation & free radical damage
In what phase of the cell cycle does Etoposide act? late S & G2: inhibits
topoisomerase II (normally this enzyme untangles chromosomes and
separates replicated DNA strands as well as DNA repair)
What is the MOA of 5Fluorouracil?

Sphasespecific antimetabolite:

\n Inhibits thymidylate synthase\n T/F blocks the production of thymidine


(no pyrimidines)
When are beta waves seen on EEG?

When someone is awake &

concentrating
When are theta waves seen on an EEG?

Stage 1 sleep: transition

from awake to sleep\n 47 cycles per second


When are sleep spindles and Kcomplexes seen on EEG

Stage 2 sleep:

physiologic sleep\n 45% of sleep


What results in delta waves of EEG

Stage 3 or deep sleep\n Slow

waves that are < 3 Hz\n 12% of total sleep


What 2 sleep stages have been merged?
sleep

3 & 4 = N3\n 13% of total

What problems occur in stage 4 (delta wave) sleep?


enuresis\nsleepwalking\nnight terrors
What are the branches of the RCA in a "rightdominate" person? SA nodal
branch\nAcute marginal artery\nAV nodal branch\nPosterior descending
artery
What does the LAD artery (a branch of the left main coronary artery)
supply?

Anterior wall of both ventricles \n Anterior 2/3 of the

interventricular septum \n AV bundle


In 80% of people, what artery supplies the AV node?

a branch of the

RCA\n\n20% = branch of the left main coronary artery


What arteries supply the RV?

2 branches of the RCA:\n Acute

marginal\n Posterior descending


What organisms are most often associated with GuillainBarre Syndrome
(GBS)?

1) Campylobacter jejuni\n2) Haemophilus influenzae\n3)

Cytomegalovirus\n4) EBV\n5) Mycoplasma pneumoniae\n6) Varicella


zoster Virus
What is albuminocytologic dissociation?

CSF: increased protein

concentration with normal cell count in the setting of normal glucose\n\n


Indicates GBS (GuillainBarre Syndrome)
What type of inheritance pattern does Leber's hereditary optic neuropathy
have? Mitochondrial:\n mother to child only, but not all children of an
affected mother will be affected.\n Ratio of normal to abnormal
mitochondria inherited will determine whether the disease manifests itself
phenotypically.
What is familial hypophosphatemic rickets?

Xlinked dominant bone

disease:\n Defect in the resorption of phosphate by the proximal renal


tubule \n Abnormal regulation of 25(OH) vitamin D activation
What 2 lysosomal storage diseases are Xlinked recessive (all others are
autosomal recessive)?

1) Fabry's\n2) Hunter's

What causes a child to present with anorexia, hyperirritability, altered


sleep patterns, & decreased play?

LEAD poisoning

What are the 1st signs of lead poisoning? How do these symptoms then
progress?

LEADinduced porphyria:\n1) Loss of developmental

milestones (esp. speech), abdominal complains (intermittent vomiting,


pain, & constipation)\n2) Ataxia, altered state of consciousness, coma,
seizures, encephalopathy\n3) Permanent consequences: learning &
cognitive deficits, aggressive behavior
From a clinical perspective, what 3 substances do lysosomes contain that
are deficient in certain genetic diseases?

Enzymes for destruction

of:\n1) Cellular Mucopolysaccharides\n2) Sphingolipids\n3)


Mucopolysaccharides\n\nLack of these enzymes: lysosomal storage
diseases
How do Gaucher's patients present?

betaglucocerebrosidase lacking

(AR):\n1) Organomegaly (esp. spleen & reticuloendothelial organs)\n2)


Bone pain, Anemia, Thrombocytopenia (bone marrow infiltration)\n3)
fatigue\n\nDx: Radiograph changes: 50% have abnormal cortex of distal
femur = "Erlenmeyer flask"
What is the mechanism of acetaminophen toxicity? Glutathione stores are
depleted & toxic metabolites accumulate in the liver
What does a patient deficient in Glucose6phosphatase (final step in
bluconeogenesis to produce glucose from glucose6phosphate) have?
vonGierke's disease\n Abnormal accumulation of glycogen within
liver cells
What does a young child with decreased eye contact, increased startle,
macrocephaly, seizures, ataxia, & cherryred spots on the macula have?
TaySachs: deficiency of hexosaminidase A & subsequent
accumulation of ganglioside within cells
What type of virus is EBV?

Herpes: doublestraded, linear, DNA

genome
What viruses have doublestranded, circular DNA?
Papillomaviruses\nPolyomaviruses

What type of genome do the following viruses


have?\nPicornaviruses\nCalciviruses\nFlaviviruses\nTogaviruses\nCoronavir
uses\nRetroviruses Singlestranded (+), linear, RNA
What 4 virus families are singlestranded (), linear RNA? 1)
Orthomyxoviruses\n2) Paramyxoviruses\n3) Rhabdoviruses\n4) Filviruses
What viral families have singlestranded, circular RNA?
Arenaviruses\nBunyaviruses\nDeltaviruses
What causes a newborn to present 27 days after birth with GI,
intracranial, umbilical, mucosal, circumcisional, &/or cutaneous bleeding?
Hemorrhagic disease of the newborn caused by vitamin K
deficiency\n\nLab: elevated PT & PTT with normal platelet count &
bleeding time
What is the hallmark of hemophilia?

Spontaneous hemarthrosis

(bleeding into joints) caused by\n Deficiency of factor VIII (hemophilia A)


or factor IX (hemophilia B)
What are 2 qualitative platelet disorders that present with petechiae &
purpura shortly after birth?

BernardSoulier\n Glanzmann's

throbasthenia
What do Pheochromacytomas produce?

Norepinephrine &

Epinephrine \n These are tumors of neural crestderived tissue\n\nRx:


alphablockage (DOC: phenoxybenzamine, longlasting, noncompetitive,
irreversible alphablockers)
What testicular cells secrete mullerianinhibiting factor, & thus causes
mullerian ducts to regress?

Sertori cells

Where are Leydig cells located? in the interstitial spaces between the
seminiferous tubules?
What viral infection presents after a fever with a facial rash (bright red,
blanchable, on the cheeks with perioral pallor) followed by a diffuse trunk
& extremities rash that waxes and wanes with temperature over 3 weeks
Parvovirus B19 (5th disease)\n\nRx: nothing, selflimiting

What is the triad for Meigs' syndrome? 1) Benign ovarian tumor


(fibroma)\n2) Ascites: hypoactive bowel sounds & some abdominal
guarding on deep palpation, fluid wave\n3) hydrothorax (pleural effusion):
decreased breath sounds
What is another name for a benign cystic teratoma of the ovary? dermoid
cyst (3 germ cell layers)
What is a Struma ovarii? A teratoma in which thyroid tissue has
overgrown\n presents with symptoms of hyperthyroidism
What is the 1st line treatment for pyoderma gangrenosum (PG)?
Extraintestinal manifestations of IBD (inflammatory bowel
disease)\n Rx: Oral corticosteroids
What is the DOC to treat cellulitis?

Ampicillin (betalactam antibiotic)

What type of drug is Bacitracin? Topical antibiotic: prevents isoprenyl


dephosphorylation & t/f bacterial wall synthesis
What are the stage 1 & stage 2 symptoms of Reye's syndrome of aspirin
tox?

1) vomiting, confusion, lethargy\n2) stupor, hyperventilation,

microvesicular fatty liver


What causes type 1 dyslipidemia?

Deficiency of lipoprotein lipase (in

capillary walls of adipose & muscle tissues)\n Normally this enzyme


cleaves triglycerides into free fatty acids & glycerol\n No activation of this
enzyme by VLDL cholesterol (apolipoprotein CII) or chylomicrons
What results from an apolipoprotein E receptor deficiency?
dysbetalipoproteinemia: VLDL cholesterol remnants are not
removed from the circulation:\n Elevated VLDL cholesterol, triglyceride, &
cholesterol\n Xanthomas
What is familiarl hyperbetalipoproteinemia? Type II hyperlipidemia (LDL
cholesterol receptor dysfunction):\n Elevated plasma LDL & t/f increased
in plasma cholesterol
What results in extremely high triglyceride levels & visibly foamy plasma?
Type V: mixed hypertriglyceridemia \n Elevated VLDL cholesterol
levels

How does bronchiectasis present on CT?

1) "tram track" appearance

of bronchi\n2) Enlarged bronchiole with thickened wall\n3) "Signetring"


sign: diameter of the airway lumen greater than adjacent
vessel\n\nClinical: large amounts of purulent sputeum due to repeated
infection
How does TB present on CT?

1) Primary: Ghon complex = hilar lymph

nodes & subpleural parenchymal lesion\n2) Secondary: lesions in the


apical or posterior segments of the upper lobe +/ cavitary lesion
How does pneumonia present on CT? 1) Diffuse opacities of the lung
parenchyma\n2) Dense consolidations\n3) Ground glass opacities
What causes Meckel's diverticulum?

A persistence of part of the

omphalomesenteric duct (vitelline duct or yok stalk)\n\nDx: 99mTc


pertechnetate radionuclide imaging b/c the diverticulum can contain
gastric or pancreatic tissue & it will absorb this dye.
What differs in presentation between an infant with intussusception &
Meckel's diverticulum?

Meckel's = mild (blood in stool, & mild

tenderness)\n\nIntussusception = abrupt & severe presentation


(paroxysmal bouts of screaming, vomiting, diarrhea, & bloody bowel
movements w/in 24 hours of onset)
What is the term for abnormal or incomplete rotation of the intestine?
volvulus = twisting of loops of bowel\n\nPresentation: bile
containing vomit or bowel obstruction
What does PT measure? PTT?

PTT: VIII, IX, XI, XII\nPT: VII\n\nBoth: X, V,

prothrombin, & fibriniogen


What are the most common paraneoplastic syndromes of a thymoma? 1)
MG = myasthenia gravis\n2) pure RBC aplasia\n3)
Hypogammaglobulinemia
What 5HT(1B/1D) receptor agnoist is used to treat acute migraine &
cluster headaches? Sumatriptan: MOA = vasoconstriction of cerebral &
meningeal vessels\n\nSE: (rare) coronary vasoconstriction

What 5HT3 receptor antagonist is used to treat the nausea & vomiting
associated with chemo?

Ondansetron

How does exercise cause an increase in SV? Is there an increase in EDV or


ESV? Exercise increases preload (EDV)\n So,ESV is decreased to
increase the cardiac ejection fraction (EDVESV)
What are the clinical signs of Eisenmenger's syndrome?

Late cyonosis

(reversal of initial LtoR shunt)\n Polycythemia\n Clubbing\n Pulmonary


regurgitation (sometimes)\n Holosystolic murmur
What causes a selflimited localized infection consisting of
nonerythematous, pearly, domeshaped papules on the skin of children or
immunosuppressed patients?

Molluscum contagiosum (a poxvirus)

What is the appearance of Papillomavirus warts?

Either: flat, raised or

resemble a cauliflower
What happens when a polyomavirus is reactivated? Progressive multifocal
leukoencephalopathy (in immunosuppressed patients)
How does Cushing's result in proximal limb weakness?

Selective

atrophy of fasttwitch (type 2) myofibers


What is the only adrenergic receptor that acts through a Gq mechanism (&
increased DAG/IP3/Ca2+) Alpha1
What type of drug is Fenaldopam?

direct sympathomimetic, selective

D1agonist
What is the MOA of Fenoldopam?

"one FELLOW of DOPAMine fires the

D1receptor"\n\nGs > increased cAMP > PKA > MLCKPi > smooth
muscle relaxation\n\nVasodilation:\n Renal, mesenteric, & coronary
vascular beds\n decreases arterial Bp \n reflex sympathetic activation
What are the 2 clinical uses of Fenoldopam? 1) Hypertensive emergencies
= IV, shortterm Tx\n2) Postoperative hypertension
What are the major side effects of Fenoldopam?

The FELLOW has FAITH

in D1receptors:\n1) Flushing\n2) Angina (rare)\n3) Increased IOP

(FENOLdopam may FILL the eye)\n4) Tachycardia (reflex)\n5)


Hypotension/Headache
What drugs are used in hypertensive emergencies? NFL ENAbled for a fast
touchdown:\n1) Nitroprusside & Nitroglycerin (NO providers)\n2)
Nicardipine (CCB)\n3) Fenoldopam (D1agonist)\n4) Labetalol (apha1, B
agonist)\n5) ENAlaprilat (ACE inhibitor)
What sympatholytic drug is a synthesis inhibitor?

Metyrosine

What sympatholytic drug is a false trasmitter?


Guanadrel\nGuanethidine
What sympatholytic drug is a storage blocker?

Reserpine

What sympatholytic drug is a release inhibitor?

Guanethidine

What sympatholytic drugs are selective alpha2 agonists?


Clonidine\nalpha
Methyldopa\nGuanabenz\nGuanfacine\nApraclonine\nBrimonidine
What sympatholytic drug is a selective alpha1 blocker?

azosin\n\nPrazosine, Doxzaosin, Terazosin\n\n osin\n\nTamsulosin


What sympatholytic drug is a nonselective alpha blocker?
Phen__amine:\nPhentolamine\nPhenoxybenzamine
What sympatholytic drug is a selective betablocker?

AM + olol: (A

BEAM)\n1) Acebutolol (also has intrinsic sympathomimetic activity)\n2)


Atenolol\n3) Betaxolol\n4) Bisoprolol\n5) Esmolol\n6) Metoprolol
What sympatholytic drug is a nonselective betablocker? MZ + olol:\n1)
Nadolol\n2) Propranolol\n3) Penbutolol (also has intrinsic
sympathomimetic activity)\n4) Pindolol (also has intrinsic
sympathomimetic activity)\n5) Timolol
What sympatholytic drug is a mixed alpha1, beta blocker??

1)

Labetalol (also has intrinsic sympathomimetic activity)\n2) Carvedilol


What type of drug is reserpine? indirect sympatholytic, storage blocker

What is the MOA of Reserpine? this SERPENT lets CA RESERve PINE away
by blocking vesicular uptake pumps:\n\nCA depletion: \n irreversibly
inhibits Mg2+, ATPdependent CA uptake pumps on storage vesicles, then
MAO breaks down extravesicular CA\n depletes CA store in
central/peripheral nerve terminals & in the Adrenal Medulla\n\n Lowers
Bp: decreased CO & PVR\n Striatum: Dopamine depletion suppresses
extrapyramidal motor activity
What are the 2 clinical uses of Reserpine?

1) Hypertension (3rd line

choice for mild/moderate cases)\n2) Huntington's (relieves dyskinesia)


What are the major side effects of Reserpine?Such a RESERVE depletion
makes me 2SAD:\n\nCNS: Sedation, pArkinsonism, Depression\n\nGI:
Secretion of gastric acid, Abdominal pain, Diarrhea
What type of drug is Clonidine? Indirect sympatholytic, selective alpha2
agonist
What is the MOA of Clonidine?

this CLOWN's CLONe ID is INE: Inhibiting

NE exocytosis by igniting presynaptic alpha2receptor\n\nGi > decrease


in cAMP > decrease PKA > decrease Ca2+ influx\n\nCentral depressor:
decreases CO & PVR to lower Bp (DOES NOT reduced renal blood flood),
initially vasoconstriction due to possynaptic alpha2 activation \n\nReduces
GI motility / Increases fluid absorption / decreases secretion
What are the 4 clinical uses of Clonidine?

the CLOWN needs to HOLD

the lighter:\n1) Hypertension (2nd choice) & hot flashes (due to


menopause)\n2) Opiod & Alcohol withdrawal\n3) Loss of behavioral control
(ADHD)\n4) Diabetic Diarrhea due to sympathetic neuropathy
What are the major side effects of Clonidine? SAD:\nSedation\nAllergic
dermatitis\nDry mouth & Depression\n\nWITHDRAWEL syndrome:
hypertensive crisis
What drug name ending indicates a selective alpha2 agnoists

onidine:\n\nClonidine\nApraclonidine\nBrimonidine
What selective alpha2 agonists are used to treat openangle glaucoma?
Apraclonidine & Brimonidine

What selective alpha2 agonist is used as a central acting skeletal muscle


relaxant?

Tizanidine

What drugs are preferred for treating hypertension in pregnancy?


About the ABC of Water. Yes, the fetus needs water to survive:\n
Alphamethyldopa (methyldopa for materal use!)\n Beta blockers,
especially labetalol\n CCBs\n Hydralazine (IV for eclampsia)
What drugs are contraindicated for treating hypertension in pregnancy?
Can't be ADded due to the risk of endangering fetus by losing
water:\nACEIs (fetopathy risk)\nARBS (fetopathy risk)\nDiuretics (use with
caution)
What type of drug is Pazosin?

adrenergic blocker, selective alpha1

blocker
What is the MOA of Prazosin?

PRessured Alpha 1 ZOne Selectively

INcised:\n Vasodilation (blocks Gq > decreased IP3/Ca2+) in veins &


arterioles. NO significant sympathetic activation\n Urinary sm. relaxation
@ base of the bladder, prostate, & urethra\n Reduces plasma lipids
What are the 2 clinical uses of Prazosin?

1) Hypertension (2nd choice

or 1st if patient has BPH)\n2) BPH: symptomatic relief


What are the major side effects of Prazosin? prazoSIN:\n Syncopy (1st
dose) due to marked orthostatic hypotension\n Increased retention of Na
& H2O\n Nonspecific dizziness & headache
What drug ending indicates a selective alpha1 blocker?

azosin:\nPrazosin\nDeoxazosin\nTerazosin\n\nuz(l)osin: BPH drugs


(Urinary alpha1 ZOne Selectively Incised\nAlfuzosin\nTamsulosin
What is the clinical use of AlfUzosin?

BPH: preferentially acts on the

lower Urinary tract, Alpha1 blocker


What is the clinical use of TamsUlosin? BPH\n\n"FLow of urine will reach its
MAX once TAM (Tight, Alpha1A dominat Muscle in the lower UI) is SO
LOOSe IN tone"\n blocks alpha1A receptors in the prostate\n SE:
Ejaculation disorders
What type of drug are Pentolamine & phenoxybenzamine? bleh

Are hyperuricemia kidney stones radiolucent or radioopaque?


Radiolucent: not seen on Xray
Are struvite calculi (ammonium, magnesium, & phosphate) radiolucent or
radiopaque? 80% radiopaque\n\nCause: Proteus vulgaris or
Staphylococcus saprophyticus
Where is lactate dehydrogenase located within the cell?

Cytoplasm

What are the main functions of rough endoplasmic reticulum?

Synthesis of secretory proteins\n Nlinked oligosaccharide addition to


proteins
What amino acids are modified in the Golgi? 1) Asparagine: N
oligosaccharide modification\n2) Serine & Threonine: addition of O
oligosaccharides
What can happen to sugars, proteins, & lysosomal proteins in the golgi?
1) Sulfonation of sugars in proteoglycans & tyrosine residues on
proteins\n2) Addition of mannose6phosphate to specific lysosomal
proteins to target them to the lysosome
Which develops quickly, vitamen B12 or folate deficiency? Folate: deficient
intake (diet), increased need (e.g. pregnancy), or impaired use
(antimetabolate drugs)
Why do vitamin B12 & folate deficiency cause megaloblastic anemia?
DNA synthesis is impaired & with fewer cell divisions & more
cytoplasm is present = increase MCV & hypersegmented neutrophils
What causes hypochromic, microcytic RBCs? iron deficient anemia: low
MCV \n+ low serum iron & ferritin & high TIBC
What type of molecule is "ras"? G protein that cycle between 2
conformations:\n1) Activated rasGTP \n2) Inactivated ras
GDP\n\nMutation = oncogenic GTP binding with a loss of GTPase activity
( = permanently turned on)
What type of molecules are the oncogenes "erb" & "HER2"?
Epidermal growth factor receptor encoding genes

What type of protooncogene is "abl"? Tyrosine kinase activity.\n In CML,


is translocated from Ch. 9 to 22 & fuses with bcr
What type of molecule is "myc"?Protein transcriptional activator that binds
DNA & activates many growthrelated genes
What is the progression of CMV retinitis (usually only found in AIDS
patients)?

1) Rapidly diminishing sight (loss of central vision with

floaters & blind sptos)\n2) Funduscopic exam: cottonwool exudates >


necrotizing retinitis > perivascular hemorrhages > (lastly) retinal
detachment\n\nRx: ganciclovir & foscarnet
What is the mechanism of the cherry0red spot on the macula in TaySachs
& NiemannPick disease? 1) Ganglion cells filled with gangliosides (Tay
Sachs) or spingolipids (NiemannPick) are located outside the fovea &
block the orangered color of the choroid\n2) Absence of ganglion cells
within the fovea allow the choroid's normal color to be seen in this area
(CHERRYRED)
How do microaneurysms show up on fundoscopic examination? Tiny,
round, red spots arougd the macular area\n Cause: diabetic retinopathy
Why does diabetic retinopathy result in neovascularization?

Abnormal

proliferation is due to impaired autoregulation of retinal blood flow


What is papilledema?

Edema of the optic disc (often due to increased

intracranial pressure)
What shunts blood from the left pulmonary artery to the aorta?

Ductus

arteriosus: allows blood to avoid the highresistance lungs & deliver more
O2 blood to the rest of the body
What are the 2 most common cause of a failed closure of the ductus
arteriosus?

1) Prematurity\n2) Maternal rubella infection during

pregnancy\n\nRx: indomethacin (blocks PGE1 & can be used to close a


PDA)
What is the most common cause of Conn's Syndrome?

Adrenal

adenoma: increased aldosterone \n\nHisto: lipidladen clear cells in a well


circumscribed adenoma

What is the drainage of the adrenal glands? RIGHT: right adrenal vein
into the IVC\n LEFT: left adreanl vein into the right adrenal vein into the
IVC
What are "B symptoms"? Night sweats, fever, & weight loss\n\nHodgkin's
lymphoma symptoms with enlarged mediastinal lymphadenopathy\n 50%
of patients have EBV
What is 1st line treatment for prostatitis & UTI?

TMPSMX: inhibits

dihydrofolate reductase (TMP) & dihydropteroate synthases (SMX) \n\n2nd


choice: fluroquinolones
What does a midshaft fracture of the humerus cause injury to?

Radial

groove structures: radial nerve & deep brachial artery\n "Great Extensor
Nerve": wrist drop (inability to extend wrist & matacarpophalangeal joints
of all digits)
What are the liver enzyme levels in alcoholic hepatitis?

AST:ALT ratio

>1.5 & normal alkaline phosphatase


What is bacterial TRANSFORMATION?

Gene TRANSFER resulting from the

uptake of DNA from a doner, often when a bacteria dies & surviving bac
pick up the residual DNA.
What is bacterial CONJUGATION?

Gene transfer from a donor to a

recipient by direct PHYSICAL contact between cells


What is bacterial TRANSDUCTION?

Gene TRANSFER from a donor to a

recipient by way of a bacteriophage


What are "transposable elements"?

Segments of DNA that are able to

move from 1 location to another


What is the Lab Value hallmark of PCOS?

LH hypersecretion:\n

Obesity\n Hirsutism\n Oligo or amenorrhea\n Acanthosis nigricans


(velvety hyperpigmentation)\n\n Hyperglycemia, hyperlipidemia, insulin
resistance
Why do Mg levels rise with renal failure?

The only regulatory method

of Mg is through renal exretion\n\nMg levels of 24 mEq/L = vomiting,

nausea, lightheadedness\n\nMg levels greater than 4 = depressed


consciousness, respiratory depression & cardiac arrest
What are negatively birefringent crystals?

yellow & parallel\n

monosodium urate\n\nIn aspirated joint fluid of a patient with gout


What are symptoms of Reye's syndrome?

Abrupt onset of nausea &

vomiting\nFollowed by encephalopathy & jaundice\n\nCause: fatty


infiltration of liver b/c of impaired mitochondrial metabolism of fatty acids
due to ASPRIN administration to a CHILD
How is DubinJohnson syndrome characterized?

AR: defective

hepatobiliary transport of certain organic anions\n Conjugated


hyperbilirubinemia\n BENIGN
What H2blocker is associated with antiandrogenic effects? Which is NOT?
Cimetidine: gynecomastia & impotence\nRanitidine: no such SE
Which antacid can buffer gastric acid? Magnesium oxide
What 3 conditions can Omeprazole be used to treat?

PPI: \n GERD\n

ZollingerEllison syndrome\n Peptic Ulcer disease


What are the 2 problems that Spironolactone is used to treat?

MOA:

aldosterone receptor antagonist\n Hyperaldosteronism\n Edematous


states (heart/liver/kidney failure)
What fractional excretion of Na indicates intrarenal renal failure? Between
2 & 4%
What 3 drug classes are associated with allergic reactions?

1)

Penicillins (esp: Methicillin & Nafcillin)\n2) Cephalosporins\n3)


Sulfonamides\n\nS/S: fever, rash, eosinophilia
What is the pathological process of interstitial nephritis?
Allergic/Infective/Immunological disorder: inflammatory process that
results in infiltration of the interstitum of the kidney with
polymorphonuclear leukocytes & lymphocytes
What drug directly inhibits testosterone biosynthesis?
(antifungal)

Ketoconzaole

What are the main causes of aplastic anemia?

1) Carbamazepine

(antiseizure drug)\n2) EBV, HEP C, Parvovirus B19\n3) Toxins (benzene &


insecticides)\n4) Cancer chemotherapeutics\n\nHypocellular bone marow
W/O abnormal cells
What are the 3 causes of Eisenmenger's syndrome?
VSD\nASD\nPDA\n\nS/S: late cyanosis & digital clubbing after the L
to R shunt shifts to R to L & becomes cyanotic
How do steroids block the production of cytokines? Inactivate NFkB, a
transcription factor that induces proinflammatory cytokine production
What does Cromyolyn do?

Mass cell mediator release

inhibitor\n\nRx: asthma prophylaxis


What causes hyperlipidemia in nephrotic syndrome?

Hepatic

overproduction of lipids + a defect in lipoprotein lipase


What are the different defects that cause the familial dyslipidemias (I, III,
IIa, IV, V)?

I: lipoprotein lipase defect\nIII: altered lipoprotein E\nIIa:

decrease in LDL receptors\nIV: hepatic overproduction of VLDL\nV:


increased heaptic production & decreased clearance of VLDL
What is the main reservoir of Borrelia burgdorferi?

Whitefooted mouse

Which diuretic groups act on the luminal surface by inhibiting transporters,


which do not?

1) Thiazides, Loops, Ksparing all act on the luminal

surface\n2) Carbonic anhydrase inhibitors inhibit a cytoplasmic


enzyme\n3) Spironolactone inhibitis steroid receptor function
What are the S/S of intussusception?

1) Vomiting (due to partial

obstruction)\n2) Bloody diarrhea\n3) Hyperactive bowel sounds\n4)


Palpable mass (usually at the ileocecal junction)
What is the histology of polymyositis? An infiltrate of CD8+ T
lymphocytes & macrophages surround & destroy healthy muscle fibers\n
SPARES the extraocular & facial expression muscles\n Progressive
symmetric muscle weakness that is often subacute
What 3 diseases are associated with HLADR4?
arthritis\n2) Diabetes mellitus\n3) SLE

1) Rheumatoid

What 2 diseases are associated with HLAB8?

Celiac

disease\nDermatitis herpetiformis
What disease is associated with HLABW22? Kawasaki's
What 3 diseases are associated with HLADR3?

Chronic active

hepatitis\nSjogren's syndrome\nType 1 diabetes mellitus


What ECG picture is a sign of Digoxin toxicity?

Severe bradycardia

(36 bpm) with complete heart block\n Often precipitated by an increase


in FUROSEMIDE (increases Ksecretion & thus, this hypokalemic state
potentiates the effects of digoxin)\n\nRx: Digoxin immune FAB
In what type of heart problem can Adenosine be used?

Reentrant

supraventricular tachycardia: binds to A1 receptors & induces a heart


block within the AV node
What type of arrhythmia is Amiodarone used to manage? Recurrent
ventricular fibrillation\n Hemodynamicallyunstale ventricular tachycardia
What 4 types of arrhythmia is cardioversion used to treat? Atrial
flutter\n Atrial fib\n Ventricular tachycardia\n Ventricular fib
How is Osteoarthritis characterized?

Loss of cartilage elasticity\n

Eburnation: polished ivorylike appearance of bone\n Cystic changes in


the subchondral bone\n Osteophyte formation at the perimeter of the
articular surface
What disease can present with Heberden's or Bouchard's nodes?
Osteoarthritis:\n Heberden's: osteophytes @ the distal
interphalangeal joints\n Bouchard's: ostephytes @ the proximal
interphalageal joints
What are the possible causes of microcytic anemia? 1) Iron deficiency:
decreased intake or increased loss through bleeding\n2) Chronic
inflammatory disease (e.g. rheumatoid arthritis)\n3) Thalassemia\n4)
Sideroblastic anemia
What are the possible causes of normocytic anemia?

1) Acute blood

loss\n2) Aplastic anemia\n3) Chronic renal failure\n4) Hemolytic

anemia\n5) Hereditary spherocytosis\n6) Sickle cell anemia\n7) G6PD


deficiency\n8) Paroxysmal nocturnal hemoglobinuria
What are the 3 possible causes of macrocytic anemia?

1) Vitamin B12

deficiency\n2) Folate deficiency\n3) Pernicious anemia


What are the 5 actions of PTH? 1) Increases Ca absorption in the small
intestine\n2) Promoting Ca reabsorption in the renal tubules\n3)
INHIBITING phosphate reabsorption in the reanl tubules\n4) Inhibiting
further PTH release (negative feedback)\n5) Stimulating 1,25
dihydroxyvitamin D
What are the 3 causes of microcytic, hypochromic RBCs (anemias)?

Irondeficiency anemia\n Thalassemias\n Lead poisoning


What are the lab values for anemia of chronic disease?

Decreased

serum iron\nDecreased TIBC\nNormal % saturation


What lab values indicated hemosiderosis?

Increased serum

iron\nNormal TIBC\nIncreased % saturation


What is the radiographic picture of Pneumocystis jiroveci pneumonia?

Ground Glass = Diffuse Bilateral infiltrates\n Absence of air


bronchograms\n No obliteration of the pulmonary vessels
What is the radiographic picture of Mycoplasma pneumoiae infection?
Peribronchial pneumonia pattern with STREAKS of infiltrates
What is the radiographic picture of Staphlococcus aureus pneumonia
(rare)?

Cavitations &/or abscesses within the lung fields

What is the radiographic picture of Streptococcus pneumoniae infection?


Lobar consolidation
What is seen in pathologica preparations of herpes zoster skin rashes?
Cowdry A inclusion bodies: eosinophilic inclusions surrounded by a
clear halo
What are Auer rods?

Rodshaped bodies in myeloid cells = fused

lysosomes\n found in APL (acute, promyelocytic leukemia; M3)

What are CallExner bodies?

Spaces between granulosa cells in ovarian

follicles & in granulosa cell tumors


How does hereditary spherocytosis result in anemia?

Shortened life

span & increased destruction of unstable RBCs:\n Disease results in


decreased elastic deformability of RBC cell membranes
What cells maintain the BBB?

Astrocytes

What cells produce CSF? ependymal cells that line the ventricles
What 3 conditions must be met to diagnose delirium?

1) Acute &

usually fluctuating (hours to days) disturbance of consciousness (reduced


ability to focus, sutain, or shift attention)\n2) A change in cognition (e.g.
memory disturbance, hallucination, or other perceptual disturbances)\n3)
Evidence of a direct physiologic cause (e.g. infection)
How is a brief psychotic disorder characterized?

1) One of the

following: hallucinations, delusions, disorganized speech or grossly


disorganized behavior\n2) Duration: lasts @ least 1 day, but less than a
month\n3) Patient returns to full functionality\n4) State NOT attributable to
a general medial condition or other psychiatric disorder
What is necessary for a diagnosis of delusional disorder?

1) Nonbizarre

delusions (i.e. plausibel situations in reality), which are not attributable to


another psychiatric disorder\n2) Duration: @ least 1 month\n3) Does NOT
markedly impair the person's life: ramifications are limited to the
delusional content
How is dementia characterized? Chronic condition of cognitive decline with
an INTACT level of consciousness
What are the acute phase symptoms of schizophrenia?

@ least 2 of the

following for a 1month period:\n1) Delusions\n2) Hallucinations\n3)


Disorganized speech\n4) Grossly disorganized or catatonic behavior\n\n&
Negative symptoms: flat affect, lack of motivation, or poverty of speech
What are the major toxins associated with TCC of the bladder?
"associated problems in your Pee SAC":\n Phenacetin (painkiller)\n

Smoking\n Aniline dyes\n Cyclophosphamide \n\nS/S: painless hematuria


(post 40 age patient)
How are bile ductules identified histologically?

Lined by simple

cuboidal epithelium & each luminal cell has a single centrallylocated


nucleus
What does the portal vein carry?

Blood from the alimentary canal,

spleen, & pancreas to hepatic cells


How does surfactant reduce surface tension? Disrupts intermolecular
forces (hydrogen bonds) between molecules of water
What is the MOA of Lidocaine as an anesthetic?

Blocks Na channels on

neuronal membranes to reduce sensory impulse conduction from the


periphery to the CNS\n Nerve Sensitivity:\n1) small > large\n2)
myelinated > unmyelinated
What causes atypical pneumonia in those recently exposed to a bird?
Chylamydia psittaci (intracellular parasite w/o cell wall)\n\nRx: 10
day course of doxycycline or tetracycline
How does HSV move in the peripheral nervous system?

Retrograde

axonal transport to the neuronal cell body to remain latent in the dorsal
root ganglia
What is the treatment for Wilms' tumor?

MOPP

regimen:\nMechlorethamine\nOncovin (vincristine =
neurotoxic)\nProcarbazine\nPrednisone
What 2 chemotherapeutic agents can cause pulmonary fibrosis?
Busulfan (CML Rx)\n Bleomycin (testicular cancer & lymphoma Rx)
What are the FEV1 & FVC values for an interstital lung disease? FEV1 &
FVC are both reduced & the ratio between the 2 remains the same
Where does pain occur in intermittent obstruction of the gall bladder?
referred pain near the right scapula & pain after eating (due to
gallbladder contraction)
Where do Naegleri fowleri live? In fresh water

What causes secondary Achalasia?

Chagas' diseases (T. cruzi):

destruction of the myenteric plexus of the esophagus\n Dx: bird's beak on


barium swallow (proximal constriction of the esophagus)
Where is ADH (vasopressin) & oxytocin synthesized ADH (aquaporins):
supraoptic nuclei of the hypothalamus\n\nOxytocin (milk "let down"):
paraventricular nuclei of the hypothalamus
What hormone stimulates spermatogenesis? FSH from the anterior
pituitary gland
What is the most common cause of vaginal discharge worldwide?
Trichomonas (motile, flagellated parasite) > symptomatic
vaginitis\n\nRx: Metronidazole
Does Candida vaginitis present with a discharge?

Yes, white &

odorless\n\nRx: Fluconazole
Which of the following are radiopaque? Calcium, Ammonium, Magnesium,
NaCl, Uric Acid

Calcium

Besides generalized decrease in bone mass (osteopenia), what other


abnormalities is Osteogenesis associated with?

Blue Sclerae\n

Dental abnormalities\n Progressive hearing loss


What is chondrodysplasia?

A group of inherited skeletal disorders

characterized by defects in type II collagen


What is achondroplasia? AD dwarfism:\n Mutation in the fibroblast
growth factor receptor 3 gene\n Short limbs with normal trunk & skull
size\n IF they have a narrow foramen magnum, this can impinge on the
brain stem & cause neurologic dysfunction or death
Along what nerve do baroreceptors from the carotid sinus send signals to
the medulla regarding vasotension?

blossopharyngeal

Which bareoreceptor (in the carotid sinur or aortic arch) only responds to
an INCREASE in Bp?

Aortic arch

Why is Alendronate a DOC to treat multiple myeloma?

It is an ornal

bisphosphonate:\n Lowers serum Ca2+\n Helps to decrease pain &

fractures by inhibiting bone resorption through the reduction of the # &


activity of osteoclasts
What type of diuretics should be avoided in a patient with multiple
myeloma?

Thiazides: worsen hypercalcemia by increasing reaborption of

Ca2+
What common GI problem makes a patient a bad candidate for
bisphosphonates like Alendronate?

GERD: bisphosphonates can cause

erosions of the esophagus


Which interlukin is secreted by helper T lymphocytes that stimulates the
growth of helper & cytotoxic (CD8) T lymphocytes to fight VIRAL
infections?

IL 2

What interleukin is secreted by activated T lymphocytes & functions


similarly to granulocyte macrophage colonystimulating factor? IL3
What interleukin is secreted by helper T lymphocytes and promotes the
growth of B lymphocytes to fight bacterial infections?

IL4

What interleukin is a major chemotactic factor for neutrophils?

IL8

What is the Schilling test? An infusion of exogenous Intrinsic factor (usually


produced by gastric parietal cells) that will enable vitamin B12 absorption
and then radiolabelled B12 will be found in the urine:\n Confirms a
diagnosis of pernicious anemia
Where is B12 absorbed? Terminal ileum (in the presence of intrinsic
factor)
What are the S/S of tuberous sclerosis (TS)? Sagreen patch: rough,
raised lesions that has the consistency of an organg peel (in the
lumbosacral area)\n Seizures\n CT/MRI showing TUBERS (calcified
hamartomas in the periventricular region = nodular proliferation of
multinucleated atypical astrocytes)\n Adenoma sebaceum: facial
angiofibromas
What is characterized by small, firm, umbilicated, skincolored, pearly
white papules?
manifestation

Contangious molluscum contagiosus virus skin

What do cafe au lait macules look like? Hyperpigmented,


sharply=bordered, ovoid macules:\n >5mm prepuberty & >15 mm post
puberty\n 6+ are needed to consider a diagnosis of NFT1
What are tender, violaceous, subcutaneous nodules of the palms & soles
indicated of? Osler's ndoes of bacterial endocarditis
What is thickened & hyperpigmented skin on the back of the neck &
axillae?

Acanthosis nigricans: can be found in patients with diabetes,

obesity, & GI malignancies


What is the typical profile of a patient that suffers from Prizmetal's
(variant) angina?

Patient is young\n Pain is NOT prompted by

activity\n Pain relieved by sublingual nitroglycerin\n Normal


ECG\n\nCause: coronary vasospasm
What is BuddChiari syndrome (BCS)? Nearly complete obstruction to the
blood flow by an acute clot in the hepatic veins or the IVC\n\nPredisposing
disorders:\n Hematologic (polycytemia veria, essential thrombocytosis)\n
Thrombotic diatheses (antiphospholipid antibody syndrome, factor V
Leiden)\n Pregnancy\n OC use\n Intraabdominal neoplasms
(hepatocellular carcinoma, renal cell)
What are predisposing disorders for BuddChiari?

Nearly complete

obstruction to the blood flow by an acute clot in the hepatic veins or the
IVC\n\nPredisposing disorders:\n Hematologic (polycytemia veria,
essential thrombocytosis)\n Thrombotic diatheses (antiphospholipid
antibody syndrome, factor V Leiden)\n Pregnancy\n OC use\n Intra
abdominal neoplasms (hepatocellular carcinoma, renal cell)
What glut transporter allows glucose to enter pancreatic beta cells?
GLUT 2
What glut transporter allows glucose to enter RBCs & brain?

GLUT 1

What glut transporter allows glucose to enter adipose & skeletal muscle
cells? GLUT 4

What syndrome is associated with bilateral obliteration of the amygdala?


KluverBucy syndrome: hyper (orality, sexuality, & disinhibited
behavior)
What area of the brain shows a decreased number of neurons in
Alzheimer's disease?
What is pica?

Nucleus basalis of Mynert

eating nonnutritive substances\n\n can indicated Lead

poisoning
Describe the time bomb of acetaminophen tox?

Initially:

asymptomatic\n 1st 24 hrs: nausea, vomiting, & malaise\n 2448 hrs:


Right upper quadrant pain only\n Day 34: jaundice & hepatic failure
What is Asprin tox at low & high doses?

LOW: tinnitus, abdominal

pain, vomiting, & tachypnea\nHIGH: respiratory alkalosis, metabolic


acidosis, hypotension, coma, & DEATH
What accidental overdose presents with hallucinations, dry mouth,
flushing, urinary retension, & mydriasis?

Tricyclic antidepressants

(excess anticholinergic activity)


What is the MOA of zileuton?

Blocks the conversion of arachidonic

acid to leukotrienes by inhibiting 5lipoxygenase\n Rx: asthma\n SE:


elevation of liver enzyme levels, sinusitis, nausea
What is asthma characterized by?

Airway hyperresponsiveness\n

Smooth muscel hypertrophy\n Mucous plugging


What chronic condition can prednisone induce?

Steroidinduced

diabetes\n Monitor blood glucose if the patient is on longterm therapy


What syndrome is associated with montelukast (Asthma med)?

Churg

Strauss (autoimmune vasculitis)


How does T1 radiculopathy present?

Pain & numbness of the medial

forearm\n Weakness to thumb flexion/abduction & finger


abduction/adduction

How does C8 radiculopathy present?

Pain & numbness of the medial

forearm & hand (4th & 5th digits)\n Weakness to wrist extension & finger
extension/abduction/adduction
How does C6 radiculopathy present?

Pain & numbness of the lateral

arm, forearm & hand (1st & 2nd digits)\n Weakness to shoulder
abduction/external rotation & elbow flexion/supination\n Diminished
biceps reflex
How does C5 radiculopathy present?

Neck & shoulder pain\n

Numbness of the skin on the lateral arm\n Weakness to shoulder


abduction/external rotation & elbow flexion/supination\n Diminished
biceps reflex
How does C7 radiculopathy present?

Pain & numbness extending to the

hand & index & middle finger\n Weakness to elbow & wrist extension\n
Diminished triceps relex
Define relative risk. The incidence rate of some outcome in those
exposed to a risk factor divided by the incidence rate of those not
exposed\n RR = probability (exposed) / probability (control)\n for
COHORT studies
What is the target organelle for chloramphenicol & erythromycin?
mitochondira: has 30S50S robosomal system (a reminent of its
prokaryotic origin)
Which cells in the skin are already dead?

Keratinized epithelial cells

(e.g. stratum corneum layer of the skin)


What are ribozymes?

RNA molecules with catalytic activity:\n bind

target RNA & cleave the phosphodiester backbone @ specific cutting sites
What builds up in the urine when a patient has lead poisoning
Aminolevuline & coproporphrin III b/c of the inhibition of
aminolevulinate dehydratase & ferrochelatase enzymes of the heme
synthesis pathway

What builds up iwn 3betahydroxysteroid dehydrogenase deficiency?


DHEA (Dehydroepiandrosterone) & pregnenolone due to this form of
congenital adreanl hyperplasia
What builds up in the urine of patients with acute intermittent porphyria?
Porphobilinogen: due to a deficiency in urophorphyrinogen I
synthases
What is found in the urine of patients with the most common porphyria?
Uroporhyrin due to a deficiency in uroporphyrinogen decarboxylase
in patients with porphyria cutanea tarda
How do patients with AIP (acute intermittent porphyria) present?
Hyponatremia & recurrent episode of abdominal pain \n RARELY:
neuropsychiatric problems\n attacks can be precipitated with barbiturates
& starvations diets\n NO Photoensitivity
What is Fanconi's Anemia?

AR: DNA repair is defective\n aplastic

anemia & defective DNA\n increased risk of malignancy


What are the common symptoms of porphyria cutanea tarda?

Cutaneous fragility of hands & forearms after SUN exposure\n


Hypertrichosis\n Sclerodermalike plaques\n Redish urine (due to
increased levels of uroporphyrin)
What is the most likely causitive organism in IV drug user osteomyelitis?
Candida albicans
Describe the pathway of an upper motor neuron.

When a Neisseria

gonorrhoeae infection spreads from the reproductive organs to the liver


capsule resulting in acute perihepatitis
What is the most likely organism to cause infection in patients with
indwelling catheters or those with prosthetic joint replacements?
Staphylococcus epidermidis: gram , coagulase +, can form
BIOFILMS on prosthetics
What is it called when TB involves the vertebral bodies?

Pott's disease or

tuberculous spondylitis \n osteomyelitis & arthritis in 2(+) vertebra

What are the 4 most popular typical antipsychotics?D2 dopamine receptor


blockers:\n1) Thioridazine\n2) Haolperidol\n3) Fluphenzaine\n4)
Chlorpromazine
What is the MOA of atypical antipsychotics? block 5HT2 & dopamine
receptors
Why do patients with ChediakHigashi disease sometimes present with
partial albinism?

This AR disease is characterised by a failure of

phagolysosome formation\n melanosomes are derivatives of lysosomes &


thus can be dysfunctional as well
To what type of infection are patients with chronic granulomatous disease
most susceptible? Microbes that produce their own catalase (e.g.
Staphylococcus & Candida)\n Due to NO NADPH oxidase \n Dx: negative
tetrazolium dye reduction
How do patients with selective IgA deficiency present?

Sinus & Lung

infections due to lack of enough functional IgA in mucous membranes


What immunodeficiency presents with severe, recurrent bacterial, viral,
protozoal, & fungal infections?

SCIDs b/c of a defect in early stemcell

differentiation
What is the product of the p53 gene? a transcription factor that
regulates apoptosis & prevents defective cells from undergoing
division.\n mutation causes uncontrolled cell division
What are the products of the BCR1 (ch. 17) & BRCA2 (ch. 13) genes?
DNA repair products
What is the product of the APC gene (ch. 5)? Membrane cell adhesion
products
What drug is used to prevent chemoinduced nausea/vomiting and has the
SE of headache & constipation? Ondansterol (5HT antagonist)
What is the MOA of metoclopramide? Prokinetic agent that promotes GI
motility\n Potent dopamine antagonist that enters the CNS & makes it
antiemetic & SE of extrapyramidal (i.e. Parkinsonianlike adverse effects)

What is Trimethobenzamide used for? Nausea & vomiting\n MOA =


unknown\n SE: headache, depression, muscle cramps, dizziness
What are the derivatives of the 1st aortic arch?

Maxillary artery (part

of it is derived from this arch)


Define "hot TBone Steak"

IL1 = fever (Hot)\nIL2 = T lymphocyte

stimulator\nIL3 = Bone marrow stimulator\nIL4 = IgE stimulation\nIL5 =


IgA stimulation
What cancers is recombinate IL2 (adesleukin) used to trat?

Renal cell

& Metastatic melanoma


What are the 3 pathologic stages in RDS?

1: exudative stage with

intraalveolar hyaline membrane formation\n\n2: proliferation stage with


increase in type II pneumocytes & fibroblast\n\n3: fibrotic stage with lung
remodeling & fibrosis
What antihypertriglyceridemia drug is contraindicated in patients with
cholesterol gallstones?

Gemfibrozile b/c fibrates can increase the

development of gallstones
What antihypertensive drugs are most useful for patients with asthma? A
BEAM: (selective B1blockers)\n Acebutolol\n Betaxolol\n Esmolol\n
Metoprolol
What is the CD marker on all lymphocytes of thymic origin CD2 is on all
helper Tcells, cytotoxic Tcells, & natural killer cells
What lymphocytes have CD3?

All CD4 & CD8 cells, but NOT natural killer

cells
What is the Tcell (TCR) receptor?

alpha/beta heterodimer encoded by

genes undergoing V(D)J recombination that is expressed by each clone of


the T lymphocyte that gives it antigen binding specificity
Why does prolonged diarrhea & vomiting cause hyponatremia (& t/f
seizures)?

1st: Loss of solute & H20 loss\n2nd: Hypovolemia induces

secretion of ADH (conserves H2O without Na)\n3rd: Hyponatremia >


neurologic dysfunction induced by cerebral edema

Why does metabolic alkalosis result in hypokalemia?

More H2O &

bicarb are delivered to the distal Ksecretory site in the kidney &
aldosterone is released = renal loss of potassium
How does hyperkalemia present?

Weakness progressing to flaccid

paralysis\n Metabolic acidosis\n Cardiac conduction disturbances by


peaked T wave on ECG
What is the most common H. influenzae type to cause acute otitis media?
NTHi = Haemophilus influenzea nonencapsulated (nontypable): \n
75% of adults & children carry this bac in their nasopharynx\n NB: Type B
is covered by the childhood vaccine
What are the 3 most common causes of acute otitis media?

1)

Streptococcus pneumoniae (aerobic, gram + coccus)\n2) Haemophilus


influenzea (NTHi)\n3) Moraxella catarrhalis (gram , diplococci)
How does a Nocardia asteroides present in an immunocompromised
patient?

lung abscesses & cavitations\n late erosion of blood vessels

and dissemination to brain: abscesses\n gram +, filamentous, weakly


acidfast\n\nRx: sulfonamides (dihydropteroate synthetase blockers =
blocks nucleotide synthesis)
What is the MOA of amphotericin B?

Antifungal that binds ergosterol &

membrane pores in the outer membrane


What fructose metabolism enzyme is missing in patients that experience
severe hypoglycemia, vomiting, jaundice, & hemorrhage? Aldolase B\n
No conversion of fructos1phosphate to glyceraldehydes &
dihydoxyacetone phosphate\n Result: intracellular trapping of fructose1
phosphate
What vasculitis is associated with Hep B infection in 10% of cases?
Polyarteritis nodosa:\n NO lung involvement\n Livedo reticularis:
puplish discoloration skin rash\n transmural infalmmation of s/m arteries
> fibrous, thickened vessel wall (each lesion is at a different age)\n
episodes last a few weeks to moths & <10% relapse
What diabetic drug causes GI adverse effects including abdominal cramps,
diarrhead, & flatulence? & is contraindicated in cirrhosis & requires liver

function monitoring?

Acarbose: alphaglucosidase inhibitior\n MOA:

decreases hydrolysis & absorption of diaccharides & polysaccharides at


the intestinal brush border \n Reduces postprandial hyperglycemia
What patients cannot be prescribed metformin?

those with renal

insufficiency (b/c of the risk of lactic acidosis)


Why were 1st generation thiazolidinediones (like troglitazone) taken off
the market? Extreme hepatotoxicity \n MOA: sensitizes the peripheral
tissues to insulin's action
What is lacking in Factor V Leiden disorder?

Mtation resulting in an

argininetoglutamine substitution @ position 506 in factor V & is thus


resistant to cleavage by protein C
What should be ruled out in all patients >50 years old that complain of
nonremitting, yet, tolerable, flank pain & hematuria without signs of
infection?

Malignancy\n check for renal cell: contrastenhancing, solid

renal mass arising from the renal cortex upon imaging


What is the antidote for methanol or ethylene glycol (antifreeze) tox?
Fomepizole
What is the antidote for tricyclic antidepressant tox?
What is the antidote for cyanide tox?

NaHCO3

Thiosulfate

What are nononcologic uses of methotrexate?

MOA: folic acid analog

natimetabolite that inhibitis dihydrofolate reductase: \n Ectopic


pregnancy\n Psoriasis\n Inflammatory bowel disease\n Rheumatoid
arthritis
Why are tetracyclines used in early rheumatoid arthritis? Inhibit the
activity of metalloproteinases that are involved in join destruction by the
rheumatoid synovium
What are nononcologic uses for cyclophosphamide?

MOA: alkylating

agent\n SLE immunosuppressant\n MS\n Autoimmune hemolytic


anemia

What is the MOA of Bblockers? Inhibit Breceptors & inhibit the G


proteins/cAMP mechanism\n Decreased cAMP & protein kinase A =
decreased Na+ & Ca2+ current within the AV node\n T/F decrease the
slope of phase 4 & ) = suppression of abnormal pacemakers
Where are the nicotinic receptors of ACh?

Direct membrane receptor

coupling to a Na+/K+ ion channel:\n ANS ganglia\n Skeletal


neuromuscular junctions\n CNS
Where are the GABAa receptors that are directly coupled to Cl ion
channels?

CNA

What receptors increase cAMP to activate protein kinase A to


phosphorylate tissue specific substrate enzymes?

Gs:\n catecholamines

(beta)\n dopamine (D1)\n glucagon\n histamine (H2)\n


prostacycline\n 5HT
What receptors decrease cAMP to down regulate protein kinase A & lower
phosphorylate tissue specific substrate enzymes?

Gi:\n Catecholamines

(alpha 2)\n ACh muscarineic (M2)\n Dopamine (D2 subtypes)\n Several


opiod & 5HT subtypes
Which receptor systems activate phospholipase C to release IP3 & DAG
from membrane PIP2 (IP3 then releases Ca2+ from the SR, which with
DAG activates protein kinase C to phosphorylate tissue specific substrate
enzymes?

ACh (M1 & M3)\n Norepinephrine (alpha 1)\n Angiotension

II\n Several opioid & 5HT subtypes


What is the mechanism of vascular smooth muscle vasodilation? Cyclic
GMP is the 2nd messanger\n Facilitates dephosphorylation of myosin light
chains to prevent their interaction with actin
How does Nitric oxide (NO) promote vasodilation?

Released from

endothelial cells by vasodilatiors (e.g. H1 & M3 agonists, or nitrates) to


activate guanylyl cyclase to increase cGMP
What receptors mediate the first steps in insulin & growth factor signaling?
EGF: endothelial growth factor\nPDGF: platelet derived growth
factor\nANF: atrial naturitic factor\n\nRecognition sites extracellularly +

tyrosine kinase cytoplasmic domains that are dimerized upon ligand


binding & result in tissuespecific substrate protein phosphorylation
What are Nn receptors?

Nicotinic: PANS & SANS ganglia & adrenal

medulla
What are Nm receptors? Nicotinic: skeletal muscle motor endplate
What are M receptors?

Muscarinic: organs/tissues innervated by post

ganglionic nerves of the PANS & thermoregulatory sweat glands of SANS


What are & receptors?

Adrenoceptors: organs/tissues innervated

by postganglionic nerves of SANS


What receptors use ACh as a neurotransmitter?

N & M receptors of

innervated tissues:\n Nicotinic: skeletal muscle motor endplate\n


Muscarinic: organs/tissues innervated by postganglionic nerves of the
PANS & thermoregulatory sweat glands of SANS
What receptors use NE as the neurotransmitter?

Most adrenoceptors in

innervated tissues
What receptors use DA (dopamine) as the neurotransmitter?

D1

receptors in renal vasculature


What receptors use E (Epinephrine) as the neurotransmitter?

Adrenal

medulla\n Activates most adrenoceptors whether or not the tissues are


innervated
What receptors use 5HT, purines, & opioid peptides as the
neurotransmitter? ENS & many ANS fibers in the GI tract
What is the neural response to an increase in Bp?

Baroreceptor

discharge\n Increase in PANS activity > bradycardia\n Decrease in SANS


> decrease in HR & decrease in contraction & decrease in
vasoconstriction
What is the neural response to a decrease in Bp?

ANS neural

feedback:\n decrease PANS outflwo & increased SANS > increased CO &
TPR

What is the hormonal response to decrease mean Bp (i.e. hypotension)?


Decrease in renal blood flow > decrease in renal pressue\n
Increase in renin release > increase in angiotensins\n Angiotensin II
increases aldosterone release from the adrenal cotex\n Retention of Na &
H2O to increase blood volume\n Increased venous return increases Co\n
Angiotension II also causes vasoconstriction to increase TPR
What is the target of indirectacting cholinomimetics?

AChE: the amjor

mechanism of termination of ACh actions


How does the Cholinergic neuroeffector junction work?

ACh is

synthesized via ChAT & accumulates in synaptic vesicles\n Excitation


opens voltagedependent Ca2+ channels and there is a Ca2+ INFLUX\n
ACh is released and activates postjunctional receptors
Where does Botulinum toxin act?

Blocks ACh release at Cholinergic

neuroeffector junctions
Name the cholinomimetics that target postjunctional cholinergic receptors
(directacting cholinomimetics)? Nicotinic: Nicotine\n Muscarinic:
bethanechol, methacholine, pilocarpine
Name the cholinoceptor antagonists that target postjunctional cholinergic
receptors (directacting antagonists/blockers)?

Nicotinic (Nn):

hexamethonium, mecamylamine\n Nicotinic (Nm): tubocurarine,


atracurium, succinylcholine\n Muscarinic: atropine, benztropine,
glycopyrrolate, scopolamine
What are the Muscarinic receptors of the Eye?

M3:\n Sphincter

(contracts = miosis)\n Ciliary muscle (contract = accomodation for near


vision)
What are the Muscarinic receptors of the Heart?

M2:\n SA node:

decrease in HR: negative inotropy\n AV node: decrease in conduction


velocity
What are the Muscarinic receptors of the Lungs?
contract/bronchospasm\n Increased secretion

M3:\n Bronchioles:

What are the Muscarinic receptors of the GI? M3: increased motility =
cramps\nM1: increased secreation\nM3: contraction = diarrhea
What are the Muscarinic receptors of the bladder?

M3: \n Wall:

contraction\n Sphincter: relaxation


What does activation of M3 receptors generally do to sphincters?Relaxes
them\nEXCEPT lower Esophageal
What are the Muscarinic receptors of the glands?

M3: \n increases

secretation (sweat = thermoregulatory)\n increases salivation\n


increases lacrimation
What are the Muscarinic receptors of the glands?

M3:\n dilation VIA NO

or EDRF (not innervation)


What are the Nicotinic receptors of the Adrenal medulla? Nn: secretion of
E & NE
What are the Nicotinic receptors of the Autonomic ganglia?

Nn:\n

Stimulation: effects depend on ANS innervation & dominance:\n BV =


SANS = vasoconstriction\n GI = PANS = increased motility & secretions
What are the Nicotinic receptors of the Neuromuscular junction? Nm: \n
Stimulation = twitch/hyperactivity
What is the mechanism of M1 & M3 receptors?

Cholinergic, Gq:\n

Increase DAG & IP3 = increased intracellular Ca2+


What is the mechanism of M2 receptors?

Cholinergic, G1:\n

Decreased adenylyl cyclase = decreased cAMP


What is the mechanism of Nn & Nm receptors?

NO 2nd

Messengers:\n Activation = opening of Na+/K+ channels


What is the activity, AChE hydrolysis, & clinical use for bethanechol?
DirectActing Cholinomimetic\n Activity: M receptors\n AChE
hydorlysis: none\n Rx: ileus (postop/neurogenic), urinary retention
What is the activity, AChE hydrolysis, & clinical use for methacholine?
DirectActing Cholinomimetic\n Activity: M > N receptors\n AChE
hydorlysis: yes\n Rx: bronchial hyperreactivity

What is the activity, AChE hydrolysis, & clinical use for pilocarpine?
DirectActing Cholinomimetic\n Activity: M receptors\n AChE
hydorlysis: none\n Rx: glaucoma (topical), xerostomia
What is the action & clinical use of edrophonium?

Indirectacting

Cholinomimetic:\n Action: shortacting\n Use: Dx myasthenia from


cholinergic crisis
What is the action & clinical use of physostigmine? Indirectacting
Cholinomimetic:\n Action: tertiary amine (enters CNS), intermediate
action\n Use:glaucoma, antidote in atropine tox
What is the action & clinical use of neostigmine/pyridostigmine? Indirect
acting Cholinomimetic:\n Action: quaternary amine (no CNS entry),
intermediate action\n Use: ileus, urinary retention, myasthenia,
myasthenia, reversal of nondepolarizing NM blockers
What is the action & clinical use of donepezil, tacrine?

Indirectacting

Cholinomimetic:\n Action: lipid soluble (CNS entry)\n Use: Rx =


Alzheimer's disease
What is the action & clinical use of organophosphates?

Indirectacting

Cholinomimetic:\n Action: lipidsoluble, longacting irreversibel


inhibitors \n Use: NONE (insecticides: malathion, parathion; NERVE GAS)
What are the symptoms of AChE inhibitor poisoning?
DUMBELS:\nDiarrhea\nUrination\nMiosis\nBronchoconstriction\nExcitiation
(muscle & CNS)\nLacrimation\nSalivation & Sweating
What is the management for AChE poisoning?

2 drugs, give

together:\n Atropine (enters the CNS)\n Pralidoxime (2PAM) to


regenrate phosphorylated AChE
What type of drug is atropine?

Muscarinic receptor antagonist;

PARASYMPATHOLYTICS\n Tertiary amine (enters the CNS)


What does atropine do?

"Dry as a bone; red as a beet, hot as a pistol,

blind as a bat, mad as a hatter"\n\n Heart: low dose = decreases HR,


moderate dose = increases HR\n Decreases secretions (salivary,
bronchiolar, sweat)\n Mydriasis & cycloplegia\n Hyperthermia with

resulting vasodilation\n Decreased GI motility\n DELUSIONS &


HALLUCINATIONS
How do you manage an atropine OD? symptomatic +/ physostigmine
What are the clinical uses of atropine? Muscarinic receptor antagonist;
PARASYMPATHOLYTIC:\n Antispasmodic\n Antidiarrheal\n
Antisecretory\n AChE inhibitor Overdose
What is the clinical use of tropicamide?

Muscarinic receptor

antagonist; PARASYMPATHOLYTIC:\n Opthalmology (topical)


What is the clinical used of ipratropium?

Muscarinic receptor

antagonist; PARASYMPATHOLYTIC:\n Ashtma & COPD (inhalational)\n NO


CNS entry
What is the clinical use of scopolamine?

Muscarinic receptor

antagonist; PARASYMPATHOLYTIC:\n Antimotion sickness\n SE: causes


sedation & shortterm memory block
What is the clinical use of Glycopyrrolate?

Antispasmodic\n

Antisecretory\n Antiulcer\n\nNO CNS entry


Where do ganglionic blocking agents act?

Competitive antagonists at

the Nn rectpors in ANS ganglia


What determines the effect of ganglionic blockers? Blockers REDUCE
dominate TONE\n\n PANS dominate in tissues with dual innervation (SA &
AV nodes; pupil, GI/GU muscles & sphincters) = BLOCKER > tachycardia,
mydriasis, & muscle relaxation & sphincter closure\n\n SANS dominate in
vascular tone & sweat glands\n= BLOCKER > vasodilation & reduced
secretion
What are the 3 ganglionic blockers?

Hexamethonium\n

Mecamylamine\n Trimethaphan
What is the use of ganglionic blockers? Shows whether a drug action is
direct (e.g. on heart rate) or due to ANS reflex response (blocks reflex
brady/tachycardia elicited by change in mean Bp)

What happens when hexamethonium is given after phenylephrine is


administered?

Phenylephrine will cause vasocontriction\n

Hexamethoium will block the reflex bradycardia


What happens when hexamethonium is given after a Mreceptor activator
is administered?

Nothing, the action is direct on the heart & not a reflex

that Hexamethonium can block


What are the general characteristics of a nondepolarizing NMJ blocker? List
the 3 main drug examples

Competitive antagonist of NM muscle

endplate\n Reversible by AChE\n E.G.: tubocurarine, atracurium,


pancuronium
What are the general characteristics of a depolarizing NMJ blocker?

Agonist at NM receptors\n Initial fasciculation then paralysis through


persistent membrane depolarization\n NOT reversibel by AchE
inhibitors\n E.G. succinylcholine
Explain the main steps of NE synthesis?

1) Tyrosine is converted to

DOPA (dihydroxyphenylalanie) via tyrosine hydroxylase\n2) DOPA is


converted to dopamine (DA) via Laromatic amino acid decarboxylase
(DOPA decarboxylase)\n3) DA is converted to NE via DA betahydroxylase
& is taken up & stored in granules
How does inactivation of NE via MAO occur? Regulation/reduction in
prejunctional levels in the mobile pool, but has NO effect on granular NE
(i.e. already NE already stored in granules)
Explain the mechanism of NE release? 1) Membrane excitation opens
voltagedependent Ca2+ channels\n2) Ca2+ influx\n3) NE released from
granules into the neuroeffector junction\n4) NE activates postjunctional
receptors & effect depends on the alpha or beta adrenoceptor subtype
What are the 4 ways that NE action can be terminated?

1) NE

transporter system (reuptake)\n2) Activation of prejunctional alpha2


adrenoceptors that are then feedback inhibitors\n3) Diffusion from
neuroeffector junction\n4) Taken up by target cells & inactivated by COMT
What are MOA of MAO inhibitors?

Drugs: phenelzine,

tranylcypromie\n Increase prejunctional NE\n\nAction: inhibits MAO A &

B:\n MAO A (in liver) = also metabolizes NE, 5HT, & tyramine\n MAO B:
mainly in Brain & metabolizes DA
What drugs act on the mobile pool of NE (indirect acting
sympathomimetics)?

Amphetamine, Ephedrine, Tyramine\n MOA:

displaces NE & t/f enhances postjunctional actions


What indirect sympathomimetic inhibits NE reuptake to increase
postjunctional NE? Cocaine
What adrenoreceptor drugs activate prejunctional alpha receptors to
inhibit tyrosine hydroxylase & t/f NE release from synaptic vesicles?

Clonidine\n Alpha methyldopa


What drugs block the granular uptake of NE within neurons?
Reserpine: decreases prejunctional levels of NE available for release
What drugs block the release of NE from granules to decrease
postjunctional actions of NE?

Guanethidine

What distinguishes typical viral meningitis from CNS invasion by HSV?


CSF will show RBCs present due to necrosis of infected brain tissue &
resultant cerebral hemorrhage\n Otherwise, the CSF findings will be viral:
increased lymphocytes, normal/elevated protein, normallow glucose
How can selective COX2 inhibitors (like Celecoxib) cause renal failure?
Inhibits prostaglandin synthesis \n If a patient becomes DEHYDRATED,
they have no physiological mechanism to retain renal blood flow
(vasodilation of the afferent & efferent arterioles)
Where is the macula densa in the kidney?

portion of the thick

ascending limb adjacent to the hilus of the glomerulus


What does inhibition of cycloxygenase2 enzyme result in?

Decrease

in production of:\n prostaglandins\n thromboxane\n prostacyclines


(reduce platelet function)
What does myxedema look like? Nonpitting edema on the anterior surface
of both legs with overlying skin that is dry & waxy & may have several
diffuse, slightly pigmented papules

What is the MOA of propylthiouracil & methimazole?

Block tyrosine

iodination (i.e. organification) & coupling\n Prophylthiouracil also


decreases peripheral conversion of thyronine to triodothyonine\n\nRx:
hyperthyroidism (e.g. Grave's)
What determines blood flow at the base of the lungs when a person is
standing?

difference between arterial & venous pressure (arterial

pressure is greatest & alveolar pressure is weakest)


In the middle lobes of the lung, what has the greatest pressure: arterial,
alveolar or venous?arterial > alveolar > venous
What is the treatment for nephrogenic diabetes insipidous?
Hydrochlorothiazide: inhibits reabsorption of Na from the H20
impermeable early distal convoluted tubule. Inhibits hyperdilution
How is the late distal convoluted tubule & collecting duct made amenable
to H20 absorption? Hypertonic intersitium enabled by the thick ascending
limb: solute transport & deposition of ions into the medullary interstitial
fluid
What organ clears the body of capsulated organisms?

spleen

Why does Wilson's cause extrapyramidal symptoms?

Basal ganglia

degeneration
What lab results indicate Wilson's?

Severe hepatic impairment >

hemolytic anemia\n Reduction in serum levels of cerulopasmin (copper


carrier protein)
What is the DOC for Wilson's

penicillamine

What disease is caused by the loss of debranching enzyme?

Cori's\n

accumulation of glycogen in liver, heart, & skeletal muscle\n S/S: (like Von
Gierke's but milder) hyperlipidemia, fasting hypoglycemia, heaptomegaly
Where does glucocerebroside accumulate in Gaucher's?

Monocytes &

macrophages = enlarged histiocytes with "wrinkled tissue paper"


appearance

Where does heparan sulfate & dermatan sulfate (mainly) accumulate in


patients with Hurler's

Heart\nBrain\nLiver

What melena indicated? Blood loss in the upper GI (lower GI blood loss =
RED stools)
What is the most common cause of iron deficiency anemia in
postmenopausal women? Occult blood loss: usually from a GI source
What are the adrenoreceptors in the eye?

Alpha1:\n Radial (dilator)

muscle: contraction/mydriasis
What are the adrenoreceptors in the arterioles (skin/viscera)?
Alpha1:\n Contraction = increased PVR & afterload
What are the adrenoreceptors in the veins?

Alpha1:\n Contraction =

increased venous return & preload


What are the adrenoreceptors in the bladder trigone & sphincter?
Alpha1:\n Contraction = urinary retention
What are the adrenoreceptors in the male sex organs?

Alpha1\n Vas

deferens contraction = ejaculation


What are the adrenoreceptors in the liver?

Alpha1:\n increased

glycogenolysis
What are the adrenoreceptors in the prejuctional nerve terminals?
Alpha2:\n decreased transmitter release & NE synthesis
What are the adrenoreceptors in the platelets?

Alpha2:\n

aggregation
What are the adrenoreceptors in the pancreas?

Alpha2:\n decreased

insulin secretion (these are the dominate receptors)


What are the adrenoreceptors in the heart?

Beta1:\n SA node =

increased HR (+ chronotrophy)\n AV node = increased conduction


velocity (+ dromotropy)\n Muscle = increased force of contration (+
inotrophy), conduction velocity, CO & O2 consumption\n HisPUrkinje =
increased automaticity & conduction velocity

What are the adrenoreceptors in the kidney? Beta1:\n Increased renin


release
Define each: chronotropy, dromotropy, inotropy

Chronotrophy:

HR\nDromotrophy: conduction velocity\nInotropy: force of contraction


What are the adrenoreceptors in the blood vessels? Beta2:\n Vasodilation
> decreased PVR, diastolic pressure, afterload
What are the adrenoreceptors in the uterus? Beta2: relaxation
What are the adrenoreceptors in the bronchioles?

Beta2: dilation

What are the adrenoreceptors in the skeletal muscle?

Beta2:

increased glycogenolysis > contractility (tremor)


What are the adrenoreceptors in the liver?

Beta2: increased

glycogenolysis
What are the adrenoreceptors in the pancreas?

Beta2: increased

insulin secretion
What is the dominace of adrenoceptor when drugs active on beta & alpha
receptors are given at low & high doses?

Low: beta dominance\nHigh:

alpha dominance\n\nE.G. Epinephrine


What type of adrenoceptors does dopamine activate?

D1 (peripheral)

in renal, mesenteric, & coronary vasculature\n Result: VASODILATION \n


In Kidney = increased GFR, RBF, & Na+ excretion
What type of receptor is alpha1?

Adrenergic\n Gq coupled:

increase in DAG & IP3 = increased intracellular Ca2+


What type of receptor is alpha2?

Adrenergic\n Gi coupled:

decreased adenylyl cyclase = decreased cAMP


What type of receptor are beta2, beta1, and D1?

Adrenergic \n Gs

coupled: increased adenylyl cyclase = increased cAMP


What is the action of alpha1 agonist drugs? Give 2 examples.

MOA:

increase Bp (& reflex bradycardia)\n Phenyleprine: decongestant

(mydriasis w/o cyloplegia)\n Methoxamine: Paroxysmal Atrial tachycardia


+ vagal reflex
What is the action of alpha2 agonist drugs? Give 2 examples.

MOA:

CNS actions > decreased vasomotor outflow & decrease in mean Bp\n
Clonidine: abrupt D/C causes rebound HTN\n Alpha methyldopa: prodrug
forming alphamethyl NE
What is the action of beta1&2 nonselective agonist drugs? Give 1
example.

MOA: increase in HR, SV, CO (beta1) & decrease in PVR

(beta2)\n increase in mean Bp = increase in pulse pressure & no reflex on


heart\n Rx: bronchospasm, heart block, bradyarrhythmias\n SE: flushing,
angina, arrhythmias\n\n ISOPROTERENOL
What is the action of beta1 > 2 nonselective agonist drugs? Give 1
example.

MOA: increase in HR, SV, CO (beta1) & no change in PVR,

GFR, or RBF\n Rx: acute CHF (tachyphylaxis)\n\n DOBUTAMINE


What are the 4 main selective beta2 agonists and what is the use of each?
Ashtma: metoproterenol, albuterol, terbutaline\n\nPremature Labor:
ritodrine
What is the action of Norepinephrine used as a drug?

MOA: alpha1,

apha2, beta1 agonist\n increased PVR = increase DIASTOLIC pressure\n


increased HR & SV = increased SYSTOLIC pressure & small increase in
PP\n increased mean BP w/ reflex bradycardia
What is the action of Epinephrine used as a drug?

MOA: Alpha 1+2, beta

1+2 agonist\n LOW dose: beta dominance = increase in HR & small


decrease in mean Bp\n HIGH dose: alpha dominace = increase PVR &
increase diastolic pressure = reflex bradycardia\n\n bronchodilation &
increased liver glycogenolysis, muscle glycogenolysis & glycolysis,
increased FFA (activates lipase)\n\n Rx: Anaphylaxis, cardiac arrest,
adjunct to local anesthetics, glaucoma
What is "epinephrine reversal"? The effect of using an alpha blocker
after eliciting a hypertensive response with a HIGH does of epinephrine\n
When alpha1 is blocked, then there is an unmasking of epinephrine's
beta2 activation = HYPOtension

What type of drug is amphetamine? What is its action & clinical use?
Indirect adrenoceptor agonist\n Releases NE from the mobile
pool\n Peripheral = sympathetic stimulation\n CNS = release of both NE
& DA\n\nRx: ADHD, weight loss, narcolepsy
What type of drug is amphetamine? What is its action & clinical use?
Indirect adrenoceptor agonist\n Releases NE from the mobile
pool\n Peripheral = sympathetic stimulation\n CNS = release of both NE
& DA\n\nRx: decongestant (also pehnlypropanolamine)
What type of drug is Dopamine? What is its action & clinical use? LOW
dose: D1 = increased renal/mesenteric blood flow > increase RBF &
GFR\n\nMEDIUM dose: B1 = increased CO (+ inotropy)\n\nHIGH dose: A1 =
vasoconstriction + increased systolic & diastolic Bp
What is the action of alpha adrenoceptor antagonists?

Decrease PVR

& mean Bp > tachycardia & H2O retention\n Marked reflex tachycardia if
alpha2 receptors are blocked too (reduced feedback inhibition)
Which alpha adrenoceptor antagonists are used to treat
pheochromocytoma?

Phenotolamine & Phenoxybenzamine:\n

Decrease PVR & mean Bp > tachycardia & H2O retention\n Marked
reflex tachycardia if alpha2 receptors are blocked too (reduced feedback
inhibition)
What type of drug is Prazosin? What is its clinical use?

Prototyle

alpha1 selective blocker\n Less reflex tachycardia\n 1stdose syncopy =


postural hypotension
What drugs have the same MOA as prazosin?
doxazosin\nterazosin\namulosin\n\nAll are alpha1 selective
blockers:\n decrease PVR & mean Bp > (a bit of) tachycardia & H2O
retention
What is the MOA of beta adrenoceptor antagonists? Heart: decreased
contractility, HR, CO, O2 demand & Bp\n Delayed hypoglycemis b/c
reduced glycogenolysis\n Decreased ciliary epithelial secretions =
decreased IOP\n SE: AV block & heart failure, bronchospasms in
asthmatics

What type of drug is propranolol? What are its SE? Prototype non
selective beta blocker\n Chronic used can lead to increased LDLC & TGs
What are the benefits & limitations of selective beta blockers?

Benefits: safer in asthma, diabetes, & PVD\n LImits: less effects on


vasculature, bronchioles, uterus, & metabolism\n\nE.G. acebutolol,
atenolol, metoprolol
What are the beta blockers with ISA?

ISA = intrinsic sympathomimetic

activity\n have less bradycardia & minimal change in plasma lipids\n


acebutolol & pindolol
Which betablockers have NO CNS entry? Which have a particularly long
halflife?

NO CNS entry: atenolol & nadolol\n Long HalfLIfe: cavedilol

& nadolol
Which beta blockers have combined alpha & beta blocking action?
Cavedilol\nLabetalol
Why should you taper doses when discontinuing beta blockers? B/C
chronic use of beta blockers leads to receptor upregulation \n T/F
tapering reduces excess CV rebound of endogenous amines
What is the action, ISA, sedation, effect on lipids, & clinical use for
acebutolol? Action: B1selective\nISA: yes (less bradycardia & minimal
change in plasma lipids )\nSedation: yes\nLipids: NA\nClinical Use: angina,
HTN
What is the action, ISA, sedation, effect on lipids, & clinical use for
atenolol?

Action: B1selective\nISA: no\nSedation: no\nLipids:

INCREASED\nClinical Use: angina, HTN


What is the action, ISA, sedation, effect on lipids, & clinical use for
esmolol?

Action: B1selective \nISA: no\nSedation: yes\nLipids:

no\nClinical Use: antiarrhythmic (Class IV); 10 minute T1/2


What is the action, ISA, sedation, effect on lipids, & clinical use for
labetalol?

Action: B nonselective + alpha1\nISA: no\nSedation:

yes\nLipids: no\nClinical Use: HTN & hypertensive crisis &


pheochromocytoma

What is the action, ISA, sedation, effect on lipids, & clinical use for
metoprolol? Action: B1selective\nISA: no\nSedation: yes\nLipids:
INCREASED\nClinical Use: angina, HTN, postMI antiarrhythmic
What is the action, ISA, sedation, effect on lipids, & clinical use for
pindolol?

Action: B nonselective \nISA: YES\nSedation: yes\nLipids:

no\nClinical Use: angina, HTN


What is the action, ISA, sedation, effect on lipids, & clinical use for
propranolol? Action: B nonselective \nISA: no\nSedation: YES\nLipids:
YES\nClinical Use: angina, HTN, postMI, migraine, tremor, performance
anxiety, thyrotoxicosis
What is the action, ISA, sedation, effect on lipids, & clinical use for timolol?
Action: B nonselective \nISA: no\nSedation: YES\nLipids:
YES\nClinical Use: HTN, migraine, glaucoma
What beta blockers are linked with worsening blood lipids? atenolol\n
metoprolol\n protranolol\n timolol
Which beta blocker does not cause ANY sedation (there is only 1)?
atenolol
Which 4 beta blockers have no affect on blood lipids?
acebutolol\nesmolol\nlabetalol\npindolol
Which 2 beta blockers are used to treat migraine?

propranolol\ntimolol

Which beta blocker is used to treat glaucoma?

timolol

Which 2 beta blockers are indicated as antiarrhythmics?


esmolol\nmetoprolol
Which betablocker is used in hypertensive crises?

labetalol (b/c also a A1

blocker)
Which is the only beta blocker not used for HTN?

esmolol (short T1/2 =

10 minutes)
What is the PANS & SANS innervation of the SA & AV nodes
M2 (decreases phase 4 slope by decreasing cAMP)\n\nSANS = B1
(increases phase 4 slope by increasing cAMP)

PANS =

What does activation of B1 heart receptors (& therefore an increase in


cAMP) cause?

Increase in upstroke velocity in pacemakers in

Increasing I (CL type)\n Decreasing action potential duration by


Increasing Ik\n Increases HR by increasing If, thus increasing slope of
phase 4
What does activation of M2 heart receptors (& therefore an decrease in
cAMP) cause?

Decreases upstroke velocity in pacemakers in

decreasing I (CL type)\n Increases action potential duration by


decreasing Ik\n Decreases HR by decreasing If, thus increasing slope of
phase 4\n Produces a K+ current (Ik/ACh), which slows diastolic
depolarization & decreases HR
What is the mechanism of Class IA antiarrhymics?

Decreases Vmax:

Blocks fast Na channels (decreases Ina) "state dependent" \n Increases


ADP (action potential duration) & ERP (effective refractory period: block of
K+ channels (decreased Ik, delayed rectifier current)
What is the MOA of Quinidine?

Class IA (Na+ channel blocker)\n Mbock:

increases HR & AV conduction\n\n SE: nausea & vomiting, cinchonism,


hypotension (due to alpha block), increased QRS & QT interval > syncopy
(torsades)\n\nInteractions: \n increases digoxin tox\n decreases effects
of AChE inhibitors in myasthenia\n hyperkalemia increases tox
What is cinchonism?

SE of Quinidine: GI, tinnitus, ocular dysfunction,

CNS excitation
What is the MOA of procainamide?

Class IA (Na+ channel blocker):\n

Less Mblock (less HR & AV conduction increase)\n Cardiodepressant\n


Prolongs ADP b/c is metabolized via Nacetyltransferase (genotypic
variation) to NAPA, that prolongs ADP\n SE: SLElike syndrome (30% of
patients) more likely in slow acetylators, hemotox (throbocytopenia &
agranulocytosis), CNS effects (dizziness, hallucinations, CV effects
(torsades)
What is "torsades de pointes"? ECG demonstrates a rapid, polymorphic,
ventricular tachycardia with a characteristic twist of the QRS complex
around the isoelectric baseline\n Can follow a prolonged QT intervals on

ECG\n Associated with a fall in arterial Bp, which can produce fainting\n
It can degenerate into ventricular fibrillation
What are the characteristics of "Trosades de pointes"?

Rotation of the

heart's electrical axis by at least 180\n Prolonged QT interval LQTS\n


Preceded by long and short RRintervals\n Triggered by an early
premature ventricular contraction (RonT PVC)
What is the treatment for "Torsades"? Withdrawal of the causitive agent
(can be class IA or class III antiarrhythmics)\n Infusion of magnesium
sulfate & antiarrhythmic drugs, & electrical therapy as needed\n\nBecause
of the polymorphic nature of torsades de pointes, synchronized
cardioversion may not be possible, and the patient may require an
unsynchronized shock (or defibrillation).\n[edit]
What electrolyte imbalances predisposed to torsades?

Diarrhea\n

Hypomagnesemia \n Hypokalemia\n\nT/F watch malnourished individuals


& chronic alcoholics
What are the signs of PCP (phycyclidine) use?
belligerance\nimpulsiveness\nfever\npsychomotor
agitation\nvertical & horizontal
nystagmus\ntachycardia\nataxia\nhomicidality\npsychosis\ndelirium
By what mechanism can PCP reintoxicate a patient during withdrawal?
When the PCP, trapped in an ionized form in the acidic gastric
lumen, is reabsorbed in the alkaline duodenum\n Check for normal or
small pupils
What is the postcocaine "crash"?

Withdrawal: severe depression,

hypersomnolence, fatigue, malaise, & severe psychological craving


What are the 4 mature defense mechanisms? sublimation\n altruism\n
humor\n suppression
Which viruses are associated with hepatocellular carcinoma?

HBV &

HCV
How are arbo & flaviruses transmitted?
Dengue fever\n Flav: Yellow fever

mosquito bites:\n Arbo:

What is the main function of the Golgi complex?

to add

oligosaccharides to proteins & lipds


What indicates that a protein is destined for the lysosomal pathway?
mannose6phosphate residuses
What are the 2 main enzymes in Peroxisomes?

1) Oxidases for

catabolic pathways (e.g. beta oxidation of longchain fatty acids)\n2)


Catalase to regulate H2O2 concentration to oxidize toxic substances
What is the main function of SER (smooth ER)?

Lipid biosynthesis \n

Membrane synthesis/repair \n Detox of metabolic byproducts\n\n NB:


no ribosomes!!
What is the main function of RER in the cell? Initiates the processing of
finalized protein products\n\nNB: Contains ribosomes
What should one do regarding the results of a study if the P value is
greater than 0.05? Accept the null hypothesis
What potential error are you making if you do NOT reject the null
hypothesis? Beta (type II) = 1power
What type of error does a P value represent? type I\n If P < 0.05, then
there is less than a 5% chance that the null hypotheses was incorrectly
rejected
What is a type I error?

Incorrectly rejecting the null hypothesis:

accepting a difference when none exists (i.e. the null is true)


What is a "CV wave"?

When, dues to tricuspid regurgitation, blood

flows backward into the atria during ventricular systole\n This pressure is
transmitted back into the RA & jugular vein & results in the JVC pressure
change: joining of the C & V wave
What sections of the nephron are responsible for Na reaborption?
Proximal Convoluted Tubule: 67%\nThick Ascending Limb:
25%\nLate Distal Convoluted Tubule: 3%\n\n1% = excreted (i.e. NOT
reabsorbed)

What type of tumors have tartrateresistant acid phosphatase as a


marker?

B lymphocyte neoplasms (e.g. hairy cell leukemia)

What cancers can be marked by CEA (carcinoembryonic antigen) in 70% of


cases?

colorectal & pancreatic

What is the marker for hepatocellular carcinoma?

Increased levels of

alpha fetoprotein
What type of tumor is marked by S100?

melanomas

What type of tumor produces bombesin (a tumor marker)?neuroblastomas


What is the pathogenesis of cor pulmonale? dysfunction of the RV due to
pulmonary hypertension in diseases affecting the lung or its vasculature
What can a chest Xray showing basilar hyperlucency in a young patient
indicate?

alpha1antitrypsin deficiency

What maternal disease can put a child at risk for transposition of the great
vessels?

Early fetal cyanosis due to maternal diabetes

What are congenital defects that can occur if a pregnant woman contracts
rubella (caused by Rubivirus, a togavirus)

PDA\n VSD\n cataracts\n

deafness
What infection can cause rapid death (within 3 weeks) in an AIDS patient
who presents with neurological deficits of speech, memory, & coordination
along with vision problems?

JC virus causing progressive multifocal

leukoencephalopathy (PML)\n causes multiple areas of demyelination


throughout the white matter of the CNS
What is a common viral casue of temporal lobe encephalitis in HIV/AIDS
patients?

Herpes simplex: rapid onset of fever & focal neurological

deficits due to damage of the temporal lobe\n memory problems,


personality changes, seizures
What is a common cause of pneumonia in HIV patients with a CD4+ count
below 200/mm3?
glass"

Pneumocystis jroveci\n Dx: Xray picture of "ground

What is a common cause of encephalitis in patients with HIV/AIDS with a


CD4+ cound below 100/mm3?

Toxoplasmosis\n S/S: seizures &

headache \n Xray: ringenhancing lesions with surrounding edema


What is the cellular mechanism of a type IV hypersensitivity reaction?
Initial exposure to antigen triggers differentiation of CD4+ T lymphocytes
into Thelper type 1 lymphocytes\n Reexposure = quick activation to
secrete cytokines that mediate the local inflammation
What are the 3 main mediators released in a type I hypersensitivity
reaction?

Histamine\n Leukotriene\n Prostaglandin

Describe the changes a female goes through during Tanner stages 15. 1:
Elevation of the breast papilla only & no pubic hair\n2: Small breast buds
with elevation of breast & papille with sparse, straight, downy hair on the
labila base\n3: Enlargement of the breast & areola with a single contour &
darker, coarse curled pubic hair\n4: Projection of the areola & papilla with
separate contous & adulttype public hair limited to the genital area\n5:
Mature breast & adult quantity/pattern of pubic hair that extends to the
thighs
When is organogesis?

Weeks 38

What are the findings of fetal alcohol syndrome?

1) pre & postnatal

developmental retardation\n2) microcephaly\n3) facial abnormalities\n4)


limb dislocation\n5) heart & lung fistulas\n\nMOA: inhibition of cell
migration
What fetal problems can occur if a mother takes chlorothiazide? fetal
thrombocytopenia & jaundice
What fetal problems can occur if a mother takes doxycycline?
Permanent tooth staining & enamel hypoplasia\n\nRx: Lyme disease
& genital infections
What fetal abnormalities are associated with lithium use? Ebsteins &
malformations of the great vessels
What fetal abnormalities are associated with Valproic acid use?
tube defects\ncleft lip\nrenal anomalies

neural

Which rash moves from the trunk outward? Which moves from the
extremities and moves inward? (Choices: Rocky Mountain & Typhus)
Rickettsia prowazekii (epidemic typhus): trunk >
extremities\n\nRocky Mountain: extremities > trunk
What is the treatment for Kawasaki's? IV IG & asprin
What are the physical exam findings in Kawasaki's? Strawberry
tongue\nLymphadenopathy\nBright RED cracked
lips\nConjunctivities\nSwollen hands/feet
What type of hematoma shows up as crescentshaped on CT?

Subdural

(usually bridging veins from cerebrum to dural sinuses in the elderly)


Where is the most common location of a Berry aneurysm? Posterior
communicating artery
What hematoma shows up as a biconvex disc on CT?

epidural:

usually middle meningeal artery due to trauma


What type of hematoma usually shows no signs on CT?
intraparenchymal due to atherosclerosis & occlusion in hypertensive
patients
What hormone is spironolactone similar to?

Progesterone\n T/F can

cause gynecomastia & erectile dysfunction


What are the 2 top causes of erosive gastritis?

H. pylori\nNSAIDs

What are the unique features of ZollingerEllison ulcers?

1) Unusual

location: beyond the 1st part of the duodenum\n2) Presense of MEN type I
(tumors in the parathyroid, pancreas, & pituitary)
What type of thalassemia is lethal in utero?

hydrops fetalis: \n no

functional alpha chains are made \n only Hb Bart's si made (y4 tetramer):
poor O2 delivery to peripheral tissues
What is Cooley's anemia? betathalassemia major (absence of both b
globin chains)\n S/S = severe hemolysis & ineffective erythropoiesis
What results from absense of the four alphaglobin genes?
Hemoglobin H\n microcytic anemia & mild hemolyisis

What microbial constituents do TLRs (tolllike receptors) recognize?

LPS (lipid A, core sugar, + variable carb chain O antigen)\n flagellin\n


unmethylated CpG DNA
What is the MOA of fluroquinolones?

Drugs: Ciprofloxacin,

Levofloxacin, Ofloxacin\n Inhibits DNA gyrase (topoisomerase II) that


relaxes DNA supercoils
What metabolic pathway produces 2,3 DPG (2,3diphosphglycerate)?

Anaerobic glycolysis\n 2,3DPG causes a DECREASED hemoglobin affinity


for O2 (= right shift of O2hemoglobin dissociation curve) so that more O2
is unloaded to tissues
What dopamine receptor antagonists are used as antipsychotics and cause
galactorrhea?

D2 blockers: (typical antipsychotics)\n

Chlorpromazine\n Thioridazine\n Haloperidol\n Fluphenazine


What is the MOA of amitriptyline?

TCA: prevents norepinephrine &

serotonin reuptake from synaptic cleft


What drug is a dopamine analog?

Bromocriptine\n agonist at

pituitary dopamine receptors & t/f prevents prolactin release\n Rx:


prolactinsecreting adenomas
What is the MOA of fluoxetine? SSRI\n Rx: depression
What is the MOA of Selegiline?

MOA B inhibitor that prevents the

breakdown of dopamine in the synaptic cleft\n Rx: Parkinson's


What cellular defect results in ChediakHigashi syndrome? Microtubule
polymerization = decreased phagocytosis
What is the MOA of cisplatin?

Alkylating agent" binds to guanine &

forms inter/intra strand crosslinks


Which prevents microtubule formation and which inhibits their breakdown:
paclitaxel or vincristine? Paclitaxel: hyperstabilizes polymerized
microtubules\n\nVincristine: prevents polymerization of microtubules

What type of bacterial infection can present at crepitus after a penetrating


wound?

Clostridium perfringens (gasproducing anaerobe)\n\nRx:

clindamycin
What 3 antibiotics are most associated with C. difficile overgrowth,
destruction of the colon's normal fora & pseudomembranous colitis?

Clindamycin\n Penicillin\n Cephalosporin


What antiviral is associated with ataxia, dissiness, & slurred speech?
Amantadine
What is the structure of a steroid hormone receptor?

Zinc finger

proteins: a polypeptide with a zinc atom bound to 4 cysteines\n Hormone


binding region + a DNA binding region that activates gene transcription
What are the 4 main actions of angiotension II?

Arteriolar

vasoconstriction\n Thirst sensation\n Increased ADH & aldosterone


secretion\n Increased Na reabsorption
What 5 tissues have angiotension II receptors?

Via angiotensin I

neuropeptide receptor (Gq _> increase IP3 & DAG bia phospholipase C
pathway):\n Vascular smooth muscle (vasoconstriction)\n CNS (increased
thirst)\n Zona glomerulosa of adrenal cortex (aldosterone secretion)\n
Renal tubules (increased Na reabsorption)
What type of receptors does ANP (atrial natriuretic peptide) work on?

GC

(guanylate cyclase) receptors \n with intrinsic GC activity to produce


cGMP\n to catalyze protein kinase G to phosphorylate serine & threonine
protein residues\n Result: arteriole vasodilation
Does insulin directly alter gene transcription? No, its second messenger
(MAP kinase) enters the nucleus to alter gene transcription:\n Insulin
binds to a receptor tyrosine kinase to autophyosphyrlate tyrosine within
the receptor\n Intracellular proteins bind the the SH2 of this
phosphotyrosine\n This activates MAP kinase (mitogenactivated protein
kinase) by covalent phosphorylation of tyrosine & threonine
What are the 2 receptormediated actions of vasopressin (ADH)? V1
receptors on arterioles:\n Activate Phospholipase C to liberate DAG & IP3
frome membrane lipids = vasoconstriction in SUPRAphysiologic

doeses\n\nV2 on principal cells of cortical & medullary collecting ducts: \n


activates G proteins that in turn activate adenylate cyclase (which
produces cAMP\n cAMP activates protein kinase A\n increased
expression of AQUAPORIN H2O channels in the collecting duct
What is the pathology of LambertEaton disease?

Autoimmune

antibodies against presynaptic voltagegated Ca channels & reduced


muscle use\n Result: reduced release of synaptic vesicles at prsynaptic
terminal b/c no Ca influx
When a muscle stimulation test shows initial unresponsiveness but
response after repetitive stimulation, what does the patient likely have?
LambertEaton: enough vesicles are released to cause activation on
the postsynaptic acetylcholine receptor.\n vesicle release is impaired in
this disease due to autoimmune attack of presynaptic votagegated Ca
channels\n only enough vesicles are released for post synaptic activation
after repetitive stimulation
What are the results of a muscle stimulation test that indicate myasthenia
gravis?

Fatigability in the test\n b/c they have reduced amounts of

acetylcholine receptors (due to autoimmune attack) to respond to the


neurotransmitter release (& their is no pathology of release)
What is the principle inhibitiory receptor of the CNS?

GABAa:

inhibitory Cl channel
What diseases are associated with EBV infection?

Endemic African

Burkitt's lymphoma\n heterophilPOSITIVE mononucleosis\n Oral hairy


leukoplakia (HIV patients)\n Hodgkin's & nonHodgkin's lymphomas in
immunocompromised patients\n Nasopharyngeal carcinoma
What infections are linked with Heterophilnegative mononucleosis?

CMV\n acute HIV\n Toxoplasmosis\n Herpies types 6 & 7


What are noninfectious states are associated with hepatocellular
carcinoma? Cirrhosis (2ndary to alcolol or crytogenic)\n
Hemochromatosis\n Aflatoxin ingestion\n Alpha1antitrypsin deficiency

What are the risk factors for gastric carcinoma?

H. pylori infection\n

atrophic gastritis\n postgastrectomy states\n achlorhydria\n pernicious


anemia\n Menetrier's disease\n Adenomatous polyps
What 3 carcinomas notoriously spread hematogenously?

Hepatocellular\n Renal Cell\n Follicular thyroid


Where does HCC often metastese to? Lung, portal vein, periportal nodes,
brain, bones
How does Hodgkin's spread?

Continguous manner

How does ovarian & appendiceal cancers spread?

Pseudomyxoma

peritonei: direct dissemination throughout the peritoneal cavity\n Diffuse


collection of gelatinous materials in the abdomen, peritoneal surface &
pelvis
What direction is DNA synthesized?

5'3' direction

What is the CD marker for pluripotent stem cells?

CD34: cells found in

the \n umbilical cord, bone marrow, endothelial progenitors of blood


vessels, mast cells and certain dendritic cells
What is the CD marker for hematopoietic stem cells that can differentiate
into erythroblasts & myeloblasts?

CD38\n Stimulated to differentiate

by graulocyte colonystimulating factor


What is the marker of antigenpresenting cells?

B7 protein

What results in the loss of phosphatidylinosital glycan A? Episodic acute


intravascular hemolysis = paroxysmal nocturnal hemoglobinuria\n this is
required for the anchoring of decayaccelerating factor on the surface of
RBCs, which BLOCKS compliment activation
Which type of hemophilia is most common in Ashkenazi Jews?

C: Factor

XI deficiency (AR)
What are the symptoms of Horner's syndrome?

Loss of sympathetic

innervation to one side of the face:\n1) Ptosis: eyelid drop\n2) Hemi


Anhidrosis: lack of sweating\n3) Miosis: pupil constriction

What lung tumor is associated with Horner's? Pancoast: apical tumor can
compress the 1st & 2nd thoracic nerve roots & t/f block sympathetic
innervation of the IPSILATERAL face
What nerve palsy leads to difficulty in lateral gaze? Abducens nerve palsy
(CN VI)
What nerve palsy/disruption can lead to drooping of the nasolabial fold?
Facial (CN VII)
What is the MOA of ganciclovir? Phosphorylated by viral protein
kinases \n Acts as an analog to dGTP & competitively inhibits the
incorporation of dGTP into viral DNA \n\nSE: myelosuppression &
nephrotox (MUST monitor Px)
What is the MOA of Enfurvirtide? Binds to the gp41 subunit of the HIV
viral envelop\n prevents entry of the virus into the host cell\n\nRx: HIV
therapy
What is the MOA of ritnovir?

Protease inhibitor: inhibits the cleavage of

viral polyproteins\n\nRx: HIV\nSE: GI & paresthesias


What is the MOA of oseltamivir? Inhibits viral neuraminidase & prevents
release of viral progeny from infected cells\n\nRx: influenza
What are the major side effects of Niacin treatment?

facial

flushing\n Gout\n Impaired glucose tolerance


What is the function of lipoprotein lipase?

degradation of VLDL

cholesterol\n upregulated by fibrates


Where are the BCRA1 & BCRA2 genes found? BRCA1 = Ch. 17\nBRCA2 =
Ch. 13\n\nBoth are gumor suppressor genes and both alleles must be loss
for the suppressive function to be lost
When does rigor mortis set in?

34 hours after death\n ATP is depleted &

without it, myosin cannot be released from the contraction complex\n


Muscles are frozen in a contracted in a contracted state

What is the mechanism of muscle relaxation after rigor mortis?

72

hours after death, enzymes are released (due to a drop in pH) & these
hydrolyze the myosinactin complex\n Result: muscle relaxation
What are the histological markers of diabetic nephropathy?

Increase in mesangial matrix by marked nodular accumulations


(KimmelsteilWilson nodules)\n Diffuse glomerulosclerosis
What is the pathogenesis of membranoproliferative glomerulonephritis?
Autoimmune disease of the young (830 years old)\n HISTO:
mesangial proliferation, thickening of peripheral capillary walls by
subENDOthelial depositis & mesangial interposition into capillary wall (=
TRAMTRACK)
What is the pathogenesis of rapidly progressive (crescentic)
glomerulonephritis?

extensive capillary damage:\n accumulation of

cells & fibrinous changes in Bowman's space \n= "crescent" on biopsy


What is the pathogenesis of focal segmental glomerulosclerosis? Nephrotic
disease\n young, hypertensive, Black men with risk factors: obesity &
HIV\n Injury to podocytes leads to mesangial, endothelia, & epithelial
cells & later shrinkage/collapse of glomerular capillaries > SCLEROSIS
(focal & segmental pattern)\n Hyaline casts on urinalysis
What causes cleft lip?

failure of the maxillary prominence to fuse with

the medial nasal prominence\n can be unilateral or bilateral


What is the marker for active viral replication of Hep B & likely
transmissibility?

HBeAg (detectable 24 months after exposure): marker

of the virus core


What Hep B marker indicates low trasmissibility?

HBeAb (detectable 5

months post exposure or 1 month after detection of HBeAg)


What Hep B marker shows former carriers or immunization?
(provides immunity)
What Hep B marker indicates chronic disease?

IgG HBcAb

What Hep B marker indicates recent disease?IgM HBcAb

HBsAb

What is gastroschisis?

Congenital abdominal wall defect: herniation of

abdominal organs not covered by a membrane as they are in an


omphalocele\n Complication of a patent urachus
What embryological structure becomes the median umbilical ligament?
urachus (a remnant of the allantosis)
What are the 2 main drug types that are oxidative and cause oxidative
stress (jaundice) in patients with G6PD deficiency? Sulfa drugs (e.g.
trimethoprimsulfamethoxazole)\n Antimalarials
What are the symptoms of HYPERcalcemia? Constipation\n Poly
uria/dipsia\n HYPOreflexia\n Lethargy
What diuretic drugs are useful to treat HYPERcalcemia?

Loop Diuretics

(Furosemide):\n Block Na+/K/2Cl transporter in thick ascending loop of


Henle\n Blocks both Na & Ca reabsorption
What diuretic class is useful to treat idiopathic hypercalciuria?

Thiazides

(hydrochlorothiazide):\n Inhibits Na/Cl cotransporter in the distal


tubule\n Decreases Na reabsorption & increases Ca reabsorption
What are Auer rodes?

Fused lysosomal granules\n Cytoplasmic

inclusions in myelocytic precursor cells\n Indicative of acute


promyelocytic leukemai (M3 variant of AML)
What happens if there is a sudden release of Auer rods due to rapid
treatment of acute promyelocytic leukemia? DIC & fatal hemorrhage\n
Histo: Helmetshaped cells & Schistocytes
What are schistocytes and when are they often found?

irregular or

fragmented RBCs\n result from attempts to squeeze through the fibrin


meshwork of small vessel thrombi (e.g. in DIC)
What are acanthocytes (aka "spur cells)?

Spiny RBCs \n Aassociated

with abetalipoproteinemia & severe liver disease


What are Burr cells (aka echinocytes)? Abnormal RBCs with short, blunt
projections around the periphery\n Found in: hemolyticuremic syndrome,
pyruvate kinase deficiency, ureamia...

What are teardrop cells? Abnormal RBCs seen in myeloid metaplasia with
myelofibrosis
How do hormones produced in the liver enter the systemic circulation (i.e.
by which vessel)? Hepatic Vein to the IVC
What vessel carries O2rich blood to the liver?

Hepatic artery

Describe the vessel flow through the hypophyseal portal system. Blood
supplied by the hypophyseal artery goes through the capillary bed
supplying the hypothalamic nuclei (which secretes tropinreleasing
hormones that affect the anterior pituitary)\n Then blood passes through
the capillary bed that supplies the anterior pituitary
What vessel delivers blood from the GI track to the liver? Portal vein
What vessel provides venous drainage from the kidneys to the IVC?
Renal veins
What does dermatitis herpetiformis look like? What disease does it
indicate?

multiple, nonblanching, purpuric, grouped lesions located

on the upper & lower extremities\n Celiac disease (glutensensitive


enteropathy) due to gladin allergy & crossreactivity of antibodies to the
small intestine villi
What is the histological picture of Celiac disease?

Lymphocytic

infiltrate into the epithelial layer\n Atrophic & blunted vili


When does Celiac disease present?

After the introduction of cereals

into the diet


What is the histological picture in lactose intolerance?
What is the pathogenesis of Whipple's disease?

BENIGN mucosa

Tropheryma whippelii

(rodshaped bacili) invade macrophages:\n Distended macrophages block


lymphatic blockage & lipid deposition\n S/S: arthropathy, diarrhea, &
weight loss
Which presents with relapsing blood diarrhea, Chrohn's or Ulcerative
colitis?

UC, along with mucus, abdominal pain, & cramps

What inhibitor of peptidoglycan synthesis is only used in topical form?


Bacitracin\n MOA: prevents transfer of mucopeptides into the
bacterial cell wall\n Rx: gram, gram+ bac via a mild skin cut
What is the primary secretory product in 55% of cases of multiple
myeloma?

IgG

What uncommon form of lymphoma results in plasma cells that secrete


monoclonal IgM paraproteins?

Waldenstrom's macrogloulinemia

What is the function of histamine?

To promote vasodilation\n To

promote leukocyte estravsation


What do these S&S indicate: chorea, fever, polyarthritis, valvular damage?
Rheumatic heart disease\n can cause congestive heart failure due
to mitral &/or aortic valve damage\n Histo: Aschoff bodies =
noncaseating granulomas with multinucleated giant cells
What histological finding is common in elderly patients with aoritic
stenosis?

degenerative calcification

What histological finding is associated with mitral valve prolapse?


myxoid degeneration
What is "contraction alkalosis"? When there is rapid loss of bicarbfree
fluids (e.g. stomach contents, urine) = increase in plasma [bicarb]\n Body
response: stimulation of the reninangiotensionaldosterone system:\n1)
angiotensionmediated increase in H+ secretion via the antiporter in the
proximal tubule\n2) aldosteronetriggered influx of Na & H2O with an
efflux of K & H+ in the distal tubule\nRESULT = metabolic alkalosis
What is the parentral rout of transmission?

Sexual activity\n Blood

transfusions\n Transplacental\n\n(Think: HIV)


What immune system deficiency most often leads to a susceptibility to
Neisseria infection?Complement component deficiency
What are 1/2 of viral cases of myocarditis caused by?

Coxsackie B

(icosahedral, Picornaviridae)
What sign of hypoglycemia do betablockers mask? tachycardia

What happens when the tubule & glomerular basement membranes are
nonenzymatically glycosylated? Diabetic neptropathy:\n increase
permeability to proteins\n cytokin release causes glomerular
hypertrophy\n HISTO: diffuse mesangial expansion in the glomeruli &
later nodular glomerulosclerosis (KimmelsteilWilson nodules)
What features characterize nodular glomerulosclerosis?

Increased

cellularity\n Mesangial matrix deposition\n Hyaline masses & thickening


of the lamina densa\n\nDiabetic nephropathy
What is the histological picture of lupus nephropathy?

wireloop

glomeruli\n subendothelial basement membrane deposits


What is the most common cause of abruptio placentae?

Rupture of

defective maternal vessels in the decidua basalis\n RF: hypertension,


cocaine use, smoking, uterine fibroids, advanced age, sudden uterine
decompression, premature rupture of the membranes, & bleeding
diathesis
What causes Placenta accretia? placental villi attach directly to the
myometrium due to a defect in the decidua basalis layer\n @ delivery,
there is incomplete separation of the placenta\n can cause severe
postpartum hemorrhage
Define labor. Regular uterine contractions that result in cervical change
Define preterm labor.

Labor that occurs between 20 & 37 weeks of

gestation.
What are the components of prokarytoic ribosomes?

Large (50S)

subunit\n 5S rRNA molecule + 23S rRNA molecule\n\nSmall (30S)


subunit\n 15S rRNA molecule\n\nAll rRNA are bound to & protected by the
ribonucleoprotein component of the ribosome
What is the best treatment for "thyroid storm"?

1st: primary

stabilization = propranolol\n2nd: propylthiouracil or methimazole to inhibit


endogenous synthesis of thyroxine

Which is better during thryotoxicosis & thyroid storm: aspirin or


acetaminophen?

Aceaminophin: antipyretic\n\nNOT asprin b/c it

displaces throxine from thyroidbinding globulin & worsens symptoms


What are the earliest visible manifestations of endometriosis?

Whitish

peritoneal plaques
When whitish periotneal plaques are found along the uterosacral ligament,
where is the endometriosis?

pelvic culdesac\n complications =

adhesions to bowel can cause obstruction


Why does a metabolic acidosis often result in hyperkalemia?

Increased H+ ions are used to balance Cl in the urine\n This prevents K+


secretion by the H+/K+ exhcanger in the distal tubule\n\nRx: loop diuretic
(e.g. furosemide) to increase Na & K excretion in the loop of Henle
What is the most common of inherited aplasic anemia?

Fanconi's

anemia = defective DNA repair \n radial ray abnormalities can result in


abnormal or absent radii or thumbs\n Other signs: kidney malformations,
hypogonadism, microcephaly, high fetal hemoglobin
What is DiamondBlackfan anemia?

Congential RBC aplasia caused by

primary failure of erythroid progenitor cells in the bone marrow


What is lacking in alkaptonuria patients?

homogentisic oxidase\n= no

effective tyrosine degradation (homogentisic acid to maleylacetoacetic


acid)\n homogentisic acid builds up in urine\n when urine is exposed to
alkali or O2 = DARK BLACK\n S/S = usually benign or presents with
arthritis
What enzyme is lacking in Maple syrup urine disease?

alphaketoacid

dehydrogenase, which catabolizes branched amino acids (valine, leucine,


isoleucine)
What is cystinuria? a defect of amino acid (cystine) transport in the renal
tubules\n can result in cystine stone formation
What are the symptoms of homocystinuria?

Mental retardation\n

Spinal abnormalities (kyphosis, Marfanoid habitus)\n Atherosclerosis\n

Lens subluxation\n\nMO: defect in conversion of methionine to cysteine\n


3rd decade of life = thromboembolic events increase
What is the treatment for primary syphilis?

benzathine penicillin G (long

acting): 1, intramuscular dose


What is the treatment fo Chlamydia or Haemophilus ducreyi?
Azithromycin (1 oral dose)
What is the most common fetal neoplasm?

sacrococcygeal teratoma

(multiple tissue types = remnant of the primitive streak)


What are the 2 types of MSUD (maple syrup urine disease)?
Intermittent: presents during times of catabolism (e.g. after
infections)\n\nClassical: ketonuria 48 hrs to 1 week after birth
What CSF finding is found in 8090% of MS (multiple sclerosis) patients?
Increased Ig levels: reflects presence of intrathecal humoral
immune activation
What CSF finding is the hallmark of GuillainBarre syndrome?
albuminocytologic dissociation: greatly increased protein
concentration with only a modest increase in cell count\n\n MOA: acute
demyelination of peripheral nerves (acute ascending paralysis) after a
viral infection or gastroente ritis from C. jejuni infection
What is the mechanism of basal ganglia destruction in Huntington's

gutamate toxicity of the ganglia in the caudate & putamen\n gliosis &
neuronal depletion result in a loss of motor inhibition = chorea & athetoid
movements
What causes neurofibrillary tangles?

Hyperphosphorylated tau

protein\n Characteristic of Alzheimer's disease


What is the treatment for phenylketonuria?

Dietary\n Avoid

phenylalanine\n Suppliment tyrosine (b/c the phenylalanine cannot be


converted to tyrosine)
What is the most common cause of pneumonia in malnourished,
debilitated, or alcoholic patients?

Klebsiella pneumoniae (gram ,

rod)\n Xray: diffuse infiltrates (NOT lobular)

What three "drugs" have zeroorder elimination?

1) ethanol\n2)

phenytoin\n3) highdose asprin


What is lateral medullary syndrome (aka posterior inferior cerebellar
artery (PICA) syndrome or Wallenberg's syndrome)? A Stroke Syndrome:\n
Symptoms: numbness of the ipsilateral face & contralateral limbs,
diplopia, dysarthria, & ipsilateral Horner's\n Cause: disruption of PICA
blood supply to dorsolateral quadrant of medulla, including nucleus
ambiguus & inferior surface of the cerebellum
What happens when the posterior inferior cerebellar artery (PICA) is
infarcted?

Lateral medullary syndrome due to dorsolateral quadrant

disfunction:\n Tract of CN V = facial pain\n Vestibular nuclei =


dysequilibrium\n Nucleus ambiguus = palate problems & hoarse voice\n
Spinothalamic tract = contralateral pain & temperature loss\n
Descending sympathetic fibers = ipsilateral Horner's
What is the 2nd most common brainstem stroke syndrome?

Ischemia

to the Anterior inferior cerebellar artery:\n Caudal lateral pontine


tementum (including spinal tegmental tract of CNV)\n Inferior surface of
the cerebellum\n\n Symptoms: ipsilateral deafness (labyrinthine artery to
cochlea & vestibular apparatus), ipsilateral facial weakness, ataxia
What is damaged in a lacunar stroke? Lateral striate arteries (penetrating
branches of the MCA):\n internal capsule, caudate nucleus, putamen, &
globus pallidus
What structures are damaged in a symptomatic internal carotid artery
occlusion?

lateral geniculate body\n globus pallidus\n posterior limb

of the internal capsule


Which results in tetany: HYPO or HYPER calcemia?

Hypocalcemia

(remember DiGeorge's symptoms)


What types of infections are you prone to if you have a T lymphocyte
deficiency? Viral & Fungal
What organism causes Hemolyticuremic syndrome & what lab values
characterize this problem?

E. coli O157:H7 (gram , rod, lactose

fermenting)\n Low platelet count, anemia, renal failure (e.g. uremia)

What are the symptoms of rheumatic fever? Fever\n Migratory


polyarthritis\n Carditis\n\n Can follow a Group A strep pharyngitis
What are the signs of bacterial endocarditis? Fever\n New Murmur\n
Janeway lesions\n Nail bed hemorrhages
What is the most common cause of acute & subacute endocarditis?

Acute: Staph aureus (1020? of endocarditis)\n SubAcute: viridans strep


(usually attacks previouslydamaged valves); causes 5060% of
endocarditis
Where does Pemphigus vulgaris attack?

Pathogenic antibodies are

directed against dsmoglein3, a cellcell adhesion protein expressed by


epidermal keratinocytes\n Result: intraepidermal acantholysis with
sparing of the basal layer\n Nikolsky's Sign: flaccid epidermal bullae that
easily slough off leaving large denuded areas of skin (prone to
infection)\n\nRx: steroids
What is lacking in patients with essential fructosuria?

BENIGN

frutosuria b/c fructose does not enter cells & does not deplete cellular
phosphate\n fructokinase: converts frutose to fructose1phosphate
What is deficient in patients with hereditary fructose intolerance?
Aldolase B (aka fructos1.6bisphosphate aldolase): converts
fructose1phosphate to dihydroxyacetonephosphate \n fructose1
phosphate accumulates in cell \n Presentation: jaundice, hepatomegaly,
vomiting, lethargy, convulsions, & hypoglycemia after ingesting
fructose\n\nRx: AVOID fructose & sucrose (glucose + fructose)
What dietary recommendation is made for patients with McArdle's?
High protein + creatinine
What is the action of endogenous opioid peptides on the GI?

Stimulate smooth muscle contraction \n Inhibit intestinal secretion of


fluids & electrolytes\n\n(Morphine has these properties as well)
What is the relationship between prevalence and the PPV of a test?
disease prevalence decreases, the probability of a positive being a true
positive decreases

As

Rank infectious agent types in order of increasing susceptibility to


chemical sterilants.Least to Most:\n1) prions\n2) spores\n3)
Myocbacteria\n4) Nonenveloped viruses\n5) Fungi\n6) Gram+ bacteria\n7)
Gram bacteria\n8) Enveloped viruses
What genetic disease results in skin that readily breaks & forms blisters
with minor trauma?

Epidermolysis bullosa\n Caused by a mutation

in either keratin 14 or 5 in basal epithelial cells


What is the classical triad of Pott's disease?

Spinal Pain\n Kyphosis

(compromised vertebra)\n Neurologic signs (radicular pain to cord


compression & paralysis)
For what systemic infection is amphotericin the DOC?

Disseminated

mycoses:\n blastomycosis\n cryptococcus\n histoplasmosis


What is the most common cause of a paraspinal abscess? Staphylococcus
aureus\n\nRx: IV vancomycin
What lab findings indicate Paget's disease of bone? Elevated Alkaline
Phosphatase + normal micronutrient levels
what are 4 typical uses of sulfa drugs, like trimethoprim? 1) UTI\n2)
Nocardiosis\n3) Toxoplasmosis\n4) Prophylaxis for recurrent otitis media
What sulfa drug perscription should be combined with folic acid
supplementation? Timethoprim\n\nWhy? To reduce megaloblastic anemia,
leukopenia, & granulocytopenia
What property of quinolones (like Gatifloxacin) makes them particularly
useful for treating UTIs?

Concentrated in the urine\n\nMOA: inhibit DNA

gyrase during bacterial replication & growth


What drug class is toxic to cartilage?

fluoroquinolones: can lead to

tendinitis & tendon rupture\n e.g. gatifloxacin


What 5 main problems are macrolides (e.g. erythromycin) used to treat?
1) Gram+\n2) Legionnaire's\n3) Syphilis\n4) Mycobacterial
pneumonia\n5) Corynebacterial infections (e.g. diptheria)

What broadspectrum antibiotic is particularly effective for meningitis?


chloamphenicol\n\nSE: aplastic anemia & gray baby syndrome
What are two genetic/anatomical risk factors for testicular cancer?

Klinefelter's (47, XXY): usually will be extragonadal, nonseminamatous


testicular cancer\n Cryptorchidism
What 2 cell types kill target cells through the release of preformed
granules?

1) NK cells: perforin & granzymes\n2) CD8+

What drug causes increased vessel permeability and subsequent edema in


the face, lips, mouth, and subglottic tissue?

ACE inhibitors:\n block the

2ndary action of ACE on the degradation of bradykinin\n High levels of


bradykinin produce these symptoms
What are the 2 main functions of prostacyclin?

Produced by vascular

endothelial cells from PGH2:\n1) prevents platelet aggregation\n2) potent


vasodiation
What causes watery, foulsmelling stool with leukocytosis following
antibiotic treatment?

C. difficile superinfection\n\nRx: metronidazole

What are the pseduomembranes of C. difficile psuedomembranous colitis?


inflammatory exudates on the injured mucosa due to the to protein
exotoxins (A & B) of C. difficile
How does S. aureus cause food poisoning?

heatlabile & heatstable

exotoxin secretion
What is the virulence factor of Clostridium perfringens?

produces

lecithinase that breaks down cell membrane lecithin (& produces gas
bubbles)
How does Yersinia enterocolitica cause mesenteric adenitis?

invades

Peyer's patches of the gut (& 2ndary fever & diarrhea)


What is the inhibitor (regulator) of phosophofructokinase & t/f of the rate
limiting step of glycolysis?

citrate (from the Krebs Cycle)

What regulates the Krebs cycle by providing negative feedback to


pyruvate dehydrogenase?Acetylcoenzyme A (produced from pyruvate by
pyruvate dehydrogenase)
What amino acids carries amino groups from the muscle to the liver?
alanine (can also be convered to pyruvate to be used in the Krebs
cycle & can also inhibit pyruvate kinase)
What enzyme produces glucose6phosphate in the 1st step of glycolysis?
1) In all tissues: hexokinase\n2) Liver ONLY: glucokinase
Which antidiarrheal is an opiate analog & derivative of haloperidol?
Loperamide: binds GI opiate receptors & inhibits bowel motility
How does aluminum chloride decrease GI motility? Osmotic & Ionic
effects
How does Bismuth subsalicylate work? Binds to the ulcer base & provides
physical protection\n Also allows HCO3 secretion to reestablish the pH
gradient in the mucous layer
What is the MOA of Kaolin?

An absorbent that binds toxic compounds

from the GI wall & promotes their excretion


What is the use of a betablocker in patients with chronic liver disease?
Treats their portal hypertension
Why do loop diuretics cause a loss of potassium?

B/C loops block Na

absorption in the thick ascending loop of Henle, the nephron attempts to


absorb more Na in the distal tubule & collecting duct. In the collecting
duct: \n1) K+ & H+ are exchanged by the intercalated cells\n2) Na & H2O
are absorbed by principle cells & K+ is secreted
How should salmonella gastroenteritis be managed?

1) Treat

dehydration with fluid & electrolyte replacement \n2) Give antibiotics if


there is evidence of bacteremia or focal infection. Otherwise, antibiotic
treatment will increase rates of relapse
What are the most common risk factors for ectopic pregnancy?
prior appendicitis\n3) endometriosis\n4) prior abdominal surgery

1) PID\n2)

List the 5 stages of neutrophil development. 1) Promyelocytes (with


primary granules)\n2) Myelocytes (with 2ndary granules)\n3)
Metamyelocytes (with more prominent indented nuclei)\n4) Band (stab)
cells (horseshoeshaped, but not yet lobulated)\n5) Mature neutrophils
(with clearly lobulated nuclei)
What is internuclear opthalmoplegia? 1) ispsilateral medial rectus palsy
on attempted lateral conjugate gaze away from the lesion\n2) monocular
horizonal nystagmus in the contralateral abducting eye\n3) Preserved
convergence\n\nCause: damage to the ipsilateral medial longitudinal
fasciculus: the connection between the abduct & oculomtor nerve, where
their actions become UNLINKED\n PEDS: CNS infection or MS\n Adults:
vascular disease
What does CN III innervate?

All extraocular muscles, except the lateral

rectus (CN VI) & superior oblique (CN IV)\n Lesion: affected eye looks
"down & out" when at rest (abducted & depressed)
What 3 diseases can cause Argyll Roberson pupil?

1) Neurosyphilis\n2)

Diabetes\n3) SLE\n\nPupillary lightnear dissociation: absent miotic


reaction to light with preserved accommodation
What is Marcus Gunn pupil and what causes it?

Relative afferent

pupillary defect\n\nCause: lesion in the afferent limb of the pupillary light


reflex \n MS: retrobulbar neuritis
What histological change in the CNS is often found in SIDS autopsies?
Astrogliosis: nonspecific response to injury to the brain stem &
cerebellum\n Hypertrophy of astrocytes \n Increase in glial fibrillary
acidic protein immunostaining
What are the histo findings in muscle tissue of patients that suffer from
ALS (amyotrophic lateral sclerosis)?

Fiber type grouping consistent

with reinnervation\n Small angular fibers consistent with deinervation &


neurogenic atrophy
What are the physical exam findings in a child with Kawasaki's?
Fever\n Conjunctivitis\n Erythema in the oral mucosa\n Cervical
lymphadenopathy

What is the typical clinical presentation of a patient with Buerger's?


Patient (2040 years, heavy smoker)\n Intermittent claudication\n
Superficial nodular phlebitis\n Cold Sensitivity
What are the physical findings in a patient with PAN (polyarteritis nodosa)?
Cottonwool spots on the retina\n Microaneurysms\n Palpable
purpure on the skin\n(+ history of Hep B)
What is the histo hallmark of PAN (polyarteritis nodosa)?

transmural

inflammation of the arterial walls in kidneys, PNS, & GI tract\n\nLAB: +


perinuclear antineutrophilic cytoplasmic antibody (against
myeloperoxidase)
What is the histo characterization of Wegener's?

Necrotizing

granulomas in the lung & upper airways + glomerulonephritis\n\nLAB: +


antineurophilic cytoplasmic antiboby
What is the treatment for Lyme disease?

oral doxycycline

What are the classical symptoms of Cooley's anemia?

severe beta

thalassemia:\n Hemolytic anemia\n Hepatosplenomegaly\n "Chipmunk


facies" due to extramedullary hematopoiesis in facial bones)\n\nLAB:
increased HbF & HBA2 (neither require B chains), absense of HbA1 shows
that there is NO beta chains \n\nRIP: cardiac failure 2ndary to
hemochromatosis
Which Thalassemia confers a mild protective effect against Plasmodium
falciparum malaria?

Beta Thalassemia minor (heterozygous defect):

b/c of shortened lifespan of RBCs\n\nLAB: decreased HBA1


What is the geographical distribution of thalassemias?

beta:

Mediterranean\nalpha: Asia / Africa


What is Kartagener's syndrome? AR disease with this Triad:\n1) Chronic
sinusitis (& otitis media)\n2) Bronchiectasis (chronic bronchitis & recurrent
pneumonia)\n3) Situs inversus\n\nPatho: poorly functioning dynein arms
on all cilia in the body\n\nPresentation: unremitting asthma & sinus
pressure headaches

What are the functions of actins in cells?

Composition: multiple

globular Gactin subunits\n\n Tracts for myosins: cell motility, vesicular


transport, or muscle contraction
What are the cellular functions of myosins?

Composition: ATPdriven

dimeric molecular motor proteins\n\n Move along actin filaments for


vesicular transport, cell motility, & muscle contraction
What is glyburide's MOA as a type II diabetic drug? Sulfonylurea:
stimulates release of endogenous insulin to reduce blood sugar levels
What class of drugs increase target cell sensitivity to insulin & decrease
hepatic gluconeogenesis? Thiazolidinedines:\n Glitazone\n
Rosiglitazone\n Pioglitazone
What drugs inhibit the intestinal brush boarder enzyme alpha
glucosidase? alphaGlucosidase inhibitors (acarbose)
What is the inherited form of hyperammonemia that presents with mental
retardation, seizures, and finally death?

Ornithine

transcarboamolyase \n Normal enzyme function: combines carbamoyl


phosphate & ornithine to make citrulline in the urea cycle
What storage disease presents with generalized hypotonia, muscle
weakness, & hypertrophic cardiomegaly?

Pompe's (absent lysosomal

alpha1,4 glucosidase)\n\nLAB: significantly elevated serum creatinine


kinase
What enzyme defect presents with vomiting, diarrhea, failure to thrive &
hypotonia within the 1st few days of life?

galactose 1phosphate

uridyltransferase galactosemia\n galactose 1phosphate is toxic to renal,


hepatic, & neuronal cells\n\nRx: galactose restriction. NB: patients may
STILL have developmental delay, ataxia, & apraxia
What are the 2 effects of an aldose B dysfunction? No splitting of fructos
1phosphate: \n1) Inhibition of glucose production\n2) Depletion of
adenosine triphosphate
What enzyme deficiency presents with angiokeratomas, hypohidrosis,
corneal & lenticular opacities, acroparesthesias, and vescular disease of

the kidney, heart & brain?

Fabry's: mutations in alphagalactosidase

A gene > accumulation of ceramide trihexoside\n\nRx: enzyme


replacement therapy
What foods should patients with hereditary fructose intolerance avoid?
fructose, sucrose, sorbitol
What are the typical clinical symptoms of ureteral or bladder calculus?
Sharp, intermittent, excruciating pain in the lower back, abdomen, or
testicular region\n+ Fever, nausea, vomiting, & hematuria
In a woman with symptoms of acute appendicitis, what hormone should be
measured?

Rule out ectopic pregnancy wiwht a betahCG level test

What is Meniere's disease?

IDIOPATHIC DISEASE WITH:\n Episodes of

vertigo lasting for a period of hourse with associated fluctuation\n


Progressive lowfrequency sensorineural hearing loss \n Aural fullness or
pressure (tinnitus)\n\nCause: increase in the volume of the endolyphatic
system (hydrops) 2ndary to malfunction of the endolymphatic system,
which is responsible for the filtration/excretion of endolymoph in the
membranous labyrinth of the inner ear
What causes BPPV (benign paroxysmal positional vertigo)?

Freely

moving crystals of Cacarbonate within the semicircular canals cause


seconds of vertigo & nystagmus
What parts of the inner ear react to acceleration & deceleration of the
head? utricle & saccule
What lesion results in impairment in the highfrequency range of hearing
& problems with speech discrimination?

Cranial Nerve VIII

(vestibulocochlear)
What 3 conditions are associated with elevated betahuman chorionic
gonadotropin levels?

1) Pregnancy\n2) Choriocarcinoma\n3) Complete

hydatidiform mole
What 2 diseases are associated with PANCA elevation?

1) Microscopic

polyangiitis\n2) ChurgStrauss syndrome: allergic rhinitis, asthma, &


eosinophilia

Which gyrus is the primary motor cotex?

precentral: top = feet,

bottem = face
What is the UMN innervation of the face?

Upper 1/3: bilateral\n

Bottom 2/3: contralateral primary motor cortex


What is the lateral collateral ligament complex?

Ankle joint ligment

complex that protects from inversion stress:\n1) anterior talofibular


ligament\n2) calcaneofibular ligament\n3) posterior talofibular ligament
What prevents eversion stress on the ankle? deltoid ligament
What ligament is most commonly affected in ankle sprains?

anterior

talofibular ligament (usually protects against inversion stresses)


What hormone acts on the hypothalamic thermregulatory center & slightly
elevates basal body temp after ovulation?

progesterone

What is given as "rescue therapy" due to cyclophosphamide induced


hemorrhagic cycstitis due to the accumulation of toxic metabolites in the
urine? Mensa: converts some of the toxic metabolites (e.g. acrolein) into as
less toxic form to be secreted in the urine
What are the clinical signs of a pineoblastoma?

headache \n

drowsiness &/or vivid dreams due to excessive melatonin production,


which regulates sleepwake cycles\n\nMRI: contrastenhancing pineal
mass located between the thalamic bodies
What is the classical sign of a craniopharyngioma? visual disturbances b/c
this pituitary stalk tumor compresses the optic chiasm
What are the 2 classical signs of a pituitary adenoma?

hyperprolactinemia\n bitemporal hemianopia (compresses the optic


chiasm)
How does initial ICP present?

headache, nausea, vomiting

What are the 2 mechanisms of iron absorption?

1) Hemeassociated

iron is taken up by heme transporter in the luminal plasma membrane of


the duodenal epithelial cell. \n2) Free ferric iron ions are converted to
ferrous iron ions by a cytochrome B enzyme on the luminal plasma

membrane of the duodenal epithelial cell. Ferrous iron ions are taken up
by the DMT channel. W/in the cell, iron is transferred to mucosal ferritin &
then shuttled to transferrin in the plasma
What glycogen storage disease occurs in adolescence?
What disease is a mild form of vonGierke's?

McArdle's

Cori's: deficiency in alpha

1,6glucosidase (glycogen debranching enzyme)\n With NORMAL blood


lactate levels!

You might also like